Combo with Module 7 and 2 others

¡Supera tus tareas y exámenes ahora con Quizwiz!

"The nurse is planning the care of a client diagnosed with lower esophageal sphincter dysfunction. Which dietary modifications should be included in the plan of care? 1. Allow any of the client's favorite foods as long as the amount is limited. 2. Have the client perform eructation exercises several times a day. 3. Eat four (4) to six (6) small meals a day and limit fluids during mealtimes. 4. Encourage the client to consume a glass of red wine with one (1) meal a day

"1. The client is instructed to avoid spicy and acidic foods and any food producing symptoms. 2. Eructation means belching, which is a symptom of GERD. 3. Clients should eat small, frequent meals and limit fluids with the meals to prevent reflux into the esophagus from a distended stomach.CORRECT 4. Clients are encouraged to forgo all alcoholic beverages because alcohol relaxes the lower esophageal sphincter and increases the risk of reflux

Which assessment data support the client's diagnosis of gastric ulcer?"1. Presence of blood in the client's stool for the past month. 2.Complaints of a burning sensation that moves like a wave. 3.Sharp pain in the upper abdomen after eating a heavy meal. 4.Comparison of complaints of pain with ingestion of food and sleep

"1. The presence of blood does not specifically indicate diagnosis of an ulcer. The client could have hemorrhoids or cancer that would result in the presence of blood. 2. A wavelike burning sensation is a symptom of gastroesophageal reflus. 3. Sharp pain in the upper abdomen after eating a heavy meal is a symptom of gallbladder disease. 4. (CORRECT) In a client diagosed with a gastric ulcer, pain usually occurs 30-60 minutes after eating, but not at night. In contrast, a client with duodenal ulcer has pain durin ghte night that is often relieved by eating food. Pain occurs 1-3 hours after meals

In planning care for the patient with Crohn's disease, the nurse recognizees that a mojor difference between ulcdrative colitis and Crohn's disease is that Crohn's disease "a. frequently results in toxic megacolon b. causes fewer nutritional deficiencies than does ulcerative colitis C. Often recurs after surgery whereas UC is curable with colectomy d. is manifested by rectal bleeding and anemia more frequently than is ulcerative colitis

"4. Correct answer: c Rationale: Because there is a high recurrence rate after surgical treatment of Crohn's disease, medications are the preferred treatment

The pernicious anemia that may accompany gastritis is due to which of the following? a. Chronic autoimmune destruction of cobalamin stores in the body b. Progressive gastric atrophy from chronic breakage in the mucosal barrier and blood loss c. A lack of intrinsic factor normally produced by acid-secreting cells of the gastric mucosa d. Hyperchlorhydria resulting from an inrease in acid-secreting parietal cells and degradation of RBC's

"Correct answer: c Rationale: Gastritis may cause a loss of parietal cells as a result of atrophy. The source of intrinsic factor is also lostthe loss of intrinsic factor, a substance essential for the absorption of cobalamin in the terminal ileum, ultimately results in cobalamin deficiency. With time, the body's storage of cobalamin is depleted, and a deficiency state exists. Because it is essential for the growth and maturation of red blood cells, the lack of cobalamin results in pernicious anemia and neurologic complications

"The nurse explains to the patient with gastroesophageal reflux disease that this disorder: "A. results in acid erosion of the esophagus caused by frequent vomiting B. Will require surgical wrapping of the pyloric sphincter to control the symptoms C. Is the protrusion of a portion of the stomach into the esophagus through the opening in the diaphragm D. Often involves relaxation of the lower esophageal sphincter, allowing the stomach contents to back up into the esophagus

"right answer: d Rationale: Gastroesophageal reflux disease (GERD) results when the defenses of the esophagus are overwhelmed by the reflux of acidic gastric contents into the lower esophagus. An incompetent lower esophageal sphincter (LES) is a common cause of gastric reflux

• The overall goals are that the patient with diarrhea will have

(1) no transmission of the microorganism causing the infectious diarrhea, (2) cessation of diarrhea and resumption of normal bowel patterns, (3) normal fluid and electrolyte and acid-base balance, (4) normal nutritional status, and (5) no perianal skin breakdown.

"Which assessment data support the client's diagnosis of gastric ulcer? 1.Presence of blood in the client's stool for the past month.2.Complaints of a burning sensation that moves like a wave.3.Sharp pain in the upper abdomen after eating a heavy meal.(4).Comparison of complaints of pain with ingestion of food and sleep

(4) In a client diagnosed with a gastric ulcer,pain usually occurs 30-60 minutes after eating, but not at night. In contrast, a client with a duodenal ulcer has pain during thenight that is often relieved by eating food.Pain occurs 1-3 hours after meals

What is the initial phase of acid secretion?

*The initial phase (cephalic phase) starts with the sight and smell of food. Mediated by Acetylcholine, the neurotransmitter of the Vagus nerve.

• Symptoms of lactase deficiency include

, flatulence, cramping abdominal pain, and diarrhea, which usually occur within a half hour to several hours after drinking a glass of milk or ingesting a milk product.

Which assessment data support to the the nurse the client's diagnosis of gastric ulcer? A. Presence of blood in the client's stool for the past month? B. Reports of a burning sensation moving like a wave. C. Sharp pain in the upper abdomen after eating a heavy meal. D. Complaints of epigastric pain 30-60 minutes after ingesting food

- Answer: D In a client diagnosed with a gastric ulcer, pain usually occurs 30-60 minutes after eating, but not at night. In contrast, a client with a duodenal ulcer has pain during the night often relieved by eating foods. Other answers: the presence of blood does not specifically indicate diagnose of an ulcer. The client could have hemorrhoids or cancer. A waveline burning sensation is a symptom of GERD. Sharp pain in the upper abdomen after eating a heavy meal is a symptom of gallbladder disease

Why do acid-peptic disorders occur?

--consequence of an increase in acid exposure, --a decrease in mucosal defense or some combo of the two.

cholelithiasis

-10-20% of adult pop >80% are silent- free of biliary pain or complications

What is the epidemiology of GERD?

-36% have monthly heartburn -15% use antacids -7% have daily heartvburn -Most common cause of esophageal CA -2nd most common cause of ASTHMA -2nd most common cause of laryngitis -Most common cause of recurrent non-cardiac chest pain ***frequent cause of failure to thrive in infants, 2nd most common cause of asthma in adults, most common cause of adenocarcinoma

Therapeutic alternatives to NSAIDs

-Acetomenophen (Tylenol) for non-inflammatory pain -COX-2 selective NSAID like celecoxib (celebrex) -Use prostaglandin like med to restore mucosal barrier like misoprostil (Cytotec) ***don't really want to give prostaglandins since cause cramping, abortions (part of morning after pill) -Decrease acid secretion w H2 receptor antagonist(cimetadine, something -adine) or a proton pump inhibitor

What are the two types of highly regulated acid secretion?

-Basal -Meal stimulated

What are the complications of ulcer disease?

-Bleeding manifested by hematemesis or melena -stricture manifested by early satiety and post prandial nausea and vomiting -weight loss **H.Pylori is most common cause of duodenal ulcer disease and NSAIDs most common with gastric ulcer disease

What are peptic ulcer diseases or PUD characterized by?

-Epigastric burning (dyspepsia) which typically occurs on an empty stomach and is relieved by eating. Pain only with pH <3

What is the Clinical presentation of ulcer disease?

-Epigastric burning pain relieved by eating/antacids -Frequently awakens pt between Midnight and 2

What are the alarm sx of dyspepsia peptic ulcer disease?

-Hematemesis (indicating GI bleeding) -Melena (indicating GI bleeding) -Back pain (suggesting ulcer penetration into the pancreas) -Early satiety (suggesting gastric outlet obstruction) -Weight loss (suggesting malignancy)

What are the common causes of acid getting into esophagus?

-Hiatal hernia -dysfunction of LES

How is GERD treated?

-PRN H2 blocker or proton pump inhibitor -Pts with multiple risk factors neeed UGI endoscopy to screen for Barrett's. (risk factors are over 50, obese white male, chronic reflux, hiatus hernia0

What are the major fxns of the stomach?

-Secretion (acid, pepsinogen, intrinsic factor, mucus, bicarbonate) -Motility for the processing of food

What stimulates acid secretion?

-a rise in gastric pH (eating).

defective transport symptoms

-abdominal pain -elevated levels of plasma amylase and lipase -referred pain to upper back -leukocytosis, hemolysis, acute respiratory distress syndrome, diffuse fat necrosis

Hepatitis B

-acute with resolution or chronic -chronic can lead to cirrhosis or fulminant (w massive liver necrosis) or lead to Hep D -worldwide carrier rate 350million and infected 2 billion -75%carriers in Asia

benign neoplasm

-adenomas -tend to occur in young women who used oral contraceptives -pale yellow, bile stained nodules -up to 30cm in diameter -will have vascular components but no portal tracts

What are the pulmonary manifestations of GERD?

-asthma -chronic bronchitis -pulmonary fibrosis -sleep apnea

What are the complications of GERD?

-benign strictures (solid food dysphagia) -bleeding (hematemesis or melena) -long-term = adenocarcinoma (lifetime risk 1-2%)

Hepatitis A

-benign, self-limiting -incubation 2-6wks -No chronic or carrier -rarely fatal <0.1%

GB pathology

-cholelithiasis(gallstones) -cholesterol stones -pigment stones

What are the ENT manifestations of GERD?

-chronic cough -chronic hoarseness -dental erosions -Globus (lump in throat) -laryngeal CA -vocal cord ulcer/granuloma

intestinal phase

-copious pancreatic secretion -dumps into duodenum -mainly due to secretin-in response to acidic chyme entering duodenum

Hepatitis D

-delta virus RNA- defective replication -must be encapsulated by HBV -seen mostl in africa, middle east and southern italy

duct obstruction

-either common bile or pancreatic duct -increased pressure -accumulation of enzyme rich fluid -lipase is secreated in active form -local fat necrosis

How does Non-ulcer dyspepsia present?

-epigastric pain, fullness, plus/minus bloating and early satiety

hepatocellular carcinoma features

-feel nodular on palpation of liver -upper ab pain, malaise, fatigue, wt loss, and sense of fullness -elevated alpha-fetoprotein-false+ Rad studies, CT or MRI best for detection of small tumor for resection

alcoholic cirrhosis

-final and IRREVERSIBLE -brown and shrunken -fibrosis and hyperplastic nodules

oxidative

-free radicals -oxidize lipid membranes -attract mononuclear cells -lead to cell necrosis, inflammation and fibrosis

chronic alcohol consumption liver diseases

-hepatic steatosis -alcoholic hepatitis -cirrhosis

alcoholic hepatitis charateristics

-heptocyte swelling and necrosis -mallory bodies-heptocytes accumulate tangled skeins of cytokeratin filaments -neutrophilic reaction(permeate lobule) -fibrosis-act stellate cells

What are the factors that promote reflux?

-hiatal hernia -meds that reduce LES pressure -Pregnancy -Lrg meals that distend stomach -recumbancy after meals -smoking -food that decreases LES pressure(fat, caffeine, chocolate, peppermint, alcohol)

secretin in intestinal phase

-huge amts of bicarbonate ion and water secretion

HBV

-incubation 4-26wks -present in all fluids except feces -mature virion has outer surface envelope -dsDNA circular

diff b/t acute and chronic

-irreversible impairment

malignant tumor

-liver and lungs-most often involved in metastatic spread

Hepatitis C

-major cause of liver disease(50% of chronic liver disease) -most common chronic blood borne infection -85% of infections lead to chronic liver disease

cholangiocarcinoma

-malignancy of biliary tree, bile duct w/in or outside the liver -low incidence in No Amer -tree-like tumor, not bile stained -usually detected late-as a result of bile obstruction -poor outlook (6month survival)

pancreatic juice composition

-mostly water -enzymes and bicarbonate

Describe the esophagus

-muscular tube -sphincter at each end (UES, LES) -Major fxns to transport food from pharynx to stomach and prevent reflux into esophagus and airway

What is Basal acid secretion?

-occurs independently from meals and maintains a continuous low level of acid secretion. -Provides a barrier to organisms and toxic substances in our Environment -Mediated by the VAGUS nerve and has diurnal pattern that is high at night, low during day. **NOTE - this accounts for pattern of dyspepsia and heartburn between midnight and 2 am-when pH drops below 3 which is necessary to perceive pain

secretin

-released when HCl detected in duodenum -stimulates pancreas

chronic pancreatitis presentations

-repeated attacks- moderate ab pain and/or back pain -may be silent until diabetes mellitus from destruction of islets of L -recurrent jaundice or indigestion attacks -gallstone destruction may be seen also

necrosis-fibrosis

-repeated episodes of acute leads to cell necrosis and fibrosis

cephalic and gastric phases

-same brain signals for stomach secretion- stim pan sec -acini secrete but not enough water to cause flow to duodenum -20% of total secretion of enzymes -gastric phase- same- accounts for another 5-10% of enzymes

What are the 2 types of dysphagia (difficulty swallowing)

-solid food dys - MECHANICAL disorder like stricture or neoplasm -solid and liquid - MOTILITY disorder

What are the alarm sx for esophageal CA?

-solid food dysphagia (sticking!) -Weight loss -Hematemesis/melena -progressive retrosternal aching over the lower chest

What is Meal-stimulated acid secretion?

-superimposed on basal acid secretion and occurs in response to ingestion of a meal. -Result of highly coordinated endocrine (gastrin), neurocrine (acetylcholine) and pancreatic (histamine) activity.

4 steps for crystal formation

-supersaturation of bile w cholesterol -GB hypomotility (promotes nucleation) -cholesterol nucleation accelerated -hypersecretion of mucus in GB traps crystals -which then aggregate into stones

toxic-metabolic

-toxins that directly affect acinar cells- can cause their destruction -leads to build up of lipids in acinars -> cell death and necrosis

The nurse is administering morning medications at 0730. Which medication should have priority? A. a proton pump inhibitor B. A nonnarcotic analgesicC. A histamine receptor antagonist D. A mucosal barrier agent

...CORRECT ANSWER: D.A. PPI's can be administered routinely. B. Pain medication is important but can be adminstered after a medication that is timed. C. A histamine receptor antagonist can be administered at routine dosing time. D. A mucosal barrier agent must be adminstered on an empty stomach for the medication to coat the stomach lining

• The two major aspects of nursing care for the patient undergoing ostomy surgery are

1 Emotional support. 2. Caring for the ostomy

What size must particles be to cross the pylorus into the duodenum?

1 mm or less. Liquids empty in minutes, solids 2-3 hrs.

glucagon potent hyprglycemic agent

1 molecule of glucagon can release 100milion molecules of glucose into blood -major target is liver

84-year-old man is being discharged from the hospital. He was admitted with complications from esophageal cancer. PEG tube has been placed to allow tube feedings. He lives with his wife, who is 82 years old. Has no other family in town 1.What risks are associated with a PEG that you should discuss with him and his wife? 2.What skills must he and his wife learn before the time of discharge? 3.How should his response to treatment be managed?

1. Aspiration, ensuring proper use of formula 2. Proper tube care 3. His weight and hydration status

A young woman calls a triage hotline complaining of severe nausea for the past 6 hours. She denies any emesis, but is unable to obtain any relief from her nausea. She states her temperature is "a little higher than normal" at 99.5o F. She has had no recent illnesses. 1.What other information should you obtain from her? 2.What remedies could she use to relieve her nausea? 3.If she started vomiting, would you change your advice?

1. Dietary history (and if anyone else has nausea), history of nausea, and discussion of possible pregnancy 2. Breathing exercises and peppermint tea 3. No, unless her vomiting is unremitting or includes blood.

27-year-old woman comes into the clinic with fatigue. She has lost 10 pounds in the past 2 months. She is a graduate student working on her dissertation. History of Crohn's disease Claims she is unable to watch her diet because she "doesn't have time to think about that" In addition to fatigue, she reports having diarrhea and no appetite. 1.What nutrients is she likely deficient in? 2.What dietary recommendations can be made? 3.How can you help her monitor her diet without added stress?

1. Fluids, electrolytes, fat 2. She should begin a high-fat, high-protein diet. 3. Provide her with a list of foods that she should try to eat.

What factors INCREASE resting LES pressure?

1. GI hormones - gastrin, bombesin, motilin, substance P 2. Foods - protein meal 3. Drugs - bethanechol, dopamine inhibitors, meoclopramide, alpha-adrenergic agonists (epinephrine, clonidine), B-adrenergic antagonists (propranalol 4. Increased gastric pH - eating, antacids 5. Miscellaneous - migrating motor complex, prostaglandin F2

What factors DECREASE resting LES pressure?

1. GI hormones - secretin, CCK, somatostatin, glucagon, progesterone 2. Foods - fatty meal, chocolate, caffeine, ethanol, peppermint 3. Drugs - anticholinergics, B-adrenergic agonists, barbiturates, calcium channel blockers, theophylline, diazepam, morphine, alpha-adrenergic antagonists, 4. Decreased gastric pH-fasting 5. Miscellaneous - Nicotine, prostaglandins, serotonin, nitric oxide

How is the dx of ulcer disease made?

1. H. pylori serology (shows exposure w/ +IGg, but would have to do another test to know if living present, however, can tx w positive blood test) 2. Gastric biopsy with CLO test for active infection (best but expensive) 3. Stool antigen test for active infection (cheap and sensitive) 4. Breath test for active infection (good but EXPENSIVE)

76-year-old man complains of feeling like food is "stuck" in his esophagus. Also complains of experiencing epigastric pain while eating History of smoking 1 pack per day for 40 years and hypertension EGD and biopsy samples taken reveal esophageal cancer. Scheduled for surgical removal of affected area 1.What alterations in function is he likely to have after surgery? 2.What complications might he experience with food intake? 3.What patient teaching should you perform?

1. He will have an NG tube immediately after surgery, and then he will need to maintain an upright position for at least 2 hours after eating. 2. He may need a permanent feeding gastrostomy. 3. Any changes in eating habits should be discussed, along with proper care of wound/gastrostomy teaching.

How do you treat acid peptic disease?

1. Heal ulcer by restoring balance by inhibiting acid or increasing mucosal protection 2. Prevent recurrence by treating underlying cause (h pyl or NSAID)

21-year-old woman comes into her physician's office for her routine check-up. She states that she has switched to a vegetarian diet because she is concerned with cruelty to animals. She eats mainly fast food and salads from her college cafeteria. She consumes milk, eggs, and cheese. No abnormalities are noted on assessment. 1.What deficiency might her diet include? 2.What foods would you stress to compensate for the lack of meat? 3.What resources could you recommend to help her learn more about a vegetarian's needs?

1. Iron deficiency 2. Beans, legumes, soy products 3. The USDA website

40-year-old woman was previously admitted to the hospital for exacerbation of Cr0hn's disease. After beginning high-dose IV corticosteroid treatment for Crohn's disease, she develops massive upper GI bleeding. Bright red blood hematemesis and melena She is treated with iced saline NG irrigations and blood transfusions. The bleeding is stopped. The following day, she has a recurrence of bleeding through her nasogastric tube and bright blood from her rectum. Gastroscopy reveals a peptic ulcer. After a third day of more emesis of blood clots, she is taken to surgery, where a Billroth II is performed. 1.What is the likely cause of the massive gastrointestinal bleeding? 2.Following the surgery, what is the priority nursing intervention for her? 3.What nutritional changes should she make because of the diagnosis of peptic ulcer? 4.What patient teaching should you do with her?

1. Peptic ulcer 2. Closely monitor I & O and vital signs. 3. Avoid acidic foods, caffeine, and alcohol. 4. Nutritional changes, need for taking only prescription medications, how to avoid future incidences, avoiding smoking, importance of monitoring for bleeding, etc.

20-year-old man complains of loss of appetite and occasional lower sternal chest pain 30 to 60 minutes after meals. He claims symptoms began about 6 months ago. He has a history of asthma. States he has needed inhaler "more than usual" Does not know what makes it worse or better EGD suggests GERD. 1.How may factors be affecting his GERD? 2.What nutritional counseling should you do? 3.He states that he doesn't want to take any medication. How can you best advise him? 4.What are long-term complications of GERD?

1. Respiratory history 2. Avoid acids foods, alcohol, and caffeine. 3. Discuss with him that if untreated, GERD can lead to esophagitis and Barrett's esophagus. 4. Esophagitis, Barrett's esophagus, respiratory problems, and dental erosion

75-year-old woman is scheduled to have exploratory surgery tomorrow for a bowel obstruction. History of type 2 diabetes Orders call for a PICC insertion because the physician anticipates a need for resting her GI tract. Her son is here for the surgery, and he is concerned about when she can eat "regular food" after the surgery. She lives in an assisted living facility, and her son visits a few times a week. 1.What can you tell the son about feeding after her surgery? 2.What are you concerned about because she is a diabetic? 3.What will you teach her and her son to monitor? 4.What resources are available to help her at home?

1. She will be fed through the PICC only until she is able to eat 60% of her calories orally. Then she will be back to eating "normally." 2. Her blood glucose should be monitored closely. A slight increase in blood glucose is to be expected at the outset of PN. 3. If she continues PN at home, she should be taught proper tube care and should learn the signs of infection. 4. 4. OLEY foundation (see resources at end of chapter for a complete list)

45-year-old woman is hospitalized for shortness of breath and respiratory distress. Physical examination findings Blood pressure 150/72 Heart rate 104 Respiratory rate 30 Temperature 98.3° F SaO2 88% Height 5'5" Weight 320 pounds History of hypertension, type 2 diabetes, COPD, obesity She states that she's "tired of being like this." 1.What opportunities for education and support do you have? 2.What other problems is she likely to have related to her weight? 3.What treatment options are available for her? 4.What tools may help her with behavior modification? 5.If she wants to have bariatric surgery, what risks does the surgery pose?

1. Teach her the negative effects of obesity. 2. Pain in joints, high cholesterol, sleep apnea 3. Drug therapy for weight reduction, such as Alli, or bariatric surgery 4. Support groups or individual therapy sessions 5. Postanesthesia problems due to her weight and pain following surgery

What are the two mechanisms by which NSAIDs cause ulcers?

1. superficial injuring mediated directly by NSAIDs which are weak bases (so, they are lipophilic in the acid environment and are passively absorbed by epithelial lining cells. Once inside the cells they become hydrophilic and can't get out becoming bioconcentrated (TOXIC) and killing the cells. 2. Inhibition of prostaglandin mediated mucosal protection

western countries pancreatitis

10-20/100,00 80% assoc w biliary tract disease or alcoholism -gallstones present in 35-60% of cases (but only 5% of pop w gallstones dev-pancreatitis)

pancreatic juice amount

1200-1500ml/day

trypsin activates

2 other pancreatic enzymes -procarboxypeptidase -chymotrypsinogen

"The nurse is teaching the patient a client with a peptic ulcer discharge instructions. The client asks the nurse which type of analgesic he may take. Which of the following responses by the nurse would be most accurate? "1. Aspirin 2. Acetaminophen 3. Naproxen 4. Ibuprofen

2.Acetaminophen is recommended for pain relief because it does no promote irritation of the mucosa. Aspirin, and nonsteroidal anti- inflammatory drugs suchs as naproxen and ibuprofen, may cause irritation of the mucosa and subsequent bleeding

The nurse is planning to teach the client with GERD about substance that will increase the lower esophageal sphincter pressure. Which item should the nurse include on this list? Saunders NCLEX Examination Review 1. Coffee 2. Chocolate 3. Fatty Foods 4.Nonfat milk

4. Nonfat milk Foods that increase LES pressure will decrease reflux and decrease symptoms. Milk will increase LES pressure

The nurse is monitoring the client with a diagnosis of peptic ulcer. Which assessment finding would most likely indicate perforation of the ulcer? 1. Bradycardia 2. Numbness in Legs 3. Nausea and Vomiting 4. A rigid, board-like abdomen

4. Perforation of an ulcer is a surgical emergency and is a sudden, sharp, intolerable severe pain beginning in the midepigastric area and spreading over the abdomen, which becomes rigid and board-like. Nausea and vomiting may occur. Tachycardia may occur as hypovolemic shock develops. Numbness in the leg is not an associated finding

pancreatic carcinoma

4th leading cause of cancer death -one of highest cancer mortality rates less than 5% 5yr survival

cystic neoplasms

5% of neoplams of pancreas -5-15% of cysts are neoplastic

location

60% in head 15% body 5% tail 20% diffuse

alcohol abuse

67% of pop drinks alcohol 14million meet criteria for abuse 200,000 deaths/ye (5th most) 25-30% of hospitalizations have problems related to abuse

What ulcers does H pylori cause

70% of duodenal ulcers and also 30% gastric ulcers *in most peopl it's a commensal organism in harmony w its environment (human stomach) and causes disease in minority of susceptible people. -Causes 90% of STOMACH CA **PUD like all acquired diseases are an interaction between external factors and host susceptibility.

CCK in intestinal phase

70-80% of total enzyme release from acini cells

age

80% b/t 60-80 yoa

2 types of cholelithiasis

80% cholesterol stones the rest bilirubin calcium salts

hepatocellular carcinoma

>85% of cases occur in countries w high rate of HBV (china/SoAfrica) -may appear as unifocal, multifocal, or diffusely infiltrative -cancer usually paler than surrounding liver -strong propensity for vascualr channels

A patient is jaundiced and her stools are clay colored (gray). This is most likely related to: a. decreased bile flow into the intestine b. increased production of urobilinogen c. increased production of cholecystokinin d. increased bile and bilirubin in the blood

A

A patient with a history of peptic ulcer disease has presented to the emergency department with complaints of severe abdominal pain and a rigid, boardlike abdomen, prompting the health care team to suspect a perforated ulcer. Which of the following actions should the nurse anticipate? A) a. Providing IV fluids and inserting a nasogastric tube B) b. Administering oral bicarbonate and testing the patient's gastric pH level C) c. Performing a fecal occult blood test and administering IV calcium gluconate D) d. Starting parenteral nutrition and placing the patient in a high-Fowler's position

A

A patient with hepatitis A is in the acute phase. The nurse plans care for the patient based on the knowledge that: a. pruritis is a common problem with jaundice in this phase b. the patient is most likely to transmit the disease during this phase c. gastrointestinal symptoms are not as severe in hepatitis A as they are in hepatitis B d.extrahepatic manifestations of glomerulonephritis and polyarteritis are common in this phase

A

A patient with type 2 diabetes and cirrhosis asks the nurse if it would be okay to take silymarin (milk thistle) to help minimize liver damage. The nurse responds based on knowledge that A) a. Milk thistle may affect liver enzymes and thus alter drug metabolism. B) b. Milk thistle is generally safe in recommended doses for up to 10 years. C) c. There is unclear scientific evidence for the use of milk thistle in treating cirrhosis. D) d. Milk thistle may elevate the serum glucose levels and is thus contraindicated in diabetes.

A

After administering a dose of promethazine (Phenergan) to a patient with nausea and vomiting, the nurse explains that which of the following may be experienced as a common temporary adverse effect of the medication? A) a. Drowsiness B) b. Reduced hearing C) c. Sensation of falling D) d. Photosensitivity

A

During assessment of a patient with obstructive jaundice, the nurse would expect to find: a. clay-colored stools b. dark urine and stools c. pyrexia and severe pruritis d. elevated urinary urobilinogen

A

During the initial postoperative period following bariatric surgery, the nurse recognizes the importance of monitoring obese patients for respiratory insufficiency based on knowledge that A) a. The body stores anesthetics in adipose tissue. B) b. Intubation may be difficult because of extra chin skin folds. C) c. Postoperative pain may cause a decreased respiratory rate. D) d. The patient's head must remain flat for a minimum of 2 hours postprocedure.

A

In planning care for a patient with metastatic liver cancer, the nurse should include interventions that: a. focus primarily on symptomatic and comfort measures b. reassure the patient that chemotherapy offers a good prognosis .c promote the patient's confidence that surgical excision of the tumor will be successful d. provide information necessary for the patient to make decisions regarding liver transplantation

A

The appropriate collaborative therapy for the patient with acute diarrhea caused by a viral infection is to: a. increase fluid intake b. administer an antibiotic c. administer antimotility drugs d. quarantine the patient to prevent spread of the virus

A

The nurse confirms initial placement of a blindly inserted small bore NG feeding tube by: a. x-ray b. air insufflation c. observing patient for coughing d. pH measurement of gastric aspirate

A

The nurse determines that the goals of dietary teaching have been met when the patient with celiac disease selects from the menu: a. scrambled eggs and sausage b. buckwheat pancakes with syrup c. oatmeal, skim milk, and orange juice d. yogurt, strawberries, and rye toast with butter

A

The nurse explains to the patient undergoing ostomy surgery that the procedure that maintains the most normal functioning of the bowel is: a. sigmoid colostomy b. a transverse colostomy c. a descending colostomy d. an ascending colostomy

A

The nurse is caring for a patient who is 5'6" tall and weighs 186 lb. The nurse has discussed reasonable weight loss goals and a low-calorie diet with the patient. Which of these statements made by the patient indicates a need for further teaching? A) a. "I will limit intake to 500 calories a day." B) b. "I will try to eat very slowly during mealtimes." C) c. "I'll try to pick foods from all of the basic food groups." D) d. "It's important for me to begin a regular exercise program."

A

The nurse is conducting discharge teaching for a patient with metastatic lung cancer who was admitted with a bowel impaction. Which of the following instructions would be most helpful to prevent further episodes of constipation? A) a. Maintain a high intake of fluid and fiber in the diet. B) b. Reduce intake of medications causing constipation. C) c. Eat several small meals per day to maintain bowel motility. D) d. Sit upright during meals to increase bowel motility by gravity.

A

The nurse is preparing to insert a nasogastric tube into a 68-year-old patient with an abdominal mass and suspected bowel obstruction. The patient asks the nurse why this procedure is necessary. Which of the following responses is most appropriate? A) a. "The tube will help to drain the stomach contents and prevent further vomiting." B) b. "The tube will push past the area that is blocked, and thus help to stop the vomiting." C) c. "The tube is just a standard procedure before many types of surgery to the abdomen." D) d. "The tube will let us measure your stomach contents, so that we can plan what type of IV fluid replacement would be best."

A

The percentage of daily calories for a healthy individual consists of: a. 50% carbohydrates, 25% protein, 25% fat, and <10% of fat from saturated fatty acids b. 65% carbohydrates, 25% protein, 25% fat, and >10% of fat from saturated fatty acids c. 50% carbohydrates, 40% protein, 10% fat, and <10% of fat from saturated fatty acids d. 40% carbohydrates, 30% protein, 30% fat, and >10% of fat from saturated fatty acids

A

Two days following a colectomy for an abdominal mass, a patient reports gas pains and abdominal distention. The nurse plans care for the patient based on the knowledge that the symptoms occur as a result of which of the following? A) a. Impaired peristalsis B) b. Irritation of the bowel C) c. Nasogastric suctioning D) d. Anastomosis site inflammation

A

When administered a dose of metoclopramide (Reglan), a patient complains of nausea. The nurse would teach the patient to report which of the following potential adverse effects? A) a. Tremors B) b. Constipation C) c. Double vision D) d. Numbness in the fingers and dose

A

When assessing a patient's abdomen, it would be most appropriate to A) a. Auscultate the abdomen before palpation. B) b. Palpate the abdomen before auscultation. C) c. Percuss the abdomen before auscultation. D) d. Perform deep palpation before light palpation.

A

When preparing a patient for a capsule endoscopy study, the nurse should A) a. Ensure the patient understands the required bowel preparation. B) b. Have the patient return to the procedure room for removal of the capsule. C) c. Teach the patient to maintain a clear liquid diet throughout the procedure. D) d. Explain to the patient that conscious sedation will be used during placement of the capsule.

A

Which of the following methods should be used to confirm the placement of a patient's newly-inserted nasogastric (NG) feeding tube? A) a. X-ray B) b. Aspiration C) c. Air auscultation D) d. Measurement of external length

A

The nurse is caring for a patient in the emergency department with complaints of acute abdominal pain, nausea, and vomiting. When the nurse palpates the patient's left lower abdominal quadrant, the patient complains of pain in the right lower quadrant. The nurse will document this as which of the following diagnostic signs of appendicitis? A) a. Rovsing sign B) b. Referred pain C) c. Chvostek's sign D) d. Rebound tenderness

A In patients with suspected appendicitis, Rovsing sign may be elicited by palpation of the left lower quadrant, causing pain to be felt in the right lower quadrant.

16. The nurse provides discharge instructions to a patient following gastric bypass surgery for treatment of obesity. The nurse teaches the patient to a. avoid foods high in carbohydrates and not to drink fluids with meals. b. maintain a liquid diet for about 6 weeks until gastric healing is complete. c. eat a high carbohydrate, high-roughage diet to promote bowel function when solid foods are allowed. d. exercise and massage to prevent the development of flabby skin resulting from massive weight loss.

A Rationale: Fluids and high carbohydrate foods tend to cause dumping syndrome and diarrhea. The patient transitions to pureed foods at about 2 to 4 weeks and by 4 to 6 weeks is eating solid foods. The diet after bariatric surgery should be high in protein and low in carbohydrates and roughage. Exercise and massage will not prevent the development of flabby skin. Cognitive Level: Application Text Reference: p. 986 Nursing Process: Implementation NCLEX: Physiological Integrity

8. A patient has a new prescription for sibutramine (Meridia). Which information about the patient indicates that the nurse should discuss the medication with the health care provider before administration? a. The patient has a history of hypertension. b. The patient has a permanent pacemaker to prevent bradycardia. c. The patient's goal is to lose 90 lb (41 kg). d. The patient used fenfluramine (Pondimin) in the past for weight loss.

A Rationale: Side effects of sibutramine (Meridia) include hypertension. A permanent pacemaker and a history of fenfluramine use are not contraindications for sibutramine. Sibutramine is prescribed for patients who have large weight-loss goals. Cognitive Level: Application Text Reference: p. 981 Nursing Process: Assessment NCLEX: Physiological Integrity

1. The nurse is assessing patients at the outpatient clinic. Which of these patients is at risk for health complications related to weight? a. A 24-year-old with a waist measurement of 30 in (75 cm) and a hip measurement of 34 in (85 cm) b. A 33-year-old who has a body mass index (BMI) of 24 kg/m2 c. A 56-year-old who is 6 ft (180 cm) tall and weighs 150 lb (68 kg) d. A 71-year-old who is 5 ft 4 in (160 cm), weighs 120 lb (55 kg), and carries most of the weight in the thighs

A Rationale: The waist-to-hip ratio for this patient is 0.88, which exceeds the recommended level of <0.80. A BMI of 24 kg/m2 is normal. A patient 6 ft tall and weighing 150 lb would have a BMI of 20 kg/m2, well within the normal range. A 5 ft 4 in patient weighing 120 lb would have a BMI of 21 kg/m2; additionally, carrying most of the weight in the hips and thighs would place the patient at lower risk. Cognitive Level: Application Text Reference: p. 974 Nursing Process: Assessment NCLEX: Health Promotion and Maintenance

5. Which of these menu selections by a patient who is attempting to lose weight indicates that the initial instructions about diet have been understood? a. 3 oz of pork roast, a cup of corn, and a sliced tomato b. A chicken breast and a cup of tossed salad with nonfat dressing c. A 6 oz can of tuna mixed with nonfat mayonnaise and chopped celery d. 3 oz of roast beef, 2 oz of low-fat cheese, and a half-cup of carrot sticks

A Rationale: This selection is most consistent with the recommendation of the American Institute for Cancer Research that one third of the diet should be from animal sources and two thirds from plant source foods. The other choices all have higher ratios of animal origin foods to plant source foods than would be recommended. Cognitive Level: Application Text Reference: p. 979 Nursing Process: Evaluation NCLEX: Health Promotion and Maintenance

Which of the following statements are accurate as they relate to medications used to manage GERD? "A)Magnesium-containing antacids can cause diarrhea. B) Aluminum-containing antacids can cause constipation. C) Cimetidine (Tagamet HB) causes osteomalacia and hypophosphatemia. D)Misoprostol's (Cytotec) major side effect is G.I. bleeding

A&B. Rationale: Magnesium-containing antacids can cause diarrhea, and should be used with caution in older persons with renal dysfunction. Aluminum-containing antacids can cause constipation, osteomalacia, and hypophosphatemia. Cimetidine has the greatest chance for adverse reactions, including erectile dysfunction, Gynecomastia, and confusion. Misoprostol's major side effects are diarrhea and abdominal pain

6. The nurse is performing a health interview on a patient who has a language barrier, and no interpreter is available. Which is the best example of an appropriate question for the nurse to ask in this situation? A) "Do you take medicine?" B) "Do you sterilize the bottles?" C) "Do you have nausea and vomiting?" D) "You have been taking your medicine, haven't you?"

A) "Do you take medicine?" Page: 46 In a situation where there is a language barrier and no interpreter available, use simple words avoiding medical jargon. Avoid using contractions and pronouns. Use nouns repeatedly and discuss one topic at a time.

30. During a mental status examination, the nurse wants to assess a patient's affect. The nurse should ask the patient which question? A) "How do you feel today?" B) "Would you please repeat the following words?" C) "Have these medications had any effect on your pain?" D) "Has this pain affected your ability to get dressed by yourself?"

A) "How do you feel today?" Page: 74. Judge mood and affect by body language and facial expression and by asking directly, "How do you feel today?" or "How do you usually feel?" The mood should be appropriate to the person's place and condition and should change appropriately with topics.

73. The nurse is performing an ear examination of an 80-year-old patient. Which of these would be considered a normal finding? A) A high-tone frequency loss B) Increased elasticity of the pinna C) A thin, translucent membrane D) A shiny, pink tympanic membrane

A) A high-tone frequency loss Pages: 337-338. A high-tone frequency hearing loss is apparent for those affected with presbycusis, the hearing loss that occurs with aging. The pinna loses elasticity, causing earlobes to be pendulous. The eardrum may be whiter in color and more opaque and duller than in the young adult.

7. A female patient does not speak English well, and the nurse needs to choose an interpreter. Which of the following would be the most appropriate choice? A) A trained interpreter B) A male family member C) A female family member D) A volunteer college student from the foreign language studies department

A) A trained interpreter Page: 46 whenever possible, the nurse should use a trained interpreter, preferably one who knows medical terminology. In general, an older, more mature interpreter is preferred to a younger, less experienced one, and the same gender is preferred when possible.

10. During an assessment, the nurse notices that a patient is handling a small charm that is tied to a leather strip around his neck. Which action by the nurse is appropriate? A) Ask the patient about the item and its significance. B) Ask the patient to lock the item with other valuables in the hospital's safe. C) Tell the patient that a family member should take valuables home. D) No action is necessary.

A) Ask the patient about the item and its significance. Page: 21 The nurse should inquire about the amulet's meaning. Amulets, such as charms, are often seen as an important means of protection from "evil spirits" by some cultures.

41. A 6-month-old infant has been brought to the well-child clinic for a check-up. She is currently sleeping. What should the nurse do first when beginning the examination? A) Auscultate the lungs and heart while the infant is still sleeping. B) Examine the infant's hips because this procedure is uncomfortable. C) Begin with the assessment of the eye and continue with the remainder of the examination in a head-to-toe approach. D) Wake the infant before beginning any portion of the examination to obtain the most accurate assessment of body systems.

A) Auscultate the lungs and heart while the infant is still sleeping. Pages: 122-124. When the infant is quiet or sleeping is an ideal time to assess the cardiac, respiratory, and abdominal systems. Assessment of the eye, ear, nose, and throat are invasive procedures and should be performed at the end of the examination.

79. When assessing tactile fremitus, the nurse recalls that it is normal to feel tactile fremitus most intensely over which location? A) Between the scapulae B) Third intercostal space, MCL C) Fifth intercostal space, MAL D) Over the lower lobes, posterior side

A) Between the scapulae Page: 424. Normally, fremitus is most prominent between the scapulae and around the sternum. These are sites where the major bronchi are closest to the chest wall. Fremitus normally decreases as one progress down the chest because more tissue impedes sound transmission.

124. The nurse knows that determining whether a person is oriented to his or her surroundings will test the functioning of which of these structures? A) Cerebrum B) Cerebellum C) Cranial nerves D) Medulla oblongata

A) Cerebrum Pages: 621-622 | Page: 660. The cerebral cortex is responsible for thought, memory, reasoning, sensation, and voluntary movement. The other options structures are not responsible for a person's level of consciousness.

46. During a nutritional assessment, why is it important for the nurse to ask a patient what medications he or she is taking? A) Certain drugs can affect the metabolism of nutrients. B) The nurse needs to assess the patient for allergic reactions. C) Medications need to be documented on the record for the physician's review. D) Medications can affect one's memory and ability to identify food eaten in the last 24 hours.

A) Certain drugs can affect the metabolism of nutrients. Page: 183 Analgesics, antacids, anticonvulsants, antibiotics, diuretics, laxatives, antineoplastic drugs, steroids, and oral contraceptives are drugs that can interact with nutrients, impairing their digestion, absorption, metabolism, or use. The other responses are not correct.

9. The nurse is conducting an interview in an outpatient clinic and is using a computer to record data. Which is the best use of the computer in this situation? Select all that apply. A) Collect the patient's data in a direct, face-to-face manner. B) Enter all the data as the patient states it. C) Ask the patient to wait as the nurse enters data. D) Type the data into the computer after the narrative is fully explored. E) Allow the patient to see the monitor during typing.

A) Collect the patient's data in a direct, face-to-face manner. D) Type the data into the computer after the narrative is fully explored. E) Allow the patient to see the monitor during typing. Page: 32 The use of a computer can become a barrier. The nurse should begin the interview as usual by greeting the patient, establishing rapport, and collecting the patient's narrative story in a direct face-to-face manner. Only after the narrative is fully explored should the nurse type data into the computer. When typing, the nurse should position the monitor so that the patient can see it.

52. A patient tells the nurse that he has noticed that one of his moles has started to burn and bleed. When assessing his skin, the nurse would pay special attention to the danger signs for pigmented lesions and would be concerned with which additional finding? A) Color variation B) Border regularity C) Symmetry of lesions D) Diameter less than 6 mm

A) Color variation Pages: 212-213. Abnormal characteristics of pigmented lesions are summarized in the mnemonic ABCD: asymmetry of pigmented lesion, border irregularity, color variation, and diameter greater than 6 mm.

5. The nurse is interviewing a patient who has a hearing impairment. What techniques would be most beneficial in communicating with this patient? A) Determine the communication method he prefers. B) Avoid using facial and hand gestures because most hearing-impaired people find this degrading. C) Request a sign language interpreter before meeting with him to help facilitate the communication. D) Speak loudly and with exaggerated facial movement when talking with him because this helps with lip reading.

A) Determine the communication method he prefers. Pages: 40-41 The nurse should ask the deaf person the preferred way to communicate—by signing, lip reading, or writing. If the person prefers lip reading, then the nurse should be sure to face him or her squarely and have good lighting on the nurse's face. The nurse should not exaggerate lip movements because this distorts words. Similarly, shouting distorts the reception of a hearing aid the person may wear. The nurse should speak slowly and should supplement his or her voice with appropriate hand gestures or pantomime.

101. The nurse is percussing the seventh right intercostal space at the midclavicular line over the liver. Which sound should the nurse expect to hear? A) Dullness B) Tympany C) Resonance D) Hyperresonance

A) Dullness Page: 541. The liver is located in the right upper quadrant and would elicit a dull percussion note.

87. During auscultation of breath sounds, the nurse should use the stethoscope correctly, in which of the following ways? A) Listen to at least one full respiration in each location. B) Listen as the patient inhales and then go to the next site during exhalation. C) Have the patient breathe in and out rapidly while the nurse listens to the breath sounds. D) If the patient is modest, listen to sounds over his or her clothing or hospital gown.

A) Listen to at least one full respiration in each location. Pages: 426-427. During auscultation of breath sounds with a stethoscope, it is important to listen to one full respiration in each location. During the examination, the nurse should monitor the breathing and offer times for the person to breathe normally to prevent possible dizziness.

1. In an interview, the nurse may find it necessary to take notes to aid his or her memory later. Which statement is true regarding note-taking? A) Note-taking may impede the nurse's observation of the patient's nonverbal behaviors. B) Note-taking allows the patient to continue at his or her own pace as the nurse records what is said. C) Note-taking allows the nurse to shift attention away from the patient, resulting in an increased comfort level. D) Note-taking allows the nurse to break eye contact with the patient, which may increase his or her level of comfort.

A) Note-taking may impede the nurse's observation of the patient's nonverbal behaviors. Page: 31 Some use of history forms and note-taking may be unavoidable. But be aware that note-taking during the interview has disadvantages. It breaks eye contact too often, and it shifts attention away from the patient, which diminishes his or her sense of importance. It also may interrupt the patient's narrative flow, and it impedes the observation of the patient's nonverbal behavior.

59. The nurse suspects that a patient has hyperthyroidism and laboratory data indicate that the patient's thyroxine and tri-iodothyronine hormone levels are elevated. Which of these findings would the nurse most likely find on examination? A) Tachycardia B) Constipation C) Rapid dyspnea D) Atrophied nodular thyroid

A) Tachycardia Thyroxine and tri-iodothyronine are thyroid hormones that stimulate the rate of cellular metabolism, resulting in tachycardia. With an enlarged thyroid as in hyperthyroidism, the nurse might expect to find diffuse enlargement (goiter) or a nodular lump, but not an atrophied gland. Dyspnea and constipation are not findings associated with hyperthyroidism.

45. The nurse is providing care for a 68-year-old woman who is complaining of constipation. What concern exists regarding her nutritional status? A) The absorption of nutrients may be impaired. B) The constipation may represent a food allergy. C) She may need emergency surgery for the problem. D) The gastrointestinal problem will increase her caloric demand.

A) The absorption of nutrients may be impaired. Page: 182. Gastrointestinal symptoms such as vomiting, diarrhea, or constipation may interfere with nutrient intake or absorption. The other responses are not correct.

120. During the assessment of an 80-year-old patient, the nurse notices that his hands show tremors when he reaches for something and his head is always nodding. There is no associated rigidity with movement. Which of these statements is most accurate? A) These are normal findings resulting from aging. B) These could be related to hyperthyroidism. C) These are the result of Parkinson disease. D) This patient should be evaluated for a cerebellar lesion.

A) These are normal findings resulting from aging. Page: 659. Senile tremors occasionally occur. These benign tremors include an intention tremor of the hands, head nodding (as if saying yes or no), and tongue protrusion. Tremors associated with Parkinson disease include rigidity, slowness, and weakness of voluntary movement. The other responses are incorrect.

90. The nurse is assessing voice sounds during a respiratory assessment. Which of these findings indicates a normal assessment? Select all that apply. A) Voice sounds are faint, muffled, and almost inaudible when the patient whispers "one, two, three" in a very soft voice. B) As the patient says "ninety-nine" repeatedly, the examiner hears the words "ninety-nine" clearly. C) When the patient speaks in a normal voice, the examiner can hear a sound but cannot distinguish exactly what is being said. D) As the patient says a long "ee-ee-ee" sound, the examiner also hears a long "ee-ee-ee" sound. E) As the patient says a long "ee-ee-ee" sound, the examiner hears a long "aaaaaa" sound.

A) Voice sounds are faint, muffled, and almost inaudible when the patient whispers "one, two, three" in a very soft voice. C) When the patient speaks in a normal voice, the examiner can hear a sound but cannot distinguish exactly what is being said. D) As the patient says a long "ee-ee-ee" sound, the examiner also hears a long "ee-ee-ee" sound. Page: 446. As a patient says "ninety-nine" repeatedly, normally, the examiner hears sound but cannot distinguish what is being said. If a clear "ninety-nine" is auscultated, then it could indicate increased lung density, which enhances transmission of voice sounds. This is a measure of bronchophony. When a patient says a long "ee-ee-ee" sound, normally the examiner also hears a long "ee-ee-ee" sound through auscultation. This is a measure of egophony. If the examiner hears a long "aaaaaa" sound instead, this could indicate areas of consolidation or compression. With whispered pectoriloquy, as when a patient whispers a phrase such as "one-two-three," the normal response when auscultating voice sounds is to hear sounds that are faint, muffled, and almost inaudible. If the examiners hears the whispered voice clearly, as if the patient is speaking through the stethoscope, then consolidation of the lung fields may exist.

85. The nurse is listening to the breath sounds of a patient with severe asthma. Air passing through narrowed bronchioles would produce which of these adventitious sounds? A) Wheezes B) Bronchial sounds C) Bronchophony D) Whispered pectoriloquy

A) Wheezes Page: 445. Wheezes are caused by air squeezed or compressed through passageways narrowed almost to closure by collapsing, swelling, secretions, or tumors, such as with acute asthma or chronic emphysema.

64. The nurse is performing an assessment on a 7-year-old child who has symptoms of chronic watery eyes, sneezing, and clear nasal drainage. The nurse notices the presence of a transverse line across the bridge of the nose, dark blue shadows below the eyes, and a double crease on the lower eyelids. These findings are characteristic of: A) allergies. B) a sinus infection. C) nasal congestion. D) an upper respiratory infection.

A) allergies. Page: 275. Chronic allergies often develop chronic facial characteristics. These include blue shadows below the eyes, a double or single crease on the lower eyelids, open-mouth breathing, and a transverse line on the nose.

86. An adult patient with a history of allergies comes to the clinic complaining of wheezing and difficulty in breathing when working in his yard. The assessment findings include tachypnea, use of accessory neck muscles, prolonged expiration, intercostal retractions, decreased breath sounds, and expiratory wheezes. The nurse interprets that these assessment findings are consistent with: A) asthma. B) atelectasis. C) lobar pneumonia. D) heart failure.

A) asthma. Page: 451. Asthma is allergic hypersensitivity to certain inhaled particles that produces inflammation and a reaction of bronchospasm, which increases airway resistance, especially during expiration. Increased respiratory rate, use of accessory muscles, retraction of intercostal muscles, prolonged expiration, decreased breath sounds, and expiratory wheezing are all characteristic of asthma. See Table 18-8 for descriptions of the other conditions.

89. The nurse is auscultating the lungs of a patient who had been sleeping and notices short, popping, crackling sounds that stop after a few breaths. The nurse recognizes that these breath sounds are: A) atelectatic crackles, and that they are not pathologic. B) fine crackles, and that they may be a sign of pneumonia. C) vesicular breath sounds. D) fine wheezes.

A) atelectatic crackles, and that they are not pathologic. Pages: 429-430. One type of adventitious sound, atelectatic crackles, is not pathologic. They are short, popping, crackling sounds that sound like fine crackles but do not last beyond a few breaths. When sections of alveoli are not fully aerated (as in people who are asleep or in the elderly), they deflate slightly and accumulate secretions. Crackles are heard when these sections are expanded by a few deep breaths. Atelectatic crackles are heard only in the periphery, usually in dependent portions of the lungs, and disappear after the first few breaths or after a cough.

17. The nurse recognizes that working with children with a different cultural perspective may be especially difficult because: A) children have spiritual needs that are influenced by their stages of development. B) children have spiritual needs that are direct reflections of what is occurring in their homes. C) religious beliefs rarely affect the parents' perceptions of the illness. D) parents are often the decision makers, and they have no knowledge of their children's spiritual needs.

A) children have spiritual needs that are influenced by their stages of development. Page: 20. Illness during childhood may be an especially difficult clinical situation. Children, as well as adults, have spiritual needs that vary according to the child's developmental level and the religious climate that exists in the family. The other statements are not correct.

37. When percussing over the liver of a patient, the nurse notices a dull sound. The nurse should: A) consider this a normal finding. B) palpate this area for an underlying mass. C) reposition the hands and attempt to percuss in this area again. D) consider this an abnormal finding and refer the patient for additional treatment.

A) consider this a normal finding. Pages: 116-117. Percussion over relatively dense organs, such as the liver or spleen, will produce a dull sound. The other responses are not correct.

76. When assessing a patient's lungs, the nurse recalls that the left lung: A) consists of two lobes. B) is divided by the horizontal fissure. C) consists primarily of an upper lobe on the posterior chest. D) is shorter than the right lung because of the underlying stomach.

A) consists of two lobes. Pages: 413-414. The left lung has two lobes, and the right lung has three lobes. The right lung is shorter than the left lung because of the underlying liver. The left lung is narrower than the right lung because the heart bulges to the left. The posterior chest is almost all lower lobe.

82. The nurse is percussing over the lungs of a patient with pneumonia. The nurse knows that percussion over an area of atelectasis in the lungs would reveal: A) dullness. B) tympany. C) resonance. D) hyperresonance.

A) dullness. Pages: 424-425. A dull percussion note signals an abnormal density in the lungs, as with pneumonia, pleural effusion, atelectasis, or tumor.

69. A 68-year-old woman is in the eye clinic for a checkup. She tells the nurse that she has been having trouble with reading the paper, sewing, and even seeing the faces of her grandchildren. On examination, the nurse notes that she has some loss of central vision but her peripheral vision is normal. These findings suggest that: A) she may have macular degeneration. B) her vision is normal for someone her age. C) she has the beginning stages of cataract formation. D) she has increased intraocular pressure or glaucoma.

A) she may have macular degeneration. Page: 285. Macular degeneration is the most common cause of blindness. It is characterized by loss of central vision. Cataracts would show lens opacity. Chronic open-angle glaucoma, the most common type of glaucoma, involves a gradual loss of peripheral vision.

50. The nurse is performing a nutritional assessment on an 80-year-old patient. The nurse knows that physiological changes that directly affect the nutritional status of the elderly include: A) slowed gastrointestinal motility. B) hyperstimulation of the salivary glands. C) an increased sensitivity to spicy and aromatic foods. D) decreased gastrointestinal absorption causing esophageal reflux.

A) slowed gastrointestinal motility. Page: 176. Normal physiological changes in aging adults that affect nutritional status include slowed gastrointestinal motility, decreased gastrointestinal absorption, diminished olfactory and taste sensitivity, decreased saliva production, decreased visual acuity, and poor dentition.

Health risks associated with obesity include (select all that apply) a. colorectal cancer b. rheumatoid arthritis c. polycystic ovary disease d. nonalcoholic steatohepatitis e. systemic lupus erythematosus

A, C, D

When caring for a patient with a biliary obstruction, the nurse will anticipate administering which of the following vitamin supplements (select all that apply)? A) a. Vitamin A B) b. Vitamin D C) c. Vitamin E D) d. Vitamin K E) e. Vitamin B

A,B,C,D

Which of the following criteria are needed for a diagnosis of metabolic syndrome (select all that apply) a. hypertension b. elevated triglycerides c. elevated plasma glucose d. increased waist circumference e. decreased low density lipoproteins

A,B,C,D

When caring for a patient with liver disease, the nurse recognizes the need to prevent bleeding resulting from altered clotting factors and rupture of varices. Which of the following nursing interventions would be appropriate to achieve this outcome (select all that apply)? A) a. Use smallest gauge possible when giving injections or drawing blood. B) b. Teach patient to avoid straining at stool, vigorous blowing of nose, and coughing. C) c. Advise patient to use soft-bristle toothbrush and avoid ingestion of irritating food. D) d. Apply gentle pressure for the shortest possible time period after performing venipuncture. E) e. Instruct patient to avoid aspirin and NSAIDs to prevent hemorrhage when varices are present.

A,B,C,E

A normal physical assessment finding of the GI system is/are: (select all the apply) a. nonpalpable liver and spleen b. borborygmi in upper right quadrant c. tympany on percussion of the abdomen d. liver edge 2 to 4 cm below the costal margin e. finding a firm nodular edge on the rectal exam

A,C

The nurse performs a detailed assessment of the abdomen of a patient with a possible bowel obstruction, knowing that a manifestation of an obstruction in the large intestine is (select all that apply) a. a largely distended abdomen b. diarrhea that is loose or liquid c. persistent, colicky abdominal pain d. profuse vomiting that relieves abdominal pain

A,C

A community health nurse is conducting an initial assessment of a new patient. Which of the following assessments would the nurse include when screening the patient for metabolic syndrome (select all that apply)? A) a. Blood pressure B) b. Resting heart rate C) c. Physical endurance D) d. Waist circumference E) e. Fasting blood glucose

A,D,E

Nursing management of the patient with acute pancreatitis includes (select all that apply) a. checking for signs of hypocalcemia b. providing a diet low in carbohydrates c. giving insulin based on a sliding scale d. observing stools for signs of steatorrhea e. monitoring for infection, particularly respiratory infection

A,E

A client with gastroesophageal reflux disease complains about having difficulty sleeping at night, what should the nurse instruct the client to do? A. sleep on several pillows B. eliminate carbohydrates from the diet C. suggest a glass of milk before retiring D. take antacids such as sodium bicarbonate

A. sleeping on pillows raises the upper torso and minimizes reflux of the gastic contents

Rolling Hernia

AKA paraesophageal hernia- only the fundus and part of the greater curvature roll into the thorax beside the esophagus. Herniated portion of the stomach can be small or large. Reflus is not usually present.

A client receives a local anesthetic to suppress the gag reflex for a diagnostic procedure of the upper GI tract. Which of the following nursing interventions is advised for this patient? a The client should be monitored for any breathing related disorder or discomforts b) The client should not be given any food and fluids until the gag reflex returns, c. The client should be monitored for cramping or abdominal distention, d) The client's fluid output should be measured for at least 24 hours after the procedure

ANSWER: B For a client receiving a local anesthetic that suppresses the gag reflex, the nurse is advised to withhold food and fluids until the reflex returns

Pre-OP teaching for Esophageal Cancer

Advise the patient to stop smoking 2-4 weeks before surgery, Include nutritional support to decrease their risk for postoperative complications, Many patients require feeding tubes, monitor weight and I&O, treat any dental disease, teach oral care 4 times daily

"The results of a patient's recent endoscopy indicate the presence of peptic ulcer disease (PUD). Which of the following teaching points should the nurse provide to the patient in light of his new diagnosis? "A) "You'll need to drink at least two to three glasses of milk daily." B) "It would likely be beneficial for you to eliminate drinking alcohol." C) "Many people find that a minced or pureed diet eases their symptoms of PUD." D) "Your medications should allow you to maintain your present diet while minimizing symptoms

Although there is no specific recommended dietary modification for PUD, most patients find it necessary to make some sort of dietary modifications to minimize symptoms. Milk may exacerbate PUD and alcohol is best avoided because it can delay healing

The nurse caring for a client diagnosed with GERD writes the client problem of "behavior modification." which intervention should be included for this problem? d. 2. encourage the client to decrease the amount of smoking. 3. instruct the client to take over the counter medication for relief of pain. 4. discuss the need to attend alcoholics anonymous to quit drinking

Answer 1 (correct): the client should elevate the hdad of the bed on blocks or use afoam wedge to use gravity to help keep the grastric acid in the stomach and prevent reflux into the esophagus. behavior modification is changing one's behavior. 2. client should not reduce but quite smoking altogether. 3. nurse should be careful when suggesting OTC meds. 4. should stem alcohol but no indication client is an alcoholic

"The nurse is caring for an adult client diagnosed with gastroesophageal reflux disease(GERD). Which condition is the most common comorbid disease associated with GERD? 1.Adult-onset asthma. 2.Pancreatitis. 3.Peptic ulcer disease. 4.Increased gastric emptying

Answer 1, "1. CORRECT - Of adult-onset asthma cases, 80%-90% are caused by gastroesophageal reflux disease(GERD) 2. Pancreatitis is not related to GERD 3. Peptic ulcer disease is related to H. pylori bacterial infections and can lead to increased levelsof gastric acid, but it is not related to reflux. 4.GERD is not related to increased gastricemptying. Increased gastric emptying would bea benefit to a client with decreased functioningof the lower esophageal sphincter

The client with hiatal hernia chronically experiences heartburn following meals. The nurses plans to teach the client to avoid which action because it is contraindicated with a hiatal hernia? 1. Lying recumbent following meals 2. Taking in small, frequent, bland meals 3. Raising the head of bed on 6-inch blocks 4. Taking H2-receptor antagonist medication

Answer 1, Hiatal hernia is caused by a protrusion of a portion of the stomach above the diaphragm where the esophagus usually is positioned. The client usually experiences pain from reflux caused by ingestion of irritating foods, lying flat following meals or at night, and eating large or fatty meals. Relief is obtained with the intake of small, frequent meals, use of H2-receptor antagonists and antacids, and elevation of the thorax following meals and during sleep

A patient has a vagotomy with antrectomy to treat a duodenal ulcer. Postoperatively, the patient develops dumping syndrome. Which of the following statements, if made by the patient, should indicate to the nurse that further dietary teaching is needed? 1. I should eat bread with each meal 2. I should eat smaller meals more frequently. 3. I should lie down after eating. 4. I should avoid drinking fluids with my meals

Answer 1, Patient should decrease intake of carbohydrates

The client with hiatal hernia chronically experiences heartburn following meals. The nurse plans to teach the client to avoid which action because it is contraindicated with a hiatal hernia? 1. Lying recumbent following meals. 2. Taking in small, frequent bland meals. 3. Raising the head of the bed on 6-inch block. 4. Taking H2-receptor antagonist medication

Answer 1,Hiatal hernia is caused by a protrusion of a portion of the stomach above the diaphragm where the esophagus is normally positioned. he client usually experiences pain from reflux caused by ingestion of irritating foods, lying flat following meals or at night, and eating large or fatty meals. Option 2-4, and actually elevating the thorax after a meal, provide relief

The male client tells the nurse he has been experiencing "heartburn" at night that awakens him. Which assessment question should the nurse ask? "1. "How much weight have you gained recently?" 2. "What have you done to alleviate the heartburn?" 3. "Do you consume many milk and dairy products?" 4. "Have you been around anyone with a stomach virus?

Answer 2, "1. Clients with heartburn are frequently diagnosed as having GERD. GERD can occasionally cause weight loss, but not weight gain. 2. Most clients with GERD have been self- medicating with over-the-counter medications prior to seeking advice from a health-care provider. It is important to know what the client has been using to treat the problem. 3. Milk and dairy products contain lactose, which are important if considering lactose intolerance, but are not important for "heartburn." 4. Heartburn is not a symptom of a viral illness

The client is diagnosed with an acute exacerbation of ulcerative colitis. Which inter- vention should the nurse implement? 1. Provide a low-residue diet. 2.Monitor intravenous fluids. 3.Assess vital signs daily.4.Administer antacids orally

Answer 2, "1. The client's bowel should be placed on rest andno foods or fluids should be introduced intothe bowel. 2. (Correct) The client requires fluids to help prevent dehydration from diarrhea and to replacethe fluid lost through normal body func-tioning. 3.The vital signs must be taken more often thandaily in a client who is having an acute exacer-bation of ulcerative colitis. 4.The client will receive anti-inflammatory andantidiarrheal medications, not antacids, whichare used for gastroenteritis

"The nurse is caring for the client diagnosed with chronic gastritis. Which symptom(s) would support this diagnosis? 1. Rapid onset of mid-sternal discomfort. 2. Epigastric pain relieved by eating food 3. Dyspepsia and hematemesis. 4. Nausea and projectile vomiting

Answer 2, "Rationale by answer: 1. Acute gastritis is characterized by sudden epigastric pain or discomfort, not mid-sternal chest pain. 2. Chronic pain in the epigastric area that is relieved by ingesting food is a sign of chronic gastritis (CORRECT). 3. Dyspepsia (heartburn) and hematemesis (vomiting blood) are frequent symptoms of acute gastritis. 4. Projective vomiting is not a sign of chronic gastritis

"Which statement made by the client indicates to the nurse the client may be experiencing GERD? 1. "My chest hurts when I walk up the stairs in my home." 2. "I take antacid tablets with me wherever I go." 3. My spouse tells me I snore very loudly at night." 4. I drink six (6) to seven (7) soft drinks every day

Answer 2, 1. Pain in the chest when walking up stairs indicates angina. 2. Frequent use of antacids indicates an acid reflux problem. 3. Snoring loudly could indicate sleep apnea, but not GERD. 4. Carbonated beverages increase stomach pressure. Six to seven soft drinks a day would not be tolerated by a client with GERD

What response should a nurse offer to a client who asks why he's having a vagotomy to treat his ulcer? 1. To repair a hole in the stomach 2. to reduce the ability of the stomach to produce acid 3. to prevent the stomach from sliding into the chest 4. to remove a potentially malignant lesion in the stomach

Answer 2: A vagotomy is perfomred to elimniate the acid-secreting stimulus to gastric cells. a perforation would be repaired with a gastric resection. Repair of hiatal hernia (fundoplication) prevents the stomach from sliding through the diaphragm. Removal of a potentially malignant tumor wouldn't reduce the entire acid-producing mechanism

The nurse is assessing the client diagnosed with chronic gastritis. Which symptom(s) support this diagnosis? 1. Rapid onset of midsternal discomfort 2. Epigastric pain relieved by eating food 3. Dyspepsia and hematemesis 4. Nausea and projectile vomiting

Answer 2: Chronic pain in the epigastric area relieved by ingesting food is a sign of chronic gastritis

A male client is diagnosed with acute gastritis secondary to alcoholism and cirrhosis. When obtaining the client's history, the nurse gives priority to the client's statement that: 1) His pain increases after meals. 2) He experiences nausea frequently. 3) His stools have a black appearance. 4) He recently joined Alcoholics Anonymous

Answer 3, 1) Investigation of bleeding takes prioritylater the nurse should help to identify irritating foods that are to be avoided. 2) Nausea is a common symptom of gastritis, but it is not life threatening. 3) Black (tarry) stools indicate upper GI bleedingdigestive enzymes act on the blood resulting in tarry stools. Hemorrhage can occur if erosion extends to blood vessels. 4) Attempts to control alcoholism should be supported but this is a long-term goalassessment of bleeding takes priority

Gastroesophageal reflux disease (GERD) weakens the lower esophageal sphincter, predisposing older persons to risk for impaired swallowing. In managing the symptoms associated with GERD, the nurse should assign the highest priority to which of the following interventions? 1. Decrease daily intake of vegetables and water, and ambulate frequently 2. Drink coffee diluted with milk at each meal, and remain in an upright position for 30 minutes. 3. Eat small, frequent meals, and remain in an upright position for at least 30 minutes after eating 4. Avoid over-the-counter drugs that have antacids in them

Answer 3, Eating small and frequent meals requires less release of hydrochloric acid. Remaining in an upright position for 30 minutes after meals prevents reflux into the esophagus which is often exacerbated when lying down, expecially after a large meal which makes the patient tired

which is the most common upper GI problem? "1. peptic ulcer disease 2. Crohns 3. Gerd 4. ulcerative colitis

Answer 3, Gerd is the only upper GI problem

The client with gastroesophageal reflux disease (GERD) complains of a chronic cough. The nurse understands that in a client with GERD this symptom may be indicative of which of the following conditions? "1. Development of laryngeal cancer. 2. Irritation of the esophagus. 3. Esophageal scar tissue formation. 4. Aspiration of gastric contents

Answer 4, "Clients with GERD can develop pulmonary symptoms, such as coughing, wheezing, and dyspnea, that are caused by the aspiration of gastric contents. GERD does not predispose the client to the development of laryngeal cancer. Irritation of the esophagus and esophageal scar tissue formation can develop as a result of GERD. However, GERD is more likely to cause painful and difficult swallowing

The doctor has ordered Tagamet for a client admitted with gastroesophageal reflux disease (GERD). After looking up the drug in the Physician's Desk Reference, you understand it is being used to:1. Neutralize stomach acid. 2. Treat a hiatal hernia. 3. Aid in the digestion of food. Decrease stomach acid production

Answer 4, 4. Treatment for GERD includes medications such as Tagamet to decrease stomach acid production and promote healing of esophagus

The nurse is monitoring a client with a diagnosis of peptic ulcer. Which assessment finding would most likely indicate perforation of the ulcer? 1. Bradycardia 2. Numbness in the legs 3. Nausea and vomiting 4. A rigid, board-like abdomen

Answer 4, Perforation of an ulcer is a surgical emergency and is characterized by sudden, sharp, intolerable severe pain beginning in the midepigastric area and spreading over the abdomen, which become rigid and board-like. Nausea and vomiting may occur. Tachycardia may occur as hypovolemic shock develops. Numbness in the legs is not an associated finding

The nurse is monitoring a client with a diagnosis of peptic ulcer. Which assessment finding would most likely indicate perforation of the ucler? 1. Bradycardia 2. Numbness in the legs 3. Nausea and vomiting 4. A rigid, board-like abdomen

Answer 4, Rationale: Perforation of an ulcer is a surgical emergency and is characterized by sudden, sharp, intolerable severe pain beginning in the midepigastric area and spreading over the abdomen, which becomes rigid and board-like. Nausea and vomiting may occur. Tachycardia may occur as hypovolemic shock develops. Numbness in the legs is not an associated finding

The nurse is preparing a client diagnosed with GERD for discharge following an esophagogastroduodenoscopy. Which statement indicates the client understands the discharge instructions?7 1. I should not eat for 24 hours following this procedure. 2 I can lie down whenever iI want after a meal. It own't make a difference. 3. The stomach contents won't bother my esophagus but will make me nauseous. 4. I should avoid drinking orange jice and eating tomatoes until my esophagus heals.

Answer 4, oragne and tomato juices are acidic, and the client diagnosed with GERD shouldavoid acidic foods until the esophagus hashad a chance to heal - A client hospitalized with a gastric ulcer is scheduled for discharge.

the nurse is monitoring a client with a diagnosis of peptic ulcer. which assessment finding would most likely indicate perforation of the ulcer? 1. bradycardia 2. numbness in legs 3. N&V 4. a rigid board-like abdomen

Answer 4, perforation of ulcer is a surgical emergency and is characterized by sudden, sharp, intolderable severe pain beginning in the midepigastric area and spreading over the abdomen, which becomes rigid and board-like. nausea and vomiting may also occur. tachycardia may occur as hypovolemic shock develops. numbness of the legs is not an associated finding

The nurse is monitoring a female client with a diagnosis of peptic ulcer. 1.Bradycardia 2.Numbness in the legs 3.Nausea and vomiting 4.A rigid, board-like abdomen

Answer 4,1.Tachycardia may occur as hypovolemic shock develops. 2.Numbness in the legs is not an associated finding. 3.Nausea and vomiting may occur. 4.Perforation of an ulcer is a surgical emergency and is characterized by sudden, sharp, intolerable severe pain beginning in the midepigastric area and spreading over the abdomen, which becomes rigid and board-like

"A patient with a history of peptic ulcer disease has presented to the ED with complaints of severe abdominal pain and a rigid, boardlike abdome, prompting the health care team to suspect a perforated ulcer. Which of the following actions should the nurse anticiptate? source: http://quizlet.com/20002414/nclex-review-lower-gi-problems-ibd-crohns-disease-ulcerative-colitis-flash-cards/ or Lewis chapt. 42 Upper GI NCLEX" "A: Providing IV fluids and inserting a nasogastric tube B:Administering oral bicarbonate and testing patient's gastric pH level C:Performing a fecal occult blood test and administering IV calcium gluconate D: Starting parenteral nutrition and placing the patient in high-fowler's position

Answer A, "A: providing IV fluids and inserting a nasogastric tube rationale: A perforated peptic ulcer requires IV replacement of fluid losses and continued gastric aspiration by NG tube. Nothing is given by mouth and gastric pH testing is not a priority. Calcium gluconate is not a medication directly relevent to the patient's suspected diagnosis and parenteral nutrition is not a priority in the short term

"A patient with a history of peptic ulcer disease has presented to the emergency department with complaints of severe abdominal pain and a rigid, boardlike abdomen, prompting the health care team to suspect a perforated ulcer. Which of the following actions should the nurse anticipate? A. Providing IV fluids and inserting a nasogastric tube B. Administering oral bicarbonate and testing the patient's gastric pH level C. Performing a fecal occult blood test and administering IV calcium gluconate D. Starting parenteral nutrition and placing the patient in a high-Fowler's position

Answer A, A perforated peptic ulcer requires IV replacement of fluid losses and continued gastric aspiration by NG tube. Nothing is given by mouth and gastric pH testing is not a priority. Calcium gluconate is not a medication directly relevant to the patient's suspected diagnosis and parenteral nutrition is not a priority in the short term

A patient with a history of peptic ulcer disease has presented to the emergency department with complaints of severe abdominal pain and a rigid, boardlike abdomen, prompting the health care team to suspect a perforated ulcer. Which of the following actions should the nurse anticipate? A. Providing IV fluids and inserting a nasogastric tube B. Administering oral bicarbonate and testing the patient's gastric pH level. C. Performing a fecal occult blood test and administering IV calcium gluconate. D. Starting parenteral nutrition and placing the patient in high-Fowler's position

Answer A, A perforated peptic ulcer requires IV replacement of fluid losses and continued gastric aspiration by NG tube. Nothing is given by mouth and gastric pH testing is not a priority. Calcium gluconate is not a medication directly relevant to the patient's suspected diagnosis and parenteral nutrition is not a priority in the short term

Which of the following nursing interventions should the nurse perform for a female client receiving enteral feedings through a gastrostomy tube? "a. Change the tube feeding solutions and tubing at least every 24 hours b. Maintain the head of the bed at a 15-degree elevation continuously. c. Check the gastrostomy tube for position every 2 days. d. Maintain the client on bed rest during the feedings

Answer A, Answer A. Tube feeding solutions and tubing should be changed every 24 hours, or more frequently if the feeding requires it. Doing so prevents contamination and bacterial growth. The head of the bed should be elevated 30 to 45 degrees continuously to prevent aspiration. Checking for gastrostomy tube placement is performed before initiating the feedings and every 4 hours during continuous feedings. Clients may ambulate during feedings

The teaching plan for the patient being discharged following an acute episode of upper GI bleeding will concern information concerning the importance of (select all that apply) a. only taking aspirin with milk or bread products b. avoiding taking aspirin and drugs containing aspirin c. taking only drugs prescribed by the health care provider d. taking all drugs 1 hour before mealtime to prevent further bleeding e. reading all OTC drug labels to avoid those containing stearic acid and calcium

Answer A, C Aspirin contributes to thinning the blood and is linked to causing things like peritonitis further increasing the risk for bleeding. Taking only health care prescribed drugs can greatly reduce the risk from accidentally using OTC meds that might contribute to bleeding

The client attends two sessions with the dietitian to learn about diet modifications to minimize gastroesophageal reflux. The teaching would be considered successful if the client says that she will decrease her intake of which of the following foods? A) Fats, B)Carbohydrates, C) High-calcium foods, D) High-Sodium foods

Answer A, Fats are associated with decreased esophageal sphincter tone, which increases reflux. Obesity contributes to the development of hiatal hernia, and a low-fat diet might also aid in weight loss. Carbohydrates and foods high in sodium or calcium do not affect gastroesophageal reflux

"The nurse is reviewing the record of a client with Crohn's disease. Which stool characteristic should the nurse expect to note documented in the client's record? "a. Diarrhea b. Chronic constipation c. Constipation alternating with diarrhea d. Stool constantly oozing from the rectum

Answer A,"(A) Crohns disease is characterized by nonbloody diarrhea of usually not more than 4 or 5 stools daily. overtime the stools increase frequency duration and severity

The patient who is admitted with a diagnosis of diverticulitis and a history of irritable bowel disease and gastroesophageal reflux disease (GERD) has received a dose of Mylanta 30 ml PO. The nurse would evaluate its effectiveness by questioning the patient as to whether which of the following symptoms has been resolved? A-Diarrhea B.Heartburn C.Constipation D. Lower abdominal pain

Answer B, "Mylanta is an antacid that contains both aluminum and magnesium. It is indicated for the relief of GI discomfort, such as with heartburn associated with GERD

What statement made by the client indicates to the nurse the client may be experiencing GERD?n "A. ""My chest hurts when I walk up the stairs in my home"" B. ""I take antacid tablets with me wherever I go"" C. ""My spouse tells me I snore very loudly at night"" D. ""I drink 6 to 7 soft drinks every day

Answer B, (B) Frequent use of antacids indicates an acid reflux problem

"The nurse has instructed the client who is experiencing diarrhea associated with irritable bowel syndrome on dietary changes to prevent diarrhea. The nurse knows the client understands the dietary changes if the client selects which of the following menu choices? a) Yogurt, crackers and sweet tea b) Salad with chicken, whole wheat crackers c) Bacon, tomato, lettuce with mayonnaise and a soft drink d) Tuna on white bread and coconut cake

Answer B,Rationale: Bacon tomato lettuce with mayonnaise and soft drink is high in fat and the soft drink is hyperosmolar both contributing to diarrhea. Salad, whole wheat crackers may decrease diarrhea due to increased fiber. Dairy increases diarrhea. Food high in carbohydrates increase diarrhea. Coconut may increase diarrhea

The nurse is caring for a male client with a diagnosis of chronic gastritis. The nurse monitors the client knowing that this client is at risk for which vitamin deficiency? a. Vitamin A b. Vitamin B12 c. Vitamin C d. Vitamin E

Answer B. Chronic gastritis causes deterioration and atrophy of the lining of the stomach, leading to the loss of the function of the parietal cells. The source of the intrinsic factor is lost, which results in the inability to absorb vitamin B12. This leads to the development of pernicious anemia. The client is not at risk for vitamin A, C, or E deficiency

"The nurse determines that a patient has experienced the beneficial effects of medication therapy with famotidine (Pepcid) when which of the following symptoms is relieved? "A) Nausea B) Belching C) Epigastric pain D) Difficulty swallowing

Answer C, "Famotidine is an H2-receptor antagonist that inhibits parietal cell output of HCl acid and minimizes damage to gastric mucosa related to hyperacidity, thus relieving epigastric pain

The nurse is caring for a female client with active upper GI bleeding. What is the appropriate diet for this client during the first 24 hours after admission? a. regular diet b. skim milk c. nothing by mouth d. clear liquids

Answer C, Answer C. Shock and bleeding must be controlled before oral intake, so the client should receive nothing by mouth. A regular diet is incorrect. When the bleeding is controlled, the diet is gradually increased, starting with ice chips and then clear liquids. Skim milk shouldn't be given because it increases gastric acid production, which could prolong bleeding. A liquid diet is the first diet offered after bleeding and shock are controlled

5. Which of the following drugs is a histamine blocker and reduces levels of gastric acid?"A. Omeprazole (Prilosec) B. Metoclopramide (Reglan) C. Cimetidine (Tagamet) D. Magnesium Hydroxide (Maalox)

Answer C, Cimetidine bind to H2 in the tissue and decreases the production of gastric acid

The most frequently used diagnostic test for persons with GERD is:a) Barium enema b) upper endoscopy c) barium swallow d) acid perfusion test

Answer C, Persons with GERD should be referred to a primary care provider for a thorough cardiac evaluation to rule out cardiac disease. The most frequently used diagnostic test is barium swallow. Upper endoscopy is the best method to assess mucosal injury. Acid perfusion tests usually are not necessary, and require the placement of an esophageal probe above the esophageal sphincter to collect esophageal contents

Gastroesophageal reflux disease (GERD) weakens the lower esophageal spinchter, predisposing older persons to risk for impaired swallowing. In managing the symptoms associated with GERD, the nurse should assign the highest priority to which of the following interventions? A. Decrease daily intake of vegetables and water and ambulate frequently, B. drink coffee diluted with milk at each meal and remain in an upright position for thirty minutes, C. Eat small, frequent meals and remain in an upright position for thirty minutes D. Avoid OTC drugs that have antacids in them

Answer C, Rationale: Small, frequent feedings requires less release of hydrochloric acid. Remaining in an upright position for 30 minutes after meals prevents reflux into the esophagus

Gastroesophageal reflux disease (GERD) weakens the lower esophageal sphincter, predisposing older persons to risk for impaired swallowing. In managing the symptoms associated with GERD, the nurse should assign the highest priority to which of the following interventions? A) Decrease daily intake of vegetables and water, and ambulate frequently. B) Drink coffee diluted with milk at each meal, and remain in an upright position for 30 minutes. C) Eat small frequent meals, and remain in an upright position for at least 30 minutes after eating. D) Avoid over-the-counter drugs that have antacids in them

Answer C, Small, frequent feedings requires less release of hydrochloric acid. Remaining in an upright position for 30 minutes after meals prevents reflux into the esophagus

"The nurse is teaching a client with a gastric ulcer about dietary management for the disease. Teaching is successful when the client states... Source: Lippincott's Review for NCLEX-RN" "A: "I should eat a low fiber diet to delay gastric emptying." B: "I cannot eat fruits and veggies because they cause too much gas." C: "As long as they don't bother my stomach, I can eat most foods. D: "I can eat bland foods to help my stomach heal

Answer C, The correct answer is C. The antiulcer diet is not severely restricted. it is the ideal to have small frequent feedings but the client can eat foods as long as they do not cause upset. Low fiber diets are more so used in Ulcerative Colitis. A bland diet is used for severe inflammation

22. Nurse Hannah is teaching a group of middle-aged men about peptic ulcers. When discussing risk factors for peptic ulcers, the nurse should mention: A. a sedentary lifestyle and smoking. B. a history of hemorrhoids and smoking, C. alcohol abuse and a history of acute renal failure. D. alcohol abuse and smoking

Answer D, Answer D. Risk factors for peptic (gastric and duodenal) ulcers include alcohol abuse, smoking, and stress. A sedentary lifestyle and a history of hemorrhoids aren't risk factors for peptic ulcers. Chronic renal failure, not acute renal failure, is associated with duodenal ulcers

"The nurse is teaching the patient and family about possible causative factors for peptic ulcers. The nurse explains that ulcer formation is a. caused by a stressful lifestyle and other acid-producing factors such as H. pylori. b. inherited within families and reinforced by bacterial spread of Staphylococcus aureus in childhood. c. promoted by factors that tend to cause oversecretion of acid, such as excess dietary fats, smoking, and H. pylori. d. promoted by a combination of possible factors that may result in erosion of the gastric mucosa, including certain drugs and alcohol

Answer D, Rationale: Peptic ulcers develop only in the presence of an acidic environment. However, an excess of hydrochloric acid (HCl) may not be necessary for ulcer development. The back-diffusion of HCl into the gastric mucosa results in cellular destruction and inflammation. Histamine is released from the damaged mucosa, resulting in vasodilation and increased capillary permeability and further secretion of acid and pepsin. A variety of agents (i.e., certain infections, medications, and lifestyle factors) can damage the mucosal barrier. Helicobacter pylori can alter gastric secretion and produce tissue damage leading to peptic ulcer disease. The response to H. pylori is likely influenced by a variety of factors, including genetics, environment, and diet. Ulcerogenic drugs, such as aspirin and NSAIDs, inhibit synthesis of prostaglandins, increase gastric acid secretion, and reduce the integrity of the mucosal barrier. Patients on corticosteroids, anticoagulants, and selective serotonin reuptake inhibitors (e.g., fluoxetine [Prozac]) are also at increased risk for ulcers. High-alcohol intake is associated with acute mucosal lesions. Alcohol stimulates acid secretion. Coffee (caffeinated and uncaffeinated) is a strong stimulant of gastric acid secretion. Psychologic distress, including stress and depression, can negatively influence the healing of ulcers after they have developed. Smoking also delays ulcer healing. Infection with herpes and cytomegalovirus (CMV) in immunocompromised patients may also lead to gastric ulcers

The nurse is reviewing the medication record of a female client with acute gastritis. Which medication, if noted on the client's record, would the nurse question? "a. Digoxin (Lanoxin) b. Furosemide (Lasix) c. Indomethacin (Indocin) d. Propranolol hydrochloride (Inderal)

Answer is C, Indomethacin (Indocin) is a nonsteroidal anti-inflammatory drug and can cause ulceration of the esophagus, stomach, or small intestine. Indomethacin is contraindicated in a client with gastrointestinal disorders. Furosemide (Lasix) is a loop diuretic. Digoxin is a cardiac medication. Propranolol (Inderal) is a β-adrenergic blocker. Furosemide, digoxin, and propranolol are not contraindicated in clients with gastric disorders

Diagnostic testing is planned for a patient with a suspected peptic ulcer. The nurse explains to the patient that the most reliable test for determining the presence and location of an ulcer is a(n) 1.endoscopy. 2.gastric analysis. 3.barium swallow. 4.serologic test for Helicobacter pylori.

Answer: 1 Rationale: Endoscopy is the primary tool for diagnosing the source of upper gastrointestinal bleeding.

The health care provider orders a 10% fat emulsion solution to be administered to a critically ill patient who is currently receiving peripheral parenteral nutrition. Which of the following assessment findings would alert the nurse to a systemic problem related to lipid administration? 1.The onset of vomiting and fever 2.Retention of fluid with peripheral edema 3.A random capillary blood glucose level of 148 mg/dl 4.Erythema, tenderness, and exudate at the catheter insertion site

Answer: 1 Rationale: Patients receiving lipids are at extreme risk for infection because fat emulsions at room temperature are a medium for microorganism growth. Catheter-related infections can occur in patients receiving parenteral nutrition; local or systemic manifestations of infection may occur. Adverse reactions from lipid administration include allergic reactions, dyspnea, cyanosis, fever, flushing, phlebitis, chest and back pain, and pain at the IV site. Hyperglycemia and fluid overload are other potential complications; however, infection would be the highest risk for patients receiving lipids.

Which of the following patients is it most important for the nurse to refer to the dietitian for a complete nutritional assessment? 1.A 38-year-old diabetic patient who is undergoing laser eye surgery 2.A 55-year-old with a history of alcoholism who is hospitalized with a fractured femur resulting from a fall 3.A 24-year-old who has been taking a burst of corticosteroid therapy for 1 week for treatment of an asthma exacerbation 4.A 45-year-old hospitalized with nausea and abdominal pain who has had no oral intake and has received only IV fluids of D5½NS for 6 days

Answer: 2 Rationale: An individual who is at nutritional risk should have a full nutritional assessment. The individual at greatest risk is the individual with chronic alcoholism and a fractured femur. Drugs such as corticosteroids place the individual at risk for malnutrition, but the person in option 3 is taking the medication only on a short-term basis. Patients with chronic illnesses (such as diabetes) (option 1) are at nutritional risk but not as high risk as option 2. The person in option 4 is at risk for malnutrition after 10 days of receiving only intravenous fluids.

When assessing the patient's nutritional status, the nurse asks which drugs the patient takes primarily because 1.foods alter the absorption or bioavailability of every drug. 2.if the patient skips a meal, medications may not be taken. 3.some drugs increase the requirements for essential nutrients. 4.medications should be taken with food to prevent GI irritation.

Answer: 3 Rationale: Certain drugs (such as folic acid, riboflavin, and fat-soluble vitamins) may increase nutrient requirements. Some medications can be taken with food; other medications must be taken without food. Food may alter the absorption or bioavailability of some drugs, but not all drugs.

Following discharge instructions for a patient who has had bariatric surgery for treatment of obesity, the nurse determines that additional teaching is needed when the patient says, 1. "I shouldn't eat concentrated sweets." 2. "I can eat small, frequent meals throughout the day." 3. "I should drink several glasses of fluids with my meals." 4. "I will need to have a cobalamin injection once a month."

Answer: 3 Rationale: Discharge teaching for a patient after bariatric surgery may include six small meals/day, diet high in protein and low in fat and carbohydrates, avoidance of ingestion of solids with fluids, avoidance of large amounts of fluids at one time, fluid intake possibly restricted to less than 1000 ml/day, and avoidance of sugary foods. The dietary restrictions will help to prevent dumping syndrome and will aid in weight loss. Cobalamin injections or intranasal spray will prevent cobalamin deficiency anemia.

After the nurse teaches a patient with gastroesophageal reflux disease (GERD) about recommended dietary modifications, which statement by the patient indicates that the teaching has been effective? 1."I can have a glass of low-fat milk at bedtime." 2."I will have to eliminate all spicy foods from my diet." 3."I will have to use herbal teas instead of caffeinated drinks." 4."I should keep something in my stomach all the time to neutralize the excess acids."

Answer: 3 Rationale: Patients with gastroesophageal reflux disease should avoid foods (such as tea and coffee) that decrease lower esophageal pressure. Patients should also avoid milk, especially at bedtime, as it increases gastric acid secretion. Patients may eat spicy foods unless these foods cause reflux. Small, frequent meals help prevent overdistention of the stomach, but patients should avoid late evening meals and nocturnal snacking.

A patient with persistent vomiting of 3 days' duration is seen at the urgent care center because of increasing weakness. Intravenous therapy with lactated Ringer's solution is started, and arterial blood gases (ABGs) are ordered. Which of the following ABG results would the nurse expect? 1.pH 7.4; PaCO2 40 mm Hg; HCO3- 25 mEq/L 2.pH 7.3; PaCO2 50 mm Hg; HCO3- 20 mEq/L 3.pH 7.6; PaCO2 30 mm Hg; HCO3- 40 mEq/L 4.pH 7.48; PaCO2 40 mm Hg; HCO3- 30 mEq/L

Answer: 3 Rationale: Vomiting is a cause of metabolic alkalosis; the arterial blood gases indicate partially compensated metabolic alkalosis. The pH is greater than 7.45 (alkalosis); the HCO3- is above 26 mEq/L (metabolic); and the PaCO2 is less than 35 mm Hg (partially compensated).

"Which of the following types of gastritis is associated with Helicobacter pylori and duodenal ulcers? 1. Erosive (hemorrhagic) gastritis 2. Fundic gland gastritis (type A) 3. Antral gland gastritis (type B) 4.Aspiring-induced gastric ulcer

Answer: 3 - Erosive (hemorrhagic) gastritis can be caused by ingestion of substances that irritate the gastric mucosa. Fundic gland gastritis (type A) is associated with diffuse severe mucosal atrophy and the presence of pernicious anemia. Antral gland gastritis (type B) is the most common form of gastritis, and is associated with Helicobacter pylori and duodenal ulcers

The nurse determines that teaching regarding a weight-loss program has been effective when the patient says, 1. "I will keep a diary of daily weights to chart my weight loss." 2. "I plan to lose 4 pounds a week until I have lost my goal of 60 pounds." 3. "I should not exercise more than what is required because increased activity increases the appetite." 4. "I plan to join a behavior-modification group to make permanent changes necessary for weight control."

Answer: 4 Rationale: Behavior-modification programs deemphasize the diet and focus on how and when to eat; support groups offer support and information on dieting tips. Patients should set a weight loss goal of 1 to 2 pounds per week. Weight should be checked weekly; daily weights are not recommended because of the frequent fluctuations resulting from retained water (including urine) and elimination of feces. No evidence indicates that increased activity promotes an increase in appetite or leads to dietary excess.

When teaching a patient with a history of upper GI bleeding to check the stools for blood, the nurse informs the patient that 1.if vomiting of bright red blood occurs, stools will not be black and sticky. 2.blood is never obvious in stools and must be detected by fecal occult blood testing. 3.acute bleeding in the upper GI tract will result in bright red blood in the stools. 4.stools that are black and tarry occur with prolonged bleeding from the stomach or small intestine.

Answer: 4 Rationale: Melena (black, tarry stools) indicates slow bleeding from an upper gastrointestinal source. The longer the passage of blood through the intestines, the darker the stool color, because of the breakdown of hemoglobin and the release of iron.

An important factor associated with both short-term and long-term weight-loss success is 1. higher initial body mass index. 2. simultaneous smoking cessation. 3. fewer dieting attempts in the past year. 4. a strong desire to improve appearance.

Answer: 4 Rationale: Motivation to lose weight is essential for a favorable and successful outcome.

Caffeinated beverages and smoking are risk factors to assess for in the development of what condition? A. Duodenal ulcers B. Peptic ulcers C. Helicobacter pylori D. Esophageal reflux

Answer: B PUD risk factors include family history, blood group O, smoking tobacco, and beverages containing caffeine

"The nurse explains to the patient with gastroesophageal reflux disease that this disorder: "A. results in acid erosion of the esophagus caused by frequent vomiting B. Will require surgical wrapping of the pyloric sphincter to control the symptoms C. Is the protrusion of a portion of the stomach into the esophagus through the opening in the diaphragm D. Often involves relaxation of the lower esophageal sphincter, allowing the stomach contents to back up into the esophagus

Answer: D. The acidic contents of the stomach touching the inside of the esophagus are responsible for the physical sensation known as "heart-burn" that is a cardinal symptom of GERD

The nurse explains to the patient with gastroesophageal reflux disease that this disorder: A. results in acid erosion and ulceration of the esophagus caused by frequent vomiting, B. will require surgical wrapping or repair of the pyloric sphincter to control the symptoms, C. is the protrusion of a portion of the stomach into to esophagus through an opening in the diaphragm, D. often involves relaxation of the lower esophageal sphincter, allowing stomach contents to back up into the espophagus

Answer: D. The acidic contents of the stomach touching the inside of the esophagus are responsible for the physical sensation known as "heart-burn" that is a cardinal symptom of GERD

Which of these agents is a major contributing factor in the promotion of peptic ulcer disorder? A) Candida albicans. B) staphyloccus infection. C) streptococcus infection D) Helibacter pylori infection

Answer: Helobacter pylori infection. Rationale: Recurrence of peptic ulcers is related to Helicobacter pylori, use of NSAIDs, smoking, and continued acid hypersecretion

Diagnosis of patient with gastric surgery

Anxiety Pain Deficient knowledge Imbalanced nutrition

Chronic Gastritis

Appears as a patchy, diffuse inflammation of the mucosal lining of the stomach. Appears to be associated with an increased risk for gastric cancer

Prevention of Infection for tube feeding

Appropriate catheter and IV site care, strict sterile technique for dressing changes, wear a mask when changing dressing, assess insertion site, assess for indicators of infection, proper IV and tubing care

A 61-year-old patient with suspected bowel obstruction has had a nasogastric tube inserted at 4:00 am. The nurse shares in the morning report that the day shift staff should check the tube for patency at which of the following times? A) a. 7:00 am, 10:00 am, and 1:00 pm B) b. 8:00 am and 12:00 pm C) c. 9:00 am and 3:00 pm D) d. 9:00 am, 12:00 pm, and 3:00 pm

B

A colectomy is scheduled for a 68-year-old woman with an abdominal mass, possible bowel obstruction, and a history of rectal polyps. The nurse should plan to include which of the following prescribed measures in the preoperative preparation of this patient? A) a. Instruction on irrigating a colostomy B) b. Administration of a cleansing enema C) c. A high-fiber diet the day before surgery D) d. Administration of IV antibiotics for bowel preparation

B

A patient has an elevated blood level of indirect (unconjugated) bilirubin. Onecause of this finding is that: a. the gallbladder is unable to contract to release stored bile b. bilirubin is not being conjugated and excreted into the bile by the liver c. the Kupffer cells in the liver are unable to remove bilirubin from the blood d. there is an obstruction in the biliary tract preventing flow of bile into the small intestine

B

A patient has been told that she has elevated liver enzymes cause by nonalcoholic fatty liver disease. The nursing teaching plan should include: a. having genetic testing done b. recommending a heart-healthy diet c. the necessity to reduce weight rapidly d.avoiding alcohol until liver enzymes return to normal

B

A patient is receiving peripheral parenteral nutrition. The parenteral nutrition solution is completed before the new solution arrives on the unit. The nurse administers: a. 20% intralipids b. 5% dextrose solution c. 5% Ringer's lactate solution d. 0.45% normal saline solution

B

A patient who has dysphagia as a consequence of a stroke is receiving enteral feedings through a percutaneous endoscopic gastrostomy (PEG). Which of the following interventions should the nurse integrate into this patient's care? A) a. Flush the tube with 30 ml of normal saline every 4 hours. B) b. Flush the tube before and after feedings if the patient's feedings are intermittent. C) c. Flush the PEG with 100 ml of sterile water before and after medication administration. D) d. To prevent fluid overload, avoid flushing when the patient is receiving continuous feeding.

B

A patient who has suffered severe burns in a motor vehicle accident will soon be started on parenteral nutrition (PN). Which of the following principles should guide the nurse's administration of the patient's nutrition? A) a. Administration of PN requires clean technique. B) b. Central PN requires rapid dilution in a large volume of blood. C) c. Peripheral PN delivery is preferred over the use of a central line. D) d. Only water-soluble medications may be added to the PN by the nurse.

B

A patient who is administering a bisacodyl (Dulcolax) suppository asks the nurse how long it will take to work. The nurse replies that the patient will probably need to use the bedpan or commode within which of the following time frames after administration? A) a. 2-5 Minutes B) b. 15-60 Minutes C) c. 2-4 Hours D) d. 6-8 Hours

B

A patient with hepatitis B is being discharged in 2 days. In the discharge teaching plan the nurse should include instructions to: a. avoid alcohol for the first three weeks b. use a condom during sexual intercourse c. have family members get an injection of immunoglobulin d. follow a low-protein, moderate-carbohydrate, moderate-fat diet

B

An 80-year-old man states that although he adds a lot of salt to his food it still does not have much taste. The nurse's response is based on the knowledge that the older adult: a. should not experience changes in taste b. has a loss of taste buds, especially sweet and salt c. has some loss of taste but no difficulty chewing food d. loses the sense of taste because the ability to smell is decreased

B

As gastric contents move into the small intestine, the bowel is normally protected from the acidity of gastric contents by the: a. inhibition of secretin release b. release of bicarbonate by the pancreas c. release of pancreatic digestive enzymes d. release of gastrin by the duodenal mucosa

B

During an examination of the abdomen the nurse should: a. position the patient in the supine position with the bed flat and the knees straight b. listen in the epigastrium and all found quadrants for 2 to 5 minutes for bowel sounds c. use the following order of techniques: inspections, palpation, percussion, auscultation d. describe bowel sounds if no sound is heard in the lower right quadrant after two minutes

B

During starvation, the order in which the body obtains substrate for energy is: a. visceral protein, skeletal protein, fat, glycogen b. glycogen, skeletal protein, fat stores, visceral protein c. visceral protein, fat stores, glycogen, skeletal protein d. fat stores, skeletal protein, visceral protein, glycogen

B

In contrast to diverticulitis, the patient with diverticulosis: a. has rectal bleeding b. often has no symptoms c. has localized cramping pain d. frequently develops peritonitis

B

In preparing a patient for a colonoscopy, the nurse explains that: a. a signed permit is not necessary b. sedation may be used during the procedure c. only one cleansing enema is necessary for preparation d. a light meal should be eaten the day before the procedure

B

Inspection of an elderly patient's mouth reveals the presence of white, curd-like lesions on the patient's tongue. The most likely etiology for this abnormal assessment finding is A) a. Herpesvirus. B) b. Candida albicans. C) c. Vitamin deficiency. D) d. Irritation from ill-fitting dentures.

B

M.J. calls to tell the nurse that her elderly mother, who is 85 years of age, has been nauseated all day and has vomited twice. Before the nurse hangs up and telephones the health care provider to communicate your assessment data, the nurse should instruct M.J. to: a.administer antispasmodic drugs and observe skin turgor b. give her mother sips of water and elevate the head of her bed to prevent aspiration c. offer her mother a high-protein liquid supplement to drink to maintain her nutritional needs d. offer her mother large quantities of gatorade to drink because elderly people are at risk for sodium depletion

B

Several patients are seen at an urgent care center with symptoms of nausea, vomiting, and diarrhea that began 2 hours ago while attending a large family reunion potluck dinner. You question the patient specifically about foods they ingested containing a. beef b. meat and milk c. poultry and eggs d. home-preserved vegetables

B

Teaching in relation to home management following a laparoscopic cholecystectomy should include: a. keeping the bandages on the puncture sites for 48 hours b. reporting any bile-colored drainage or pus from the incision c. using over the counter antiemetics if nausea and vomiting occur d. emptying and measuring the contents of the bile bage from the T tube every day

B

The nurse has completed initial instruction with a patient regarding a weight loss program. The nurse determines that the teaching has been effective when the patient says A) a. "I plan to lose 4 lb a week until I have lost the 60-pound goal." B) b. "I will keep a diary of weekly weights to illustrate my weight loss." C) c. "I will restrict my carbohydrate intake to less than 30 g/day to maximize weight loss." D) d. "I should not exercise more than my program requires since increased activity increases the appetite."

B

The nurse is caring for a 45-year-old woman with a herniated lumbar disk and obesity. The patient realizes that weight loss is necessary to lessen back strain. The patient is 5'6" tall and weighs 186 lb (84.5 kg) with a body mass index (BMI) of 28 kg/m2. The nurse explains to the patient that this measurement places her in which of the following weight categories? A) a. Normal weight B) b. Overweight C) c. Obese D) d. Morbidly obese

B

The nurse is caring for a patient treated with intravenous fluid therapy for severe vomiting. As the patient recovers and begins to tolerate oral intake, the nurse understands that which of the following food choices would be most appropriate? A) a. Ice tea B) b. Dry toast C) c. Warm broth D) d. Plain hamburger

B

The nurse is planning care for a 68-year-old patient with an abdominal mass and suspected bowel obstruction. Which of the following factors in the patient's history increases the patient's risk for colorectal cancer? A) a. Osteoarthritis B) b. History of rectal polyps C) c. History of lactose intolerance D) d. Use of herbs as dietary supplements

B

The nurse should administer a prn dose of magnesium hydroxide (MOM) after noting which of the following while reviewing a patient's medical record? A) a. Abdominal pain and bloating B) b. No bowel movement for 3 days C) c. A decrease in appetite by 50% over 24 hours D) d. Muscle tremors and other signs of hypomagnesemia

B

The nurse would increase the comfort of the patient with appendicitis by: a. having the patient lie prone b. flexing the patients right knee c. sitting the patient upright in a chair d. turning the patient onto his or her left side

B

The patient who is admitted with a diagnosis of diverticulitis and a history of irritable bowel disease and gastroesophageal reflux disease (GERD) has received a dose of Mylanta 30 ml PO. The nurse would evaluate its effectiveness by questioning the patient as to whether which of the following symptoms has been resolved? A) a. Diarrhea B) b. Heartburn C) c. Constipation D) d. Lower abdominal pain

B

The patient with advanced cirrhosis asks why his abdomen is so swollen. The nurse's response is based on the knowledge that: a. a lack of clotting factors promotes the collection of blood in the abdominal cavity b. portal hypertension and hypoalbuminemia cause a fluid shift into the peritoneal space c. decreased peristalsis in the GI tract contributes to gas formation and distention of the bowel d. bile salts in the blood irritate the peritoneal membranes, causing edema and pocketing of fluid

B

The results of a patient's recent endoscopy indicate the presence of peptic ulcer disease (PUD). Which of the following teaching points should the nurse provide to the patient in light of his new diagnosis? A) a. "You'll need to drink at least two to three glasses of milk daily." B) b. "It would likely be beneficial for you to eliminate drinking alcohol." C) c. "Many people find that a minced or pureed diet eases their symptoms of PUD." D) d. "Your medications should allow you to maintain your present diet while minimizing symptoms."

B

Which of the following assessments should the nurse prioritize in the care of a patient who has recently begun receiving parenteral nutrition (PN)? A) a. Skin integrity and bowel sounds B) b. Electrolyte levels and daily weights C) c. Auscultation of the chest and tests of blood coagulability D) d. Peripheral vascular assessment and level of consciousness (LOC)

B

Which of the following would be the highest priority information to include in preoperative teaching for a 68-year-old patient scheduled for a colectomy? A) a. How to care for the wound B) b. How to deep breathe and cough C) c. The location and care of drains after surgery D) d. What medications will be used during surgery

B

18. A patient is being evaluated at the clinic for possible metabolic syndrome. The nurse will instruct the patient about the purpose of the a. resting electrocardiogram. b. fasting blood glucose test. c. postural blood pressures. d. cardiac enzyme tests.

B Rationale: A fasting blood glucose test >100 mg/dl is one of the diagnostic criteria for metabolic syndrome. The other tests are not used to diagnose metabolic syndrome, although they may be used to check for cardiovascular complications of the disorder. Cognitive Level: Application Text Reference: p. 987

6. When working with an obese patient who is enrolled in a behavior modification program, which nursing action is appropriate? a. Having the patient write down the caloric intake of each meal b. Asking the patient about situations that tend to increase appetite c. Suggesting that the patient have a reward, such as a piece of sugarless candy, after achieving a weight-loss goal d. Encouraging the patient to eat small amounts throughout the day rather than having scheduled meals

B Rationale: Behavior modification programs focus on how and when the person eats and de-emphasize aspects such as calorie counting. Nonfood rewards are recommended for achievement of weight-loss goals. Patients are often taught to restrict eating to designated meals when using behavior modification. Cognitive Level: Application Text Reference: p. 980 Nursing Process: Implementation NCLEX: Health Promotion and Maintenance

4. The nurse is developing a weight loss plan for a 21-year-old obese patient. Which statement by the nurse is most likely to help the patient in losing weight on the planned 800-calorie diet? a. "You will decrease your risk for future health problems such as diabetes by losing weight now." b. "You are likely to start to notice changes in how you feel with just a few weeks of diet and exercise." c. "Most of the weight that you lose during the first weeks of dieting is water weight rather than fat." d. "It will be necessary to change lifestyle habits permanently to maintain weight loss."

B Rationale: Motivation is a key factor in successful weight loss and a short-term outcome provides a higher motivation. A 21-year-old patient is developmentally unlikely to be motivated by future health problems. Telling a patient that the initial weight loss is water will be discouraging, although this may be correct. Changing lifestyle habits is necessary, but this process occurs over time and discussing this is not likely to motivate the patient. Cognitive Level: Application Text Reference: p. 979 Nursing Process: Implementation NCLEX: Psychosocial Integrity

115. In obtaining a history on a 74-year-old patient the nurse notes that he drinks alcohol daily and that he has noticed a tremor in his hands that affects his ability to hold things. With this information, what should the nurse's response be? A) "Does your family know you are drinking every day?" B) "Does the tremor change when you drink the alcohol?" C) "We'll do some tests to see what is causing the tremor." D) "You really shouldn't drink so much alcohol; it may be causing your tremor."

B) "Does the tremor change when you drink the alcohol?" Page: 632. Intention tremor/ senile tremor is relieved by alcohol, although this is not a recommended treatment. The nurse should assess whether the person is abusing alcohol in an effort to relieve the tremor.

31. During a mental status assessment, which question by the nurse would best assess a person's judgment? A) "Do you feel that you are being watched, followed, or controlled?" B) "Tell me about what you plan to do once you are discharged from the hospital." C) "What does the statement, 'People in glass houses shouldn't throw stones,' mean to you?" D) "What would you do if you found a stamped, addressed envelope lying on the sidewalk?"

B) "Tell me about what you plan to do once you are discharged from the hospital." Pages: 76-77. A person exercises judgment when he or she can compare and evaluate the alternatives in a situation and reach an appropriate course of action. Rather than testing the person's response to a hypothetical situation (as illustrated in the option with the envelope), the nurse should be more interested in the person's judgment about daily or long-term goals, the likelihood of acting in response to delusions or hallucinations and the capacity for violent or suicidal behavior.

108. During an abdominal assessment, the nurse is unable to hear bowel sounds in a patient's abdomen. Before reporting this finding as "silent bowel sounds" the nurse should listen for at least: A) 1 minute. B) 5 minutes. C) 10 minutes. D) 2 minutes in each quadrant.

B) 5 minutes. Pages: 539-540. Absent bowel sounds are rare. The nurse must listen for 5 minutes before deciding bowel sounds are completely absent.

107. During an abdominal assessment, the nurse would consider which of these findings as normal? A) The presence of a bruit in the femoral area B) A tympanic percussion note in the umbilical region C) A palpable spleen between the ninth and eleventh ribs in the left midaxillary line D) A dull percussion note in the left upper quadrant at the midclavicular line

B) A tympanic percussion note in the umbilical region Pages: 539-540. Tympany should predominate in all four quadrants of the abdomen because air in the intestines rises to the surface when the person is supine. Vascular bruits are not usually present. Normally the spleen is not palpable. Dullness would not be found in the area of lung resonance (left upper quadrant at the midclavicular line).

113. The nurse is assessing the joints of a woman who has stated, "I have a long family history of arthritis, and my joints hurt." The nurse suspects that she has osteoarthritis. Which of these are symptoms of osteoarthritis? Select all that apply. A) Symmetric joint involvement B) Asymmetric joint involvement C) Pain with motion of affected joints D) Affected joints are swollen with hard, bony protuberances E) Affected joints may have heat, redness, and swelling

B) Asymmetric joint involvement C) Pain with motion of affected joints D) Affected joints are swollen with hard, bony protuberances Page: 608. In osteoarthritis, asymmetric joint involvement commonly affects hands, knees, hips, and lumbar and cervical segments of the spine. Affected joints have stiffness, swelling with hard bony protuberances, pain with motion, and limitation of motion. The other options reflect signs of rheumatoid arthritis.

40. The nurse is examining an infant and prepares to elicit the Moro reflex at which time during the examination? A) When the infant is sleeping B) At the end of the examination C) Before auscultation of the thorax D) Halfway through the examination

B) At the end of the examination Page: 123. Elicit the Moro or "startle" reflex at the end of the examination because it may cause the infant to cry.

63. During an assessment of an infant, the nurse notes that the fontanels are depressed and sunken. The nurse suspects which condition? A) Rickets B) Dehydration C) Mental retardation D) Increased intracranial pressure

B) Dehydration Pages: 265-266. Depressed and sunken fontanels occur with dehydration or malnutrition. Mental retardation and rickets have no effect on fontanels. Increased intracranial pressure would cause tense or bulging, and possibly pulsating fontanels.

53. An elderly woman is brought to the emergency department after being found lying on the kitchen floor 2 days, and she is extremely dehydrated. What would the nurse expect to see upon examination? A) Smooth mucous membranes and lips B) Dry mucous membranes and cracked lips C) Pale mucous membranes D) White patches on the mucous membranes

B) Dry mucous membranes and cracked lips Page: 215. With dehydration, mucous membranes look dry and lips look parched and cracked. The other responses are not found in dehydration.

The nurse determines that a patient has experienced the beneficial effects of medication therapy with famotidine (Pepcid) when which of the following symptoms is relieved? 1. Ice tea 2. Dry toast 3. warm broth 4. plain hamburger

B) Dry toast (Dry toast or crackers may alleviate the feeling of nausea and prevent further vomiting. Extremely hot or cold liquids and fatty foods are generally not well tolerated

49. The nurse needs to perform anthropometric measures of an 80-year-old man who is confined to a wheelchair. Which of the following is true in this situation? A) Changes in fat distribution will affect the waist-to-hip ratio. B) Height measurements may not be accurate because of changes in bone. C) Declining muscle mass will affect the triceps skinfold measure. D) Mid-arm circumference is difficult to obtain because of loss of skin elasticity.

B) Height measurements may not be accurate because of changes in bone. Page: 191. Height measures may not be accurate in individuals confined to a bed or wheelchair or those over 60 years of age because of osteoporotic changes.

123. During an assessment of a 22-year-old woman who has a head injury from a car accident 4 hours ago, the nurse notices the following change: pupils were equal, but now the right pupil is fully dilated and nonreactive, left pupil is 4 mm and reacts to light. What does finding this suggest? A) Injury to the right eye B) Increased intracranial pressure C) Test was not performed accurately D) Normal response after a head injury

B) Increased intracranial pressure Pages: 662-663. In a brain-injured person, a sudden, unilateral, dilated, and nonreactive pupil is ominous. Cranial nerve III runs parallel to the brainstem. When increasing intracranial pressure pushes the brainstem down (uncal herniation), it puts pressure on cranial nerve III, causing pupil dilation. The other responses are incorrect.

29. The nurse is preparing to do a mental status examination. Which statement is true regarding the mental status examination? A) A patient's family is the best resource for information about the patient's coping skills. B) It is usually sufficient to gather mental status information during the health history interview. C) It takes an enormous amount of extra time to integrate the mental status examination into the health history interview. D) It is usually necessary to perform a complete mental status examination to get a good idea of the patient's level of functioning.

B) It is usually sufficient to gather mental status information during the health history interview. Page: 73. The full mental status examination is a systematic check of emotional and cognitive functioning. The steps described here, though, rarely need to be taken in their entirety. Usually, one can assess mental status through the context of the health history interview.

16. The nurse is reviewing concepts of cultural aspects of pain. Which statement is true regarding pain? A) All patients will behave the same way when in pain. B) Just as patients vary in their perceptions of pain, so will they vary in their expressions of pain. C) Cultural norms have very little to do with pain tolerance, because pain tolerance is always biologically determined. D) A patient's expression of pain is largely dependent on the amount of tissue injury associated with the pain.

B) Just as patients vary in their perceptions of pain, so will they vary in their expressions of pain. Page: 25 In addition to expecting variations in pain perception and tolerance, the nurse should expect variations in the expression of pain. It is well known that individuals turn to their social environment for validation and comparison. The other statements are incorrect.

93. The nurse is preparing to auscultate for heart sounds. Which technique is correct? A) Listen to the sounds at the aortic, tricuspid, pulmonic, and mitral areas. B) Listen by inching the stethoscope in a rough Z pattern, from the base of the heart across and down, then over to the apex. C) Listen to the sounds only at the site where the apical pulse is felt to be the strongest. D) Listen for all possible sounds at a time at each specified area.

B) Listen by inching the stethoscope in a rough Z pattern, from the base of the heart across and down, then over to the apex. Pages: 475-476. Do not limit auscultation of breath sounds to only four locations. Sounds produced by the valves may be heard all over the precordium. Inch the stethoscope in a rough Z pattern from the base of the heart across and down, then over to the apex. Or, start at the apex and work your way up. See Figure 19-22. Listen selectively to one sound at a time.

47. The nurse is reviewing the nutritional assessment of an 82-year-old patient. Which of these factors is most likely to affect the nutritional status of an elderly person? A) Increase in taste and smell B) Living alone on a fixed income C) Change in cardiovascular status D) Increase in gastrointestinal motility and absorption

B) Living alone on a fixed income Page: 176. Socioeconomic conditions frequently affect the nutritional status of the aging adult; these factors should be closely evaluated. Physical limitations, income, and social isolation are frequent problems that interfere with the acquisition of a balanced diet. A decrease in taste and smell and decreased gastrointestinal motility and absorption occur with aging. Cardiovascular status is not a factor that affects an elderly person's nutritional status.

116. During the neurologic assessment of a "healthy" 35-year-old patient, the nurse asks him to relax his muscles completely. The nurse then moves each extremity through full range of motion. Which of these results would the nurse expect to find? A) Firm, rigid resistance to movement B) Mild, even resistance to movement C) Hypotonic muscles as a result of total relaxation D) Slight pain with some directions of movement

B) Mild, even resistance to movement Page: 637. Tone is the normal degree of tension (contraction) in voluntarily relaxed muscles. It shows a mild resistance to passive stretch. Normally, the nurse will notice a mild, even resistance to movement. The other responses are not correct.

100. The nurse is reviewing an assessment of a patient's peripheral pulses and notices that the documentation states that the radial pulses are "2+." The nurse recognizes that this reading indicates what type of pulse? A) Bounding B) Normal C) Weak D) Absent

B) Normal Pages: 506-507. When documenting the force, or amplitude, of pulses, 3+ indicates an increased, full, or bounding pulse, 2+ indicates a normal pulse, 1+ indicates a weak pulse, and 0 indicates an absent pulse.

36. The nurse would use bimanual palpation technique in which situation? A) Palpating the thorax of an infant B) Palpating the kidneys and uterus C) Assessing pulsations and vibrations D) Assessing the presence of tenderness and pain

B) Palpating the kidneys and uterus Pages: 115-116. Bimanual palpation requires the use of both hands to envelop or capture certain body parts or organs such as the kidneys, uterus, or adnexa. The other situations are not appropriate for bimanual palpation.

98. The nurse is attempting to assess the femoral pulse in an obese patient. Which of these actions would be most appropriate? A) Have the patient assume a prone position. B) Ask the patient to bend his or her knees to the side in a froglike position. C) Press firmly against the bone with the patient in a semi-Fowler position. D) Listen with a stethoscope for pulsations because it is very difficult to palpate the pulse in an obese person.

B) Regular "lub, dub" pattern Pages: 510-511. To help expose the femoral area, particularly in obese people, the nurse should ask the person to bend his or her knees to the side in a froglike position.

111. The nurse suspects that a patient has appendicitis. Which of these procedures are appropriate for use when assessing for appendicitis or a perforated appendix? Select all that apply. A) Test for Murphy's sign. B) Test for Blumberg's sign. C) Test for shifting dullness. D) Perform iliopsoas muscle test. E) Test for fluid wave.

B) Test for Blumberg's sign. D) Perform iliopsoas muscle test. Pages: 543-544 | Page: 551. Testing for Blumberg's sign (rebound tenderness) and performing the iliopsoas muscle test should be used to assess for appendicitis. Murphy's sign is used to assess for an inflamed gallbladder or cholecystitis. Testing for a fluid wave and shifting dullness is done to assess for ascites.

109. A patient is suspected of having inflammation of the gallbladder, or cholecystitis. The nurse should conduct which of these techniques to assess for this condition? A) Obturator test B) Test for Murphy's sign C) Assess for rebound tenderness D) Iliopsoas muscle test

B) Test for Murphy's sign Page: 551. Normally, palpating the liver causes no pain. In a person with inflammation of the gallbladder, or cholecystitis, pain occurs as the descending liver pushes the inflamed gallbladder onto the examining hand during inspiration (Murphy's test). The person feels sharp pain and abruptly stops inspiration midway.

71. During an examination, a patient states that she was diagnosed with open-angle glaucoma 2 years ago. The nurse assesses for characteristics of open-angle glaucoma. Which of these are characteristics of open-angle glaucoma? Select all that apply. A) The patient may experience sensitivity to light, nausea, and halos around lights. B) The patient experiences tunnel vision in late stages. C) Immediate treatment is needed. D) Vision loss begins with peripheral vision. E) It causes sudden attacks of increased pressure that cause blurred vision. F) There are virtually no symptoms.

B) The patient experiences tunnel vision in late stages. D) Vision loss begins with peripheral vision. F) There are virtually no symptoms. Pages: 308-309. Open-angle glaucoma is the most common type of glaucoma; there are virtually no symptoms. Vision loss begins with the peripheral vision, which often goes unnoticed because individuals learn to compensate intuitively by turning their heads. The other characteristics are those of closed-angle glaucoma.

75. The nurse is testing the hearing of a 78-year-old man and keeps in mind the changes in hearing that occur with aging include which of the following? Select all that apply. A) Hearing loss related to aging begins in the mid 40s. B) The progression is slow. C) The aging person has low-frequency tone loss. D) The aging person may find it harder to hear consonants than vowels. E) Sounds may be garbled and difficult to localize. F) Hearing loss reflects nerve degeneration of the middle ear.

B) The progression is slow. D) The aging person may find it harder to hear consonants than vowels. E) Sounds may be garbled and difficult to localize. Page: 326. Presbycusis is a type of hearing loss that occurs with aging and is found in 60% of those older than 65 years. It is a gradual sensorineural loss caused by nerve degeneration in the inner ear or auditory nerve, and it slowly progresses after age 50. The person first notices a high-frequency tone loss; it is harder to hear consonants (high-pitched components of speech) than vowels. This makes words sound garbled. The ability to localize sound is impaired also.

8. The nurse is conducting an interview. Which of these statements is true regarding open-ended questions? Select all that apply. A) They elicit cold facts. B) They allow for self-expression. C) They build and enhance rapport. D) They leave interactions neutral. E) They call for short one- to two-word answers. F) They are used when narrative information is needed.

B) They allow for self-expression. C) They build and enhance rapport. F) They are used when narrative information Page: 32 Open-ended questions allow for self-expression, build rapport, and obtain narrative information. These features enhance communication during an interview. The other statements are appropriate for closed or direct questions.

66. The nurse is performing an eye assessment on an 80-year-old patient. Which of these findings is considered abnormal? A) A decrease in tear production B) Unequal pupillary constriction in response to light C) The presence of arcus senilis seen around the cornea D) Loss of the outer hair on the eyebrows due to a decrease in hair follicles

B) Unequal pupillary constriction in response to light Pages: 305-308. Pupils are small in old age, and the pupillary light reflex may be slowed, but pupillary constriction should be symmetric. The assessment findings in the other responses are considered normal in older persons.

39. When performing a physical examination, safety must be considered to protect the examiner and the patient against the spread of infection. Which of these statements describes the most appropriate action the nurse should take when performing a physical examination? A) There is no need to wash one's hands after removing gloves, as long as the gloves are still intact. B) Wash hands before and after every physical patient encounter. C) Wash hands between the examination of each body system to prevent the spread of bacteria from one part of the body to another. D) Wear gloves throughout the entire examination to demonstrate to the patient concern regarding the spread of infectious diseases.

B) Wash hands before and after every physical patient encounter. Page: 120. The nurse should wash his or her hands before and after every physical patient encounter; after contact with blood, body fluids, secretions, and excretions; after contact with any equipment contaminated with body fluids; and after removing gloves. Hands should be washed after gloves have been removed, even if the gloves appear to be intact. Gloves should be worn when there is potential contact with any body fluids.

91. During an assessment of a 68-year-old man with a recent onset of right-sided weakness, the nurse hears a blowing, swishing sound with the bell of the stethoscope over the left carotid artery. This finding would indicate: A) a valvular disorder. B) blood flow turbulence. C) fluid volume overload. D) ventricular hypertrophy.

B) blood flow turbulence. Page: 471. A bruit is a blowing, swishing sound indicating blood flow turbulence; normally none is present.

51. The nurse keeps in mind that a thorough skin assessment is very important because the skin holds information about a person's: A) support systems. B) circulatory status. C) socioeconomic status. D) psychological wellness.

B) circulatory status. Page: 211. The skin holds information about the body's circulation, nutritional status, and signs of systemic diseases as well as topical data on the integument itself.

11. In the majority culture of America, coughing, sweating, and diarrhea are symptoms of an illness. For some individuals of Mexican-American origin, however, these symptoms are a normal part of living. The nurse recognizes that this is true, probably because Mexican-Americans: A) have less efficient immune systems and are often ill. B) consider these symptoms a part of normal living, not symptoms of ill health. C) come from Mexico and coughing is normal and healthy there. D) are usually in a lower socioeconomic group and are more likely to be sick.

B) consider these symptoms a part of normal living, not symptoms of ill health. Page: 27 The nurse needs to identify the meaning of health to the patient, remembering that concepts are derived, in part, from the way in which members of the cultural group define health.

88. During palpation of the anterior chest wall, the nurse notices a coarse, crackling sensation over the skin surface. On the basis of these findings, the nurse suspects: A) tactile fremitus. B) crepitus. C) friction rub. D) adventitious sounds.

B) crepitus. Page: 424. Crepitus is a coarse, crackling sensation palpable over the skin surface. It occurs in subcutaneous emphysema when air escapes from the lung and enters the subcutaneous tissue, as after open thoracic injury or surgery.

34. The nurse is assessing a patient's skin during an office visit. What is the best technique to use to best assess the patient's skin temperature? Use the: A) fingertips because they're more sensitive to small changes in temperature. B) dorsal surface of the hand because the skin is thinner than on the palms. C) ulnar portion of the hand because there is increased blood supply that enhances temperature sensitivity. D) palmar surface of the hand because it is most sensitive to temperature variations because of increased nerve supply in this area.

B) dorsal surface of the hand because the skin is thinner than on the palms. The dorsa (backs) of hands and fingers are best for determining temperature because the skin there is thinner than on the palms. Fingertips are best for fine, tactile discrimination; the other responses are not useful for palpation.

96. During an assessment, the nurse uses the "profile sign" to detect: A) pitting edema. B) early clubbing. C) symmetry of the fingers. D) insufficient capillary refill.

B) early clubbing. Page: 506. The nurse should use the profile sign (viewing the finger from the side) to detect early clubbing.

33. When performing a physical assessment, the technique the nurse will always use first is: A) palpation. B) inspection. C) percussion. D) auscultation.

B) inspection. Pages: 115-116. The skills requisite for the physical examination are inspection, palpation, percussion, and auscultation. The skills are performed one at a time and in this order (with the exception of the abdominal assessment, where auscultation takes place before palpation and percussion). The assessment of each body system begins with inspection. A focused inspection takes time and yields a surprising amount of information.

105. A patient has hypoactive bowel sounds. The nurse knows that a potential cause of hypoactive bowel sounds is: A) diarrhea. B) peritonitis. C) laxative use. D) gastroenteritis.

B) peritonitis. Page: 561. Diminished or absent bowel sounds signal decreased motility from inflammation as seen with peritonitis, with paralytic ileus after abdominal surgery, or with late bowel obstruction.

62. The physician reports that a patient with a neck tumor has a tracheal shift. The nurse is aware that this means that the patient's trachea is: A) pulled to the affected side. B) pushed to the unaffected side. C) pulled downward. D) pulled downward in a rhythmic pattern.

B) pushed to the unaffected side. Pages: 262-263. The trachea is pushed to the unaffected side with an aortic aneurysm, a tumor, unilateral thyroid lobe enlargement, and pneumothorax. The trachea is pulled to the affected side with large atelectasis, pleural adhesions, or fibrosis. Tracheal tug is a rhythmic downward pull that is synchronous with systole and occurs with aortic arch aneurysm.

56. A 40-year-old woman reports a change in mole size, accompanied by color changes, itching, burning, and bleeding over the past month. She has a dark complexion and has no family history of skin cancer, but she has had many blistering sunburns in the past. The nurse would: A) tell the patient to watch the lesion and report back in 2 months. B) refer the patient because of the suspicion of melanoma on the basis of her symptoms. C) ask additional questions regarding environmental irritants that may have caused this condition. D) suspect that this is a compound nevus, which is very common in young to middle-aged adults.

B) refer the patient because of the suspicion of melanoma on the basis of her symptoms. The ABCD danger signs of melanoma are asymmetry, border irregularity, color variation, and diameter. In addition, individuals may report a change in size, development of itching, burning, bleeding, or a new-pigmented lesion. Any of these signs raise suspicion of malignant melanoma and warrant immediate referral.

68. A patient comes to the emergency department after a boxing match, and his left eye is swollen almost shut. He has bruises on his face and neck. He says he is worried because he "can't see well" from his left eye. The physician suspects retinal damage. The nurse recognizes that signs of retinal detachment include: A) loss of central vision. B) shadow or diminished vision in one quadrant or one half of the visual field. C) loss of peripheral vision. D) sudden loss of pupillary constriction and accommodation.

B) shadow or diminished vision in one quadrant or one half of the visual field. Page: 316. With retinal detachment, the person has shadows or diminished vision in one quadrant or one half of the visual field. The other responses are not signs of retinal detachment.

13. An individual who takes the magicoreligious perspective of illness and disease is likely to believe that his or her illness was caused by: A) germs and viruses. B) supernatural forces. C) eating imbalanced foods. D) an imbalance within his or her spiritual nature.

B) supernatural forces. Page: 21 The basic premise of the magicoreligious perspective is that the world is seen as an arena in which supernatural forces dominate. The fate of the world and those in it depends on the actions of supernatural forces for good or evil. The other answers do not reflect the magicoreligious perspective.

The teaching plan for the patient being discharged following an acute episode of upper GI bleeding will include information concerning the importance of (select all that apply) a. only taking aspirin with milk or bread products b. avoiding taking aspirin and drugs containing aspirin c. taking only drugs prescribed by the health care provider d. taking all drugs 1 hour before mealtime to prevent further bleeding e. reading all OTC drug labels to avoid those containing stearic acid and calcium

B,C

The nurse would recognize that the liver performs which of the following functions (select all that apply)? A) a. Bile storage B) b. Detoxification C) c. Red blood cell (RBC) destruction D) d. Protein metabolism E) e. Steroid metabolism

B,C,D,E

25. The nurse is performing a functional assessment on an 82-year-old patient who recently had a stroke. Which of these questions would be most important to ask? A. "Do you wear glasses?" B. "Are you able to dress yourself?" C. "Do you have any thyroid problems?" D. "How many times a day do you have a bowel movement?"

B. "Are you able to dress yourself?" Page: 67. Functional assessment measures how a person manages day-to-day activities. For the older person, the meaning of health becomes those activities that they can or cannot do. The other responses do not relate to functional assessment.

26. The nurse is conducting a developmental history on a 5-year-old child. Which questions are appropriate to ask the parents for this part of the assessment? Select all that apply. A. "How much junk food does your child eat?" B. "How many teeth has he lost, and when did he lose them?" C. "Is he able to tie his shoelaces?" D. "Does he take a children's vitamin?" E. "Can he tell time?" F. "Does he have any food allergies?"

B. "How many teeth has he lost, and when did he lose them?" C. "Is he able to tie his shoelaces?" E. "Can he tell time?" Page: 61. Questions about tooth loss, ability to tell time, and ability to tie shoelaces are appropriate questions for a developmental assessment. Questions about junk food intake and vitamins are part of a nutritional history. Questions about food allergies are not part of a developmental history.

20. When the nurse is evaluating the reliability of a patient's responses, which of these statements would be correct? The patient: A. has a history of drug abuse and therefore is not reliable. B. provided consistent information and therefore is reliable. C. smiled throughout interview and therefore is assumed reliable. D. would not answer questions concerning stress and therefore is not reliable.

B. provided consistent information and therefore is reliable. Page: 50. A reliable person always gives the same answers, even when questions are rephrased or are repeated later in the interview. The other statements are not correct.

Mucus and bicarbonate secretion?

Bicarb is secreted by epithelia cells and mixes with mucus to form pH barrier to the back diffusion of acid. Creates the GASTRIC MUCOSAL BARRIER

Pyloric obstruction Symptoms

Bloating, N/V, may become hypochloremic, hypokalemia

Blood flow of Lv 2

Blood from central V. dumps into hepatic vv

Blood flow of Lv 1

Blood from hepatic A & V travel through sinusoids to central V.

Blood flow of Lv 3

Blood from hepatic vv goes to Inf vena cava

Esophageal Trauma Causes

Blunt injuries, Chemical Burns, Surgery, Endoscopy, Stess, Severe Vomiting

A 54-year-old patient admitted with diabetes mellitus, malnutrition, osteomyelitis, and alcohol abuse has a serum amylase level of 280 U/L and a serum lipase level of 310 U/L. To which of the following diagnoses does the nurse attribute these findings? A) a. Malnutrition B) b. Osteomyelitis C) c. Alcohol abuse D) d. Diabetes mellitus

C

A patient is scheduled to receive "Colace 100 mg PO." The patient asks to take the medication in liquid form, and the nurse obtains an order for the interchange. Available is a syrup that contains 150 mg/15 ml. How many milliliters does the nurse administer? A) a. 3 B) b. 5 C) c. 10 D) d. 12

C

A patient who has undergone an esophagectomy for esophageal cancer develops increasing pain, fever, and dyspnea when a full liquid diet is started postoperatively. The nurse recognizes that these symptoms are most indicative of: a. an intolerance to the feedings b. extension of the tumor into the aorta c. leakage of fluid or foods into the mediastinum d. esophageal perforation with fistula formation into the lung

C

A patient with metastatic colorectal cancer is scheduled for both chemotherapy and radiation therapy. Patient teaching regarding these therapies for this patient would include an explanation that: a. chemotherapy can be used to cure colorectal cancer b. radiation is routinely used as adjuvant therapy following surgery c. both chemotherapy and radiation can be used as palliative therapy d. the patient should expect few if any side effects from chemotherapeutic agencts

C

A patient with pancreatic cancer is admitted to the hospital for evaluation of possible treatment options. The patient asks the nurse to explain the Whipple procedure that the surgeon has described. The explanation includes the information that a Whipple procedure involves: a. creating a bypass around the obstruction caused by the tumor by joining the gallbladder to the jejunum b. resection of the entire pancreas and the distal portion of the stomach, with anastomosis of the common bile duct and stomach into the duodenum c. removal of part of the pancreas, part of the stomach, the duodenum, and the gallbladder, with joining of the pancreatic duct, common bile duct, and stomach into the jejunum d.radical removal of the pancreas, duodenum, and spleen, and attaching the stomach to the jejunum, which requires oral supplementation of pancreatic digestive enzymes and insulin replacement therapy

C

Following administration of a dose of metoclopramide (Reglan) to the patient, the nurse determines that the medication has been effective when which of the following is noted? A) a. Decreased blood pressure B) b. Absence of muscle tremors C) c. Relief of nausea and vomiting D) d. No further episodes of diarrhea

C

Following bowel resection, a patient has a nasogastric tube to suction, but complains of nausea and abdominal distention. The nurse irrigates the tube prn as ordered, but the irrigating fluid does not return. Which of the following should be the priority action by the nurse? A) a. Notify the physician. B) b. Auscultate for bowel sounds. C) c. Reposition the tube and check for placement. D) d. Remove the tube and replace it with a new one.

C

In developing a weight reduction program with a 45-year-old patient who weighs 186 lb, the nurse encourages the patient to set a weight loss goal of how many pounds in 4 weeks? A) a. 1-2 B) b. 3-5 C) c. 4-8 D) d. 5-10

C

In developing an effective weight reduction plan for an overweight patient who states a willingness to try to lose weight, it is most important for the nurse to first assess which of the following factors? A) a. The length of time the patient has been obese B) b. The patient's current level of physical activity C) c. The patient's social, emotional, and behavioral influences on obesity D) d. Anthropometric measurements, such as body mass index and skinfold thickness

C

In planning care for the patient with Crohn's disease, the nurse recognizes that a major difference between ulcerative colitis and Crohn's disease is that Crohn's disease is: a. frequently results in toxic megacolon b. causes fewer nutritional deficiencies than does ulcerative colitis c. often occurs after surgery, whereas ulcerative colitis is curable with a colectomy d. is manifested by rectal bleeding and anemia more frequently than is ulcerative colitis

C

The health care team is assessing a patient for acute pancreatitis after he presented to the emergency department with severe abdominal pain. Which of the following laboratory values are the best diagnostic indicators of acute pancreatitis? A) a. Gastric pH B) b. Blood glucose C) c. Serum amylase D) d. Serum potassium

C

The nurse determines that a patient has experienced the beneficial effects of medication therapy with famotidine (Pepcid) when which of the following symptoms is relieved? A) a. Nausea B) b. Belching C) c. Epigastric pain D) d. Difficulty swallowing

C

The nurse is caring for a patient admitted to the hospital for asthma who weighs 186 lb (84.5 kg). During dietary counseling, the patient asks the nurse how much protein he should ingest each day. How many grams of protein does the nurse recommend should be included in the diet based on the patient's current weight? A) a. 24 B) b. 41 C) c. 68 D) d. 93

C

The nurse is involved in health promotion related to oral cancer. Teaching young adults about behaviors that put them at risk for oral cancer includes: a. discouraging use of chewing gum b. avoiding use of perfumed lip gloss c. avoiding use of smokeless tobacco d. discouraging drinking of carbonated beverages

C

The nurse is providing care for a 23-year-old woman who is a strict vegetarian. To prevent the consequences of iron deficiency, the nurse should recommend A) a. Brown rice and kidney beans. B) b. Cauliflower and egg substitutes. C) c. Soybeans and hot breakfast cereal. D) d. Whole-grain bread and citrus fruits.

C

The nurse recognizes that the majority of patients' caloric needs should come from which of the following sources? A) a. Fats B) b. Proteins C) c. Polysaccharides D) d. Monosaccharides

C

The patient receiving chemotherapy rings the call bell and reports an onset of nausea. The nurse should prepare a prn dose of which of the following medications? A) a. Morphine sulfate B) b. Zolpidem (Ambien) C) c. Ondansetron (Zofran) D) d. Dexamethasone (Decadron)

C

The pernicious anemia that may accompany gastritis is due to which of the following? a. chronic autoimmune destruction of cobalamin stores in the body b. progressive gastric atrophy from chronic breakage in the mucosal barrier and blood loss c. a lack of intrinsic factor normally produced by acid-secreting cells in the gastric mucosa d. hyperchlorhydria resulting from an increase in acid-secreting parietal cells and degradations of RBCs

C

When assessing the health perception-health maintenance pattern as related to GI function, an appropriate question by the nurse is: a. what is your usual bowel elimination pattern? b. what percentage of your income is spent on food? c. have you traveled to a foreign country in the last year? d. do you have diarrhea when you are under a lot of stress?

C

Which of the following statements best described the etiology of obesity: a. obesity primarily results from a genetic predisposition b. psychosocial factors can override the effects of genetics in the etiology of obesity c. Obesity is the result of complex interactions between genetic and environmental factors d. genetic factors are more important than environmental factors in the etiology of obesity

C

14. In planning preoperative teaching for a patient undergoing a Roux-en-Y gastric bypass as treatment for morbid obesity, the nurse places the highest priority on a. demonstrating passive range-of-motion exercises to the legs. b. teaching the patient about the postoperative presence of a NG tube connected to suction. c. teaching the patient proper coughing and deep-breathing techniques and methods of turning and positioning. d. discussing the necessary postoperative modifications in lifestyle.

C Rationale: Coughing, deep breathing, and turning can prevent major postoperative complications such as carbon monoxide retention, hypoxemia, and deep vein thrombosis. Passive range of motion is not as helpful in preventing deep vein thrombosis and other complications of immobility as active range of motion. The patient will be instructed about the NG tube, but this is not as important as pulmonary function. The focus during the preoperative teaching is the immediate recovery from the surgery, rather than the long-term lifestyle changes that are needed. Cognitive Level: Application Text Reference: p. 985 Nursing Process: Planning NCLEX: Physiological Integrity

13. An obese patient asks the nurse about using orlistat (Xenical) for weight reduction. The nurse advises the patient that a. this drug can cause serious depletion of fat-soluble vitamins and should be used for only several weeks. b. weight-reduction drugs of any type are used for only those who do not have the willpower to reduce their intake of food. c. drugs may be helpful in weight loss, but weight gain is likely to recur unless changes in diet and exercise are maintained. d. the long-term effect of orlistat is not known, and the drug may cause serious side effects such as heart valve problems.

C Rationale: Drugs can assist with weight loss, but should be just one part of a program that includes diet and exercise changes, or weight gain will occur when the drugs are stopped. Orlistat can decrease the absorption of fat-soluble vitamins, but vitamin supplements can be used to prevent deficiency. A reply indicating that the patient is lacking in willpower is judgmental and nontherapeutic. Orlistat may be used for weight maintenance as well as for weight loss and does not cause serious side effects. Cognitive Level: Application Text Reference: p. 981 Nursing Process: Implementation NCLEX: Physiological Integrity

7. After discussing appropriate exercise activities with an overweight patient who is starting to exercise as part of a weight-loss program, the nurse will determine that the instructions have been understood when the patient reports a. playing soccer for an hour on the weekend. b. running for 10 to 15 minutes 3 times/week. c. walking for 40 minutes 6 or 7 days a week. d. lifting weights with friends three times a week.

C Rationale: Exercise should be done daily for 30 minutes to an hour. Exercising in highly aerobic activities for short bursts or only once a week is not helpful and may be dangerous in an individual who has not been exercising. Running may be appropriate, but a patient should start with an exercise that is less stressful and can be done for a longer period. Weight-lifting is not as helpful as aerobic exercise in weight loss. Cognitive Level: Application Text Reference: p. 980 Nursing Process: Evaluation NCLEX: Health Promotion and Maintenance

2. When taking the health history from an obese patient, which information obtained by the nurse is most helpful in determining whether the patient will be successful in losing weight? a. The patient's body mass index is 39 kg/m2. b. The patient has a history of losing weight successfully in the past. c. The patient says, "I am ready to make some changes in my lifestyle." d. The patient's rides a stationary bicycle.

C Rationale: Motivation is essential for a successful outcome and will predict the chance for success more than the patient's initial BMI, previous history of weight loss, and activity level. Cognitive Level: Application Text Reference: p. 978 Nursing Process: Assessment NCLEX: Health Promotion and Maintenance

12. When developing a weight-reduction plan for an obese patient who is starting a weight-loss program, which question is most important for the nurse to ask? a. "What kind of physical activities do you enjoy?" b. "How long have you been overweight?" c. "What factors do you think led to your obesity?" d. "Have you been on any previous diets?"

C Rationale: The nurse should obtain information about the patient's perceptions of the reasons for the obesity to develop a plan individualized to the patient. The other information will also be obtained from the patient, but the patient is more likely to make changes when the patient's beliefs are considered in planning. Cognitive Level: Application Text Reference: p. 976 Nursing Process: Assessment NCLEX: Health Promotion and Maintenance

10. A moderately obese patient has been on a 1000-calorie diet with a daily exercise routine and a prescription for sibutramine (Meridia) for 10 weeks. Which information obtained by the nurse is important to report to the health care provider? a. The patient has not lost any weight for the last 2 weeks. b. The patient complains about having chronic constipation. c. The patient tells the nurse about occasional palpitations. d. The patient reports walking only 3 days during the last week.

C Rationale: The patient may be experiencing an increase in heart rate caused by the sibutramine (Meridia) that should be evaluated further by the health care provider. Plateaus during weight-loss programs are common. Chronic constipation may be a side effect of the sibutramine, and the nurse should instruct the patient in measures such as eating more high-fiber foods and increasing fluid intake. The nurse should reinforce the need to exercise more frequently, but no additional intervention by the health care provider is necessary regarding the patient's activity level. Cognitive Level: Application Text Reference: p. 981 Nursing Process: Evaluation NCLEX: Physiological Integrity

17. The nurse in the clinic is assessing a new patient who has abdominal obesity and hypertension. What further assessment should the nurse do to assess for possible metabolic syndrome? a. Take the patient's apical pulse. b. Ask the patient about dietary intake. c. Measure the patient's waist size. d. Determine the patient's ethnic origin.

C Rationale: Waist size greater than 40 inches in men or 35 inches in women is one of the diagnostic criteria for metabolic syndrome. The other criteria are: increased triglycerides, low high-density lipoprotein (HDL), hypertension, and increased fasting glucose. Dietary intake and ethnicity are risk factors for metabolic syndrome but will not help determine whether the patient has this disorder. Cardiovascular disease may occur in patients with metabolic syndrome, but the pulse rate will not assist in the diagnosis of metabolic syndrome. Cognitive Level: Application Text Reference: p. 987 Nursing Process: Assessment NCLEX: Health Promotion and Maintenance

15. A patient returns to the surgical nursing unit following a vertical banded gastroplasty with a nasogastric tube to low, intermittent suction and a PCA machine for pain control. During the postoperative care of the patient, the nurse recognizes the need to a. promote return of bowel sounds by discouraging excessive PCA use. b. maintain patency of the NG tube with frequent normal saline irrigations. c. support the surgical incision during coughing to prevent dehiscence of the wound. d. position the patient flat in bed on the right side to promote normal stomach emptying.

C Rationale: Wound dehiscence is a more common problem in obese patients postoperatively and the patient should cough and deep breathe every 2 hours. The patient should be encouraged to use the PCA, since pain control will improve cough effort and patient mobility. NG irrigation may damage the suture line and should only be done by the health care provider. The patient's head should be elevated to allow better respiratory effort. Cognitive Level: Application Text Reference: p. 986 Nursing Process: Implementation NCLEX: Physiological Integrity

65. A mother asks when her newborn infant's eyesight will be developed. The nurse should reply: A) "Vision is not totally developed until 2 years of age." B) "Infants develop the ability to focus on an object at around 8 months." C) "By about 3 months, infants develop more coordinated eye movements and can fixate on an object." D) "Most infants have uncoordinated eye movements for the first year of life."

C) "By about 3 months, infants develop more coordinated eye movements and can fixate on an object." Page: 284. Eye movements may be poorly coordinated at birth, but by 3 to 4 months of age, the infant should establish binocularity and should be able to fixate on a single image with both eyes simultaneously.

12. Among many Asians there is a belief in the yin/yang theory, rooted in the ancient Chinese philosophy of Tao. The nurse recognizes which statement that most accurately reflects "health" in an Asian with this belief? A) A person is able to work and produce. B) A person is happy, stable, and feels good. C) All aspects of the person are in perfect balance. D) A person is able to care for others and function socially.

C) All aspects of the person are in perfect balance. Page: 21 Many Asians believe in the yin/yang theory, in which health is believed to exist when all aspects of the person are in perfect balance. The other statements do not describe this theory.

78. A 65-year-old patient with a history of heart failure comes to the clinic with complaints of "being awakened from sleep with shortness of breath." Which action by the nurse is most appropriate? A) Obtain a detailed history of the patient's allergies and history of asthma. B) Tell the patient to sleep on his or her right side to facilitate ease of respirations. C) Assess for other signs and symptoms of paroxysmal nocturnal dyspnea. D) Assure the patient that this is normal and will probably resolve within the next week.

C) Assess for other signs and symptoms of paroxysmal nocturnal dyspnea. Pages: 419-420. The patient is experiencing paroxysmal nocturnal dyspnea: being awakened from sleep with shortness of breath and the need to be upright to achieve comfort.

60. During an examination, the nurse knows that Paget's disease would be indicated by which of these assessment findings? A) Positive Macewen sign B) Premature closure of the sagittal suture C) Headache, vertigo, tinnitus, and deafness D) Elongated head with heavy eyebrow ridge

C) Headache, vertigo, tinnitus, and deafness Paget's disease occurs more often in males and is characterized by bowed long bones, sudden fractures, and enlarging skull bones that press on cranial nerves causing symptoms of headache, vertigo, tinnitus, and progressive deafness.

94. The nurse is assessing a patient's apical impulse. Which of these statements is true regarding the apical impulse? A) It is palpable in all adults. B) It occurs with the onset of diastole. C) Its location may be indicative of heart size. D) It should normally be palpable in the anterior axillary line.

C) Its location may be indicative of heart size. Page: 473 | Page: 492. The apical impulse is palpable in about 50% of adults. It is located in the fifth left intercostal space in the midclavicular line. Horizontal or downward displacement of the apical impulse may indicate an enlargement of the left ventricle.

122. The nurse is caring for a patient who has just had neurosurgery. To assess for increased intracranial pressure, what would the nurse include in the assessment? A) Cranial nerves, motor function, and sensory function B) Deep tendon reflexes, vital signs, and coordinated movements C) Level of consciousness, motor function, pupillary response, and vital signs D) Mental status, deep tendon reflexes, sensory function, and pupillary response

C) Level of consciousness, motor function, pupillary response, and vital signs Pages: 660-661. Some hospitalized persons have head trauma or a neurologic deficit from a systemic disease process. These people must be monitored closely for any improvement or deterioration in neurologic status and for any indication of increasing intracranial pressure. The nurse should use an abbreviation of the neurologic examination in the following sequence: level of consciousness, motor function, pupillary response, and vital signs.

32. The nurse is performing a mental status examination. Which statement is true regarding the assessment of mental status? A) Mental status assessment diagnoses specific psychiatric disorders. B) Mental disorders occur in response to everyday life stressors. C) Mental status functioning is inferred through assessment of an individual's behaviors. D) Mental status can be assessed directly, just like other systems of the body (e.g., cardiac and breath sounds).

C) Mental status functioning is inferred through assessment of an individual's behaviors. Page: 71. Mental status functioning is inferred through assessment of an individual's behaviors. It cannot be assessed directly like characteristics of the skin or heart sounds.

27. During an examination, the nurse can assess mental status by which activity? A) Examining the patient's electroencephalogram B) Observing the patient as he or she performs an IQ test C) Observing the patient and inferring health or dysfunction D) Examining the patient's response to a specific set of questions

C) Observing the patient and inferring health or dysfunction Page: 71. Mental status cannot be scrutinized directly like the characteristics of skin or heart sounds. Its functioning is inferred through assessment of an individual's behaviors, such as consciousness, language, mood and affect, and other aspects.

95. During an assessment of an older adult, the nurse should expect to notice which finding as a normal physiologic change associated with the aging process? A) Hormonal changes causing vasodilation and a resulting drop in blood pressure B) Progressive atrophy of the intramuscular calf veins, causing venous insufficiency C) Peripheral blood vessels growing more rigid with age, producing a rise in systolic blood pressure D) Narrowing of the inferior vena cava, causing low blood flow and increases in venous pressure resulting in varicosities

C) Peripheral blood vessels growing more rigid with age, producing a rise in systolic blood pressure Pages: 504-505. Peripheral blood vessels grow more rigid with age, resulting in a rise in systolic blood pressure. Aging produces progressive enlargement of the intramuscular calf veins, not atrophy. The other options are not correct.

58. A patient has been admitted for severe psoriasis. The nurse can expect to see what finding in the patient's fingernails? A) Splinter hemorrhages B) Paronychia C) Pitting D) Beau lines

C) Pitting Pages: 248-250. Pitting nails are characterized by sharply defined pitting and crumbling of the nails with distal detachment, and they are associated with psoriasis. See Table 12-13 for descriptions of the other terms.

84. During auscultation of the lungs of an adult patient, the nurse notices the presence of bronchophony. The nurse should assess for signs of which condition? A) Airway obstruction B) Emphysema C) Pulmonary consolidation D) Asthma

C) Pulmonary consolidation Page: 446. Pathologic conditions that increase lung density, such as pulmonary consolidation, will enhance transmission of voice sounds, such as bronchophony. See Table 18-7.

55. The nurse has discovered decreased skin turgor in a patient and knows that this is an expected finding in which of these conditions? A) Severe obesity B) Childhood growth spurts C) Severe dehydration D) Connective tissue disorders such as scleroderma

C) Severe dehydration Page: 215. Decreased skin turgor is associated with severe dehydration or extreme weight loss.

99. When using a Doppler ultrasonic stethoscope, the nurse recognizes arterial flow when which sound is heard? A) Low humming sound B) Regular "lub, dub" pattern C) Swishing, whooshing sound D) Steady, even, flowing sound

C) Swishing, whooshing sound Pages: 515-516. When using the Doppler ultrasonic stethoscope, the pulse site is found when one hears a swishing, whooshing sound.

42. The nurse is assessing an 80-year-old male patient. Which assessment findings would be considered normal? A) An increase in body weight from younger years B) Additional deposits of fat on the thighs and lower legs C) The presence of kyphosis and flexion in the knees and hips D) A change in overall body proportion, a longer trunk, and shorter extremities

C) The presence of kyphosis and flexion in the knees and hips Page: 149. Changes that occur in the aging person include more prominent bony landmarks, decreased body weight (especially in males), a decrease in subcutaneous fat from the face and periphery, and additional fat deposited on the abdomen and hips. Postural changes of kyphosis and slight flexion in the knees and hips also occur.

48. When the mid-upper arm circumference and triceps skinfold of an 82-year-old man are evaluated, which is important for the nurse to remember? A) These measurements are no longer necessary for the elderly. B) Derived weight measures may be difficult to interpret because of wide ranges of normal. C) These measurements may not be accurate because of changes in skin and fat distribution. D) Measurements may be difficult to obtain if the patient is unable to flex his elbow to at least 90 degrees.

C) These measurements may not be accurate because of changes in skin and fat distribution. Page: 191 Accurate mid-upper arm circumference and triceps skinfold measurements are difficult to obtain and interpret in older adults because of sagging skin, changes in fat distribution, and declining muscle mass. Body mass index and waist-to-hip ratio are better indicators of obesity in the elderly.

110. During an assessment the nurse notices that a patient's umbilicus is enlarged and everted. It is midline, and there is no change in skin color. The nurse recognizes that the patient may have which condition? A) Intra-abdominal bleeding B) Constipation C) Umbilical hernia D) An abdominal tumor

C) Umbilical hernia Page: 537. The umbilicus is normally midline and inverted, with no signs of discoloration. With an umbilical hernia, the mass is enlarged and everted. The other responses are incorrect.

67. The nurse notices the presence of periorbital edema when performing an eye assessment on a 70-year-old patient. The nurse should: A) check for the presence of exophthalmos. B) suspect that the patient has hyperthyroidism. C) ask the patient if he or she has a history of heart failure. D) assess for blepharitis because this is often associated with periorbital edema.

C) ask the patient if he or she has a history of heart failure. Page: 312. Periorbital edema occurs with local infections, crying, and systemic conditions such as heart failure, renal failure, allergy, and hypothyroidism. Periorbital edema is not associated with blepharitis.

97. When performing a peripheral vascular assessment on a patient, the nurse is unable to palpate the ulnar pulses. The patient's skin is warm and capillary refill time is normal. The nurse should next: A) check for the presence of claudication. B) refer the individual for further evaluation. C) consider this a normal finding and proceed with the peripheral vascular evaluation. D) ask the patient if he or she has experienced any unusual cramping or tingling in the arm.

C) consider this a normal finding and proceed with the peripheral vascular evaluation. Pages: 506-507. It is not usually necessary to palpate the ulnar pulses. The ulnar pulses are often not palpable in the normal person. The other responses are not correct.

15. An elderly Mexican-American woman with traditional beliefs has been admitted to an inpatient care unit. A culturally-sensitive nurse would: A) contact the hospital administrator about the best course of action. B) automatically get a curandero for her because it is not culturally appropriate for her to request one. C) further assess the patient's cultural beliefs and offer the patient assistance in contacting a curandero or priest if she desires. D) ask the family what they would like to do because Mexican-Americans traditionally give control of decisions to their families.

C) further assess the patient's cultural beliefs and offer the patient assistance in contacting a curandero or priest if she desires. Pages: 22-23 In addition to seeking help from the biomedical/scientific health care provider, patients may also seek help from folk or religious healers. Some people, such as those of Mexican-American or American Indian origins, may believe that the cure is incomplete unless the body, mind, and spirit are also healed (although the division of the person into parts is a Western concept).

43. When assessing the force, or strength, of a pulse, the nurse recalls that it: A) is usually recorded on a 0- to 2-point scale. B) demonstrates elasticity of the vessel wall. C) is a reflection of the heart's stroke volume. D) reflects the blood volume in the arteries during diastole.

C) is a reflection of the heart's stroke volume. Page: 134. The heart pumps an amount of blood (the stroke volume) into the aorta. The force flares the arterial walls and generates a pressure wave, which is felt in the periphery as the pulse.

80. The nurse is reviewing the technique of palpating for tactile fremitus with a new graduate. Which statement by the graduate nurse reflects a correct understanding of tactile fremitus? "Tactile fremitus: A) is caused by moisture in the alveoli." B) indicates that there is air in the subcutaneous tissues." C) is caused by sounds generated from the larynx." D) reflects the blood flow through the pulmonary arteries."

C) is caused by sounds generated from the larynx." Pages: 422-423. Fremitus is a palpable vibration. Sounds generated from the larynx are transmitted through patent bronchi and the lung parenchyma to the chest wall where they are felt as vibrations. Crepitus is the term for air in the subcutaneous tissues.

14. If an American Indian has come to the clinic to seek help with regulating her diabetes, the nurse can expect that she: A) will comply with the treatment prescribed. B) has obviously given up her beliefs in naturalistic causes of disease. C) may also be seeking the assistance of a shaman or medicine man. D) will need extra help in dealing with her illness and may be experiencing a crisis of faith.

C) may also be seeking the assistance of a shaman or medicine man. Page: 23 When self-treatment is unsuccessful, the individual may turn to the lay or folk healing systems, to spiritual or religious healing, or to scientific biomedicine. In addition to seeking help from a biomedical or scientific health care provider, patients may also seek help from folk or religious healers.

28. The nurse is assessing a 75-year-old man. As the nurse begins the mental status portion of the assessment, the nurse expects that this patient: A) will have no decrease in any of his abilities, including response time. B) will have difficulty on tests of remote memory because this typically decreases with age. C) may take a little longer to respond, but his general knowledge and abilities should not have declined. D) will have had a decrease in his response time because of language loss and a decrease in general knowledge.

C) may take a little longer to respond, but his general knowledge and abilities should not have declined. Page: 72. The aging process leaves the parameters of mental status mostly intact. There is no decrease in general knowledge and little or no loss in vocabulary. Response time is slower than in youth. It takes a bit longer for the brain to process information and to react to it. Recent memory, which requires some processing is somewhat decreased with aging, but remote memory is not affected.

77. During an assessment, the nurse knows that expected assessment findings in the normal adult lung include the presence of: A) adventitious sounds and limited chest expansion. B) increased tactile fremitus and dull percussion tones. C) muffled voice sounds and symmetrical tactile fremitus. D) absent voice sounds and hyperresonant percussion tones.

C) muffled voice sounds and symmetrical tactile fremitus. Pages: 429-430. Normal lung findings include symmetric chest expansion, resonant percussion tones, vesicular breath sounds over the peripheral lung fields, muffled voice sounds, and no adventitious sounds.

61. A woman comes to the clinic and states, "I've been sick for so long! My eyes have gotten so puffy, and my eyebrows and hair have become coarse and dry." The nurse will assess for other signs and symptoms of: A) cachexia. B) Parkinson's syndrome. C) myxedema. D) scleroderma.

C) myxedema. Pages: 276-277. Myxedema (hypothyroidism) is a deficiency of thyroid hormone that, when severe, causes a nonpitting edema or myxedema. The patient will have a puffy edematous face especially around eyes (periorbital edema), coarse facial features, dry skin, and dry, coarse hair and eyebrows. See Table 13-4, Abnormal Facial Appearances with Chronic Illnesses, for descriptions of the other responses.

104. While examining a patient, the nurse observes abdominal pulsations between the xiphoid and umbilicus. The nurse would suspect that these are: A) pulsations of the renal arteries. B) pulsations of the inferior vena cava. C) normal abdominal aortic pulsations. D) increased peristalsis from a bowel obstruction.

C) normal abdominal aortic pulsations. Pages: 538-539. Normally, one may see the pulsations from the aorta beneath the skin in the epigastric area, particularly in thin persons with good muscle wall relaxation.

114. During an assessment of an 80-year-old patient, the nurse notices the following: inability to identify vibrations at the ankle and to identify position of big toe, slower and more deliberate gait, and slightly impaired tactile sensation. All other neurologic findings are normal. The nurse should interpret that these findings indicate: A) cranial nerve dysfunction. B) lesion in the cerebral cortex. C) normal changes due to aging. D) demyelinization of nerves due to a lesion.

C) normal changes due to aging. Page: 629. Some aging adults show a slower response to requests, especially for those calling for coordination of movements. The findings listed are normal in the absence of other significant abnormal findings. The other responses are incorrect.

83. The nurse knows that auscultation of fine crackles would most likely be noticed in: A) a healthy 5-year-old child. B) a pregnant woman. C) the immediate newborn period. D) association with a pneumothorax.

C) the immediate newborn period. Pages: 436-437. Fine crackles are commonly heard in the immediate newborn period as a result of the opening of the airways and clearing of fluid. Persistent fine crackles would be noticed with pneumonia, bronchiolitis, or atelectasis.

74. During an examination, the patient states he is hearing a buzzing sound and says that it is "driving me crazy!" The nurse recognizes that this symptom indicates: A) vertigo. B) pruritus. C) tinnitus. D) cholesteatoma.

C) tinnitus. Pages: 328-329. Tinnitus is a sound that comes from within a person; it can be a ringing, crackling, or buzzing sound. It accompanies some hearing or ear disorders.

38. The nurse is unable to palpate the right radial pulse on a patient. The best action would be to: A) auscultate over the area with a fetoscope. B) use a goniometer to measure the pulsations. C) use a Doppler device to check for pulsations over the area. D) check for the presence of pulsations with a stethoscope.

C) use a Doppler device to check for pulsations over the area. Page: 120. Doppler devices are used to augment pulse or blood pressure measurements. Goniometers measure joint range of motion. A fetoscope is used to auscultate fetal heart tones. Stethoscopes are used to auscultate breath, bowel, and heart sounds.

3. A nurse is taking complete health histories on all of the patients attending a wellness workshop. On the history form, one of the written questions asks, "You don't smoke, drink, or take drugs, do you?" This question is an example of: A) talking too much. B) using confrontation. C) using biased or leading questions. D) using blunt language to deal with distasteful topics.

C) using biased or leading questions. Page: 36 This is an example of using leading or biased questions. Asking, "You don't smoke, do you?" implies that one answer is "better" than another. If the person wants to please someone, he or she is either forced to answer in a way corresponding to their implied values or is made to feel guilty when admitting the other answer.

81. When auscultating the lungs of an adult patient, the nurse notes that over the posterior lower lobes low-pitched, soft breath sounds are heard, with inspiration being longer than expiration. The nurse interprets that these are: A) sounds normally auscultated over the trachea. B) bronchial breath sounds and are normal in that location. C) vesicular breath sounds and are normal in that location. D) bronchovesicular breath sounds and are normal in that location.

C) vesicular breath sounds and are normal in that location. Pages: 428-429. Vesicular breath sounds are low-pitched, soft sounds with inspiration being longer than expiration. These breath sounds are expected over peripheral lung fields where air flows through smaller bronchioles and alveoli.

The results of a patient's recent endoscopy indicate the presence of peptic ulcer disease (PUD). Which of the following teaching points should the nurse provide to the patient in light of his new diagnosis? You'll need to drink at least two to three glasses of milk daily. B."It would likely be beneficial for you to eliminate drinking alcohol." C. Many people find that a minced or pureed diet eases their symptoms of PUD. D. Your medications should allow you to maintain your present diet while minimizing symptoms

CORRECT ANSWER: B Although there is no specific recommended dietary modification for PUD, most patients find it necessary to make some sort of dietary modifications to minimize symptoms. Milk may exacerbate PUD and alcohol is best avoided because it can delay healing

A nurse provides feeding instructions to a mother of an infant diagnosed with gastroesophageal reflux disease. To assist in reducing the episodes of emesis, the nurse tells the mother to:1) Provide less frequent, larger feedings. 2) Burp the infant less frequently during feedings. 3) Thin the feedings by adding water to the formula. 4) Thicken the feedings by adding rice cereal to the formula

CORRECT: 4) Thicken the feedings by adding rice cereal to the formula. Rationale: GERD is backflow of gastric contents into the esophagus as a result of relaxation or incompetence of the lower esophageal or cardiac sphincter. Small, more frequent feedings with frequent burping often are prescibed in the treatment of GER. Feedings thickened with rice cereal may reduce episodes of emesis. If thickened formula is used, cross-cutting of the nipple may be required

Gastritis

Can be scattered or localized, cellular changes, distribution of lesions, can be errosive or non-errosive

• Patients receiving antibiotics are susceptible to

Clostridium difficile (C. difficile), which is a serious bacterial infection.

How is H pylori treated?

Combo of proton pump inhibitor like Oemprazole (prilosec) or lansoprazol (Prevacid) PLUS!!!! 2 Abx (usually amoxicillin and clarithromycin) for 10-14 days. -Successful 80% of time -Need to confirm cure if ulcer was complicated by bleeding, perforation, or obstruction

A client with inflammatory bowel disease undergoes an ileostomy. On the first day after surgery, the nurse notes that the client's stoma appears dusky. How should the nurse interpret this finding?"a) The ostomy bag should be adjusted. b) Blood supply to the stoma has been interrupted. c) An intestinal obstruction has occurred. d) This is a normal finding 1 day after surgery

Correct Answer: (B), Blood supply to the stoma has been interrupted An ileostomy stoma forms as the ileum is brought through the abdominal wall to the surface skin, creating an artificial opening for waste elimination. The stoma should appear cherry red, indicating adequate arterial perfusion. A dusky stoma suggests decreased perfusion. The nurse should interpret this finding as an indication that the stoma's blood supply is interuppted, which may lead to tissue damage or necrosis. A dusky stoma isn't a normal finding 1 day after surgery. Adjusting the ostomy bag wouldn't affect stoma color, which depends on blood supply to the area. An intestinal obstruction also wouldn't change stoma color

The nurse is planning to teach a client with GERD about substances that will increase the LES pressure.Which item shoud the nurse include on this list.1. Coffee 2. Chocolate 3. Fatty Foods 4. Nonfat MIlk

Correct Answer: Nonfat MIlkFoods that will increase LES pressure will decrease reflux and lessen the symptoms of GERD. The food that will increase LES pressure is nonfat milk. The other substances listed decrease LES pressure, thus increasing reflux symptoms. Aggravating substances include the others listed and alcohol

The male client tells the nurse he has been experiencing "heartburn" at night that awakens him. Which assessment question should the nurse ask? A. How much weight have you gained recently? B. What have you done to alleviate the heartburn? C. Do you consume many milk and dairy products? D Have you been around anyone with a stomach virus

Correct answer is B,Most clients with GERD have been self medicating with over-the counter medications prior to seeking advice from a healthcare provider. It is important to know what the client has been using to treat the problem

When assessing the client with the diagnosis of peptic ulcer disease, which physical examination should the nurse implement first? Auscultate the client's bowel sounds in all four quadrants. 2.Palpate the abdominal area for tenderness. 3.Percuss the abdominal borders to identify organs. 4.Assess the tender area progressing to nontender

Correct answer: #1. Auscultation should be used prior to palpa-tion or percussion when assessing the abdomen. If the nurse manipulates the abdomen, the bowel sounds can be altered and give false information

The nurse is performing an admission assessment on a client diagnosed with gastroesophageal reflux disease (GERD). Which signs and symptoms would indicate GERD? 1. Pyrosis, water brash, and flatulence 2. Weight loss, dysarthria, and diarrhea 3. Decreased abdominal fat, proteinuria, and constipation 4. Mid-epigastric pain, positive H. pylori test, and melena

Correct answer: 1 (pyrosis, water brash, and flatulence)1. Pyrosis is heartburn, water brash is the feeling of saliva secretion as a result of reflux, and flatulence is gas—all symptoms of GERD 2. Gastroesophageal reflux disease does not cause weight loss 3. There is no change in abdominal fat, no proteinuria (the result of a filtration problem inthe kidney), and no alteration in bowel elimination for the client diagnosed with GERD 4. Mid-epigastric pain, a positive H. pylori test, and melena are associated with gastric ulcer disease

The client with a hiatal hernia chronically experiences heartburn following meals. The nurse planc to teach the client to avoid which action because it is contraindicated with hiatal hernia?1. Lying recumbent following meals 2. Taking in small, frequent, bland meals 3. Raising the head of the bed on 6-inch blocks 4. Taking H2-receptor antagonist medication

Correct answer: 1Laying recumbant following meals or at night will cause reflux and pain. Relief is usually achieved with the intake of small, bland meals, use of H2 receptor antagonists and antacids, and elevation of the thorax after meals and during sleep

Which of the following types of gastritis ic associated with Helicobacter pylori and duodenal ulcers? 1. Erosive (hemorrhagic) gastritis 2. Fundic gland gastritis (type A) 3. Antral gland gastritis (type B) 4. Aspiring-induced gastric ulcer

Correct answer: Antral gland gastritis ( type B). Rationale: Antral gland gastritis is the most common form of gastritis and is associated with Helicobacter pylori and duodenal ulcers

What are less common causes of ulcer disease?

Crohn's, Zollinger-Ellison syndrome (gastrin-secreting tumor of pancrease or duodenum), or idiopathic

A morbidly obese patient has undergone Roux-en-Y gastric bypass surgery. In planning postoperative care, the nurse anticipates that the patient a. may have severe diarrhea early in the postoperative period b. will not be allowed to ambulate for 1 or 2 days postoperatively c. will require nasogastric suction until healing of the incision occurs d. may have only liquids orally, and in very limited amounts, during the early postoperative period

D

A nursing intervention that is most appropriate to decrease post-operative edema and pain following an inguinal herniorrhaphy is: a. applying a truss to the hernia site b. allowing the patient to stand to void c. supporting the incision during coughing d. applying a scrotal support with ice bag

D

A patient is admitted to the hospital with a diagnosis of diarrhea with dehydration. The nurse recognized that increased peristalsis resulting in diarrhea can be related to: a. sympathetic inhibition b. mixing and propulsion c. sympathetic stimulation d. parasympathetic stimulation c

D

A patient reports having dry mouth and asks for some liquid to drink. The nurse reasons that this symptom can most likely be attributed to a common adverse effect of which of the following medications? A) a. Digoxin (Lanoxin) B) b. Cefotetan (Cefotan) C) c. Famotidine (Pepcid) D) d. Promethazine (Phenergan)

D

A patient with anorexia nervosa shows signs of malnutrition. During initial refeeding, the nurse carfully assesses the patient for: a. hyperkalemia b. hypoglycemia c. hypercalcemia d. hypophosphatemia

D

An optimal teaching plan for an outpatient with stomach cancer receiving radiation therapy should include information about a. cancer support groups, alopecia, and stomatitis b, avitaminosis, ostomy care, and community resources c. prosthetic devices, skin conductance, and grief counseling d. wound and skin care, nutrition, drugs, and community resources

D

During the assessment of a patient with acute abdominal pain, the nurse should: a. perform deep palpation before auscultation b. obtain blood pressure and pulse rate to determine hypervolemic changes c. auscultate bowel sounds because hyperactive bowel sounds suggest paralytic ileus d. measure body temperature because an elevated temperature may indicate an inflammatory or infectious process

D

The best nutritional therapy plan for a person who is obese is: a. the Zone diet b. the Atkins diet c. Sugar busters d. foods from the basic food groups

D

The family of a patient newly diagnosed with hepatitis A asks the nurse what they can do to prevent becoming ill themselves. Which of the following responses by the nurse is most appropriate? A) a. "The hepatitis vaccine will provide immunity from this exposure and future exposures." B) b. "I am afraid there is nothing you can do since the patient was infectious before admission." C) c. "You will need to be tested first to make sure you don't have the virus before we can treat you." D) d. "An injection of immunoglobulin will need to be given to prevent or minimize the effects from this exposure."

D

The health care provider orders lactulose for a patient with hepatic encephalopathy. The nurse will monitor for effectiveness of this medication for this patient by assessing which of the following? A) a. Relief of constipation B) b. Relief of abdominal pain C) c. Decreased liver enzymes D) d. Decreased ammonia levels

D

The nurse asks a 68-year-old patient scheduled for colectomy to sign the operative permit as directed in the physician's preoperative orders. The patient states that the physician has not really explained well what is involved in the surgical procedure. Which of the following is the most appropriate action by the nurse? A) a. Ask family members whether they have discussed the surgical procedure with the physician. B) b. Have the patient sign the form and state the physician will visit to explain the procedure before surgery. C) c. Explain the planned surgical procedure as well as possible, and have the patient sign the consent form. D) d. Delay the patient's signature on the consent and notify the physician about the conversation with the patient.

D

The nurse explains to the patient with Vincent's infection that treatment will include: a. smallpox vaccinations b. viscous lidocaine rinses c. amphotericin B suspension d. topical application of antibiotics

D

The nurse explains to the patient with acute pancreatitis that the most common pathogenic mechanism of the disorder is: a. cellular disorganization b. overproduction of enzymes c. lack of secretion of enzymes d. autodigestion of the pancreas

D

The nurse explains to the patient with gastroesophageal reflux disease that this disorder: a. results in acid erosion and ulceration of the esophagus caused by frequent vomiting b. will require surgical wrapping or repair of the pyloric sphincter to control the symptoms c. is the protrusion of a portion of the stomach into the esophagus through an opening in the diaphragm d. often involves relation of the lower esophageal sphincter, allowing stomach contents to back up into the esophagus

D

The nurse is caring for a 68-year-old patient admitted with abdominal pain, nausea, and vomiting. The patient has an abdominal mass and a bowel obstruction is suspected. The nurse auscultating the abdomen listens for which of the following types of bowel sounds that is consistent with the patient's clinical picture? A) a. Low pitched and rumbling above the area of obstruction B) b. High pitched and hypoactive below the area of obstruction C) c. Low pitched and hyperactive below the area of obstruction D) d. High pitched and hyperactive above the area of obstruction

D

The nurse is caring for a postoperative patient with a colostomy. The nurse is preparing to administer a dose of famotidine (Pepcid) when the patient asks why the medication was ordered since the patient does not have a history of heartburn or gastroesophageal reflux disease (GERD). Which of the following would be the most appropriate response by the nurse? A) a. "This will prevent air from accumulating in the stomach, causing gas pains." B) b. "This will prevent the heartburn that occurs as a side effect of general anesthesia." C) c. "The stress of surgery is likely to cause stomach bleeding if you do not receive it." D) d. "This will reduce the amount of HCl in the stomach until the nasogastric tube is removed, and you can eat a regular diet again."

D

The nurse is performing a focused abdominal assessment of a patient who has been recently admitted. In order to palpate the patient's liver, the nurse will palpate the patient's A) a. Left lower quadrant. B) b. Left upper quadrant. C) c. Right lower quadrant. D) d. Right upper quadrant.

D

The nurse is preparing to administer a dose of bisacodyl (Dulcolax). In explaining the medication to the patient, the nurse would state that it acts in which of the following ways? A) a. Increases bulk in the stool B) b. Lubricates the intestinal tract to soften feces C) c. Increases fluid retention in the intestinal tract D) d. Increases peristalsis by stimulating nerves in the colon wall

D

The nurse is preparing to administer a scheduled dose of docusate sodium (Colace) when the patient complains of an episode of loose stool and does not want to take the medication. Which of the following is the appropriate action by the nurse? A) a. Write an incident report about this untoward event. B) b. Attempt to have the family convince the patient to take the ordered dose. C) c. Withhold the medication at this time and try to administer it later in the day. D) d. Chart the dose as not given on the medical record and explain in the nursing progress notes.

D

The nurse is reviewing the laboratory test results for a 71-year-old patient with metastatic lung cancer. The patient was admitted with a diagnosis of malnutrition. Serum albumin level is 4.0 g/dl and prealbumin is 10 mg/dl. The nurse should draw which of the following conclusions? A) a. The albumin level is normal, and therefore the patient does not have protein malnutrition. B) b. The albumin level is increased, which is a common finding in patients with cancer who have malnutrition. C) c. Both the serum albumin and prealbumin levels are reduced, consistent with the admitting diagnosis of malnutrition. D) d. Although the serum albumin level is normal, the prealbumin level more accurately reflects the patient's nutritional status.

D

The nurse is teaching the patient and family about possible causative factors for peptic ulcers. The nurse explains that ulcer formation is: a. caused by a stressful lifestyle and other acid producing factors such as h.pylori b. inherited within families and reinforced by bacterial spread of staph aureus in childhood c. promoted by factors that tend to cause oversecretion of acid such as excess dietary fats, smoking and h.pylori d. promoted by a combination of possible factors that may result in erosion of the gastric mucosa, including certain drugs and alcohol

D

The nurse who inserted a nasogastric tube for a 68-year-old patient with suspected bowel obstruction should write which of the following priority nursing diagnoses on the patient's problem list? A) a. Anxiety related to nasogastric tube placement B) b. Abdominal pain related to nasogastric tube placement C) c. Risk for deficient knowledge related to nasogastric tube placement D) d. Altered oral mucous membrane related to nasogastric tube placement

D

The nurse would instruct the patient to do which of the following to best enhance the effectiveness of a daily dose of docusate sodium (Colace)? A) a. Take a dose of mineral oil at the same time. B) b. Add extra salt to food on at least one meal tray. C) c. Ensure dietary intake of 10 g of fiber each day. D) d. Take each dose with a full glass of water or other liquid.

D

The nurse would question the use of which of the following cathartic agents in a patient with renal insufficiency? A) a. Bisacodyl B) b. Lubiprostone C) c. Cascara sagrada D) d. Milk of magnesia

D

The nursing management of the patient with cholecystitis associated with cholelithiasis is based on the knowledge that: a. shock-wave therapy should be tried initially b. once gallstones are removed,they tend not to recur c. the disorder can be successfully treated with oral bile salts that dissolve gallstones d. laparoscopic cholecystectomy is the treatment of choice in most patient who are symptomatic

D

The obesity classification that is most often associated with cardiovascular health problems is: a. primary obesity b. secondary obesity c. gynoid fat distribution d. android fat distribution

D

The optimal way to administer medications via a feeding tube is to: a. use only sustained-release medications b. pour medications into the enteral formula c. grind all the pills together and dilute with water d. remover powder from a gelatin capsule and dilute with water

D

This bariatric surgical procedure involves creating a stoma and gastric pouch that is reversible and no malabsorption occurs. What surgical procedure is this? a. vertical gastric banding b. biliopacreatic diversion c. roux-en-y gastric bypass d. adjustable gastric banding

D

When planning care for a patient with cirrhosis, the nurse will give highest priority to which of the following nursing diagnoses? A) a. Imbalanced nutrition: less than body requirements B) b. Impaired skin integrity related to edema, ascites, and pruritus C) c. Excess fluid volume related to portal hypertension and hyperaldosteronism D) d. Ineffective breathing pattern related to pressure on diaphragm and reduced lung volume

D

Which of the following should a patient be taught after a hemorrhoidectomy: a. take mineral oil prior to bedtime b. eat a low-fiber diet to rest the colon c. administer oil-retention enema to empty the colon d. use prescribed pain medication before a bowel movement

D

9. A patient has been taking orlistat (Xenical) for several months as part of a weight loss program which also includes a low-fat diet. Which of these data obtained by the nurse indicate that a change in therapy may be needed? a. The patient complains of abdominal bloating after meals. b. The patient has lost 31 lb (14 kg) of the original 65 lb (30 kg) goal. c. The patient frequently has liquid stools. d. The patient is pale and has many bruises.

D Rationale: Because orlistat blocks the absorption of fat-soluble vitamins, the patient may not be receiving an adequate amount of vitamin K, resulting in a decrease in clotting factors. Abdominal bloating and liquid stools are common side effects of orlistat and indicate that the nurse should remind the patient that fat in the diet may increase these side effects. The patient's weight loss indicates that the diet and medication are successful and should be continued. Cognitive Level: Analysis Text Reference: p. 981 Nursing Process: Evaluation NCLEX: Physiological Integrity

11. A few months after bariatric surgery, the patient tells the nurse, "My skin is hanging in folds. I think I need cosmetic surgery." Which response by the nurse is most appropriate? a. "The skin folds will gradually disappear once most of the weight is lost." b. "The important thing is that your weight loss is improving your health." c. "Perhaps you would like to talk to a counselor about your body image." d. "Cosmetic surgery is certainly a possibility once your weight has stabilized."

D Rationale: Reconstructive surgery may be used to eliminate excess skin folds after at least a year has passed since the surgery. Skin folds may not disappear over time, especially in older patients. The response, "The important thing is that your weight loss is improving your health" ignores the patient's concerns about appearance and implies that the nurse knows what is important. Whereas it may be helpful for the patient to talk to a counselor, it is more likely to be helpful to know that cosmetic surgery is possible later. Cognitive Level: Application Text Reference: p. 986 Nursing Process: Implementation NCLEX: Physiological Integrity

3. A patient who has been consistently following a diet and exercise program and successfully losing 1 lb weekly for several months is weighed at the clinic and has not lost any weight for the last month. The nurse should first a. review the diet and exercise guidelines with the patient. b. instruct the patient to weigh weekly and record the weights. c. discuss the possibility that the patient has reached a temporary weight loss plateau. d. ask the patient whether there have been any changes in exercise or diet patterns.

D Rationale: The initial nursing action should be assessment of any reason for the failure to lose weight. The other actions may be needed, but further assessment is required before any interventions are planned or implemented. Cognitive Level: Application Text Reference: p. 976 Nursing Process: Assessment NCLEX: Health Promotion and Maintenance

72. The nurse is taking the history of a patient who may have a perforated eardrum. What would be an important question in this situation? A) "Do you ever notice ringing or crackling in your ears?" B) "When was the last time you had your hearing checked?" C) "Have you ever been told you have any type of hearing loss?" D) "Was there any relationship between the ear pain and the discharge you mentioned?"

D) "Was there any relationship between the ear pain and the discharge you mentioned?" Pages: 327-328. Typically with perforation, ear pain occurs first, stopping with a popping sensation, and then drainage occurs.

18. When providing culturally competent care, nurses must incorporate cultural assessments into their health assessments. Which statement is most appropriate to use when initiating an assessment of cultural beliefs with an elderly American Indian patient? A) "Are you of the Christian faith?" B) "Do you want to see a medicine man?" C) "How often do you seek help from medical providers?" D) "What cultural or spiritual beliefs are important to you?"

D) "What cultural or spiritual beliefs are important to you?" Page: 17. The nurse needs to assess the cultural beliefs and practices of the patient. American Indians may seek assistance from a medicine man or shaman, but the nurse should not assume this. An open-ended question regarding cultural and spiritual beliefs is best used initially when performing a cultural assessment.

44. When assessing the quality of a patient's pain, the nurse should ask which question? A) "When did the pain start?" B) "Is the pain a stabbing pain?" C) "Is it a sharp pain or dull pain?" D) "What does your pain feel like?"

D) "What does your pain feel like?" Page: 164. To assess the quality of a person's pain, have the patient describe the pain in his or her own words.

112. When assessing muscle strength, the nurse observes that a patient has complete range of motion against gravity with full resistance. What Grade should the nurse record using a 0 to 5 point scale? A) 2 B) 3 C) 4 D) 5

D) 5 Pages: 578-579. Complete range of motion against gravity is normal muscle strength and is recorded as Grade 5 muscle strength.

118. During the history of a 78-year-old man, his wife states that he occasionally has problems with short-term memory loss and confusion: "He can't even remember how to button his shirt." In doing the assessment of his sensory system, which action by the nurse is most appropriate? A) The nurse would not do this part of the examination because results would not be valid. B) The nurse would perform the tests, knowing that mental status does not affect sensory ability. C) The nurse would proceed with the explanations of each test, making sure the wife understands. D) Before testing, the nurse would assess the patient's mental status and ability to follow directions at this time.

D) Before testing, the nurse would assess the patient's mental status and ability to follow directions at this time. The nurse should ensure validity of the sensory system testing by making sure the patient is alert, cooperative, comfortable, and has an adequate attention span. Otherwise, the nurse may obtain misleading and invalid results.

54. A 65-year-old man with emphysema and bronchitis has come to the clinic for a follow-up appointment. On assessment, the nurse might expect to see which assessment finding? A) Anasarca B) Scleroderma C) Pedal erythema D) Clubbing of the nails

D) Clubbing of the nails Pages: 217-218. Clubbing of the nails occurs with congenital cyanotic heart disease, neoplastic, and pulmonary diseases. The other responses are assessment findings not associated with pulmonary diseases.

92. During an assessment of a healthy adult, where would the nurse expect to palpate the apical impulse? A) Third left intercostal space at the midclavicular line B) Fourth left intercostal space at the sternal border C) Fourth left intercostal space at the anterior axillary line D) Fifth left intercostal space at the midclavicular line

D) Fifth left intercostal space at the midclavicular line Pages: 473-474. The apical impulse should occupy only one intercostal space, the fourth or fifth, and it should be at or medial to the midclavicular line.

19. When planning a cultural assessment, the nurse should include which component? A) Family history B) Chief complaint C) Medical history D) Health-related beliefs

D) Health-related beliefs Pages: 19-20. Health-related beliefs and practices are one component of a cultural assessment. The other items reflect other aspects of the patient's history.

119. In assessing a 70-year-old patient who has had a recent cerebrovascular accident, the nurse notices right-sided weakness. What might the nurse expect to find when testing his reflexes on the right side? A) Lack of reflexes B) Normal reflexes C) Diminished reflexes D) Hyperactive reflexes

D) Hyperactive reflexes Hyperreflexia is the exaggerated reflex seen when the monosynaptic reflex arc is released from the influence of higher cortical levels. This occurs with upper motor neuron lesions (e.g., a cerebrovascular accident). The other responses are incorrect

2. During an interview, the nurse states, "You mentioned shortness of breath. Tell me more about that." Which verbal skill is used with this statement? A) Reflection B) Facilitation C) Direct question D) Open-ended question

D) Open-ended question Page: 32 The open-ended question asks for narrative information. It states the topic to be discussed but only in general terms. The nurse should use it to begin the interview, to introduce a new section of questions, and whenever the person introduces a new topic.

103. The nurse suspects that a patient has a distended bladder. How should the nurse assess for this condition? A) Percuss and palpate in the lumbar region. B) Inspect and palpate in the epigastric region. C) Auscultate and percuss in the inguinal region. D) Percuss and palpate the midline area above the suprapubic bone.

D) Percuss and palpate the midline area above the suprapubic bone. Pages: 539-540. Dull percussion sounds would be elicited over a distended bladder, and the hypogastric area would seem firm to palpation.

102. Which structure is located in the left lower quadrant of the abdomen? A) Liver B) Duodenum C) Gallbladder D) Sigmoid colon

D) Sigmoid colon Page: 530. The sigmoid colon is located in the left lower quadrant of the abdomen.

35. The nurse is preparing to assess a patient's abdomen by palpation. How should the nurse proceed? A) Avoid palpation of reported "tender" areas because this may cause the patient pain. B) Quickly palpate a tender area to avoid any discomfort that the patient may experience. C) Begin the assessment with deep palpation, encouraging the patient to relax and take deep breaths. D) Start with light palpation to detect surface characteristics and to accustom the patient to being touched.

D) Start with light palpation to detect surface characteristics and to accustom the patient to being touched. Pages: 115-116. Light palpation is performed initially to detect any surface characteristics and to accustom the person to being touched. Tender areas should be palpated last, not first.

57. The nurse is assessing for clubbing of the fingernails and would expect to find: A) a nail base that is firm and slightly tender. B) curved nails with a convex profile and ridges across the nail. C) a nail base that feels spongy with an angle of the nail base of 150 degrees. D) an angle of the nail base of 180 degrees or greater with a nail base that feels spongy.

D) an angle of the nail base of 180 degrees or greater with a nail base that feels spongy. Pages: 217-218. The normal nail is firm at its base and has an angle of 160 degrees. In clubbing, the angle straightens to 180 degrees or greater and the nail base feels spongy.

121. While the nurse is taking the history of a 68-year-old patient who sustained a head injury 3 days earlier, he tells the nurse that he is on a cruise ship and is 30 years old. The nurse knows that this finding is indicative of: A) a great sense of humor. B) uncooperative behavior. C) inability to understand questions. D) decreased level of consciousness.

D) decreased level of consciousness. Pages: 660-661. A change in consciousness may be subtle. The nurse should notice any decreasing level of consciousness, disorientation, memory loss, uncooperative behavior, or even complacency in a previously combative person. The other responses are incorrect.

106. The physician comments that a patient has abdominal borborygmi. The nurse knows that this term refers to: A) a loud continuous hum. B) a peritoneal friction rub. C) hypoactive bowel sounds. D) hyperactive bowel sounds.

D) hyperactive bowel sounds. Pages: 539-540. Borborygmi is the term used for hyperperistalsis when the person actually feels his or her stomach growling.

70. An ophthalmic examination reveals papilledema. The nurse is aware that this finding indicates: A) retinal detachment. B) diabetic retinopathy. C) acute-angle glaucoma. D) increased intracranial pressure.

D) increased intracranial pressure. Pages: 319-320. Papilledema, or choked disk, is a serious sign of increased intracranial pressure, which is caused by a space-occupying mass such as a brain tumor or hematoma. This pressure causes venous stasis in the globe, showing redness, congestion, and elevation of the optic disc, blurred margins, hemorrhages, and absent venous pulsations. Papilledema is not associated with the conditions in the other responses.

117. When the nurse asks a 68-year-old patient to stand with feet together and arms at his side with his eyes closed, he starts to sway and moves his feet farther apart. The nurse would document this finding as a(n): A) ataxia. B) lack of coordination. C) negative Homans' sign. D) positive Romberg sign.

D) positive Romberg sign. Page: 638. Abnormal findings for Romberg test include swaying, falling, and widening base of feet to avoid falling. Positive Romberg sign is loss of balance that is increased by closing of the eyes. Ataxia is uncoordinated or unsteady gait. Homans' sign is used to test the legs for deep vein thrombosis.

4. During an interview, a parent of a hospitalized child is sitting in an open position. As the interviewer begins to discuss his son's treatment, however, he suddenly crosses his arms against his chest and crosses his legs. This would suggest that the parent is: A) just changing positions. B) more comfortable in this position. C) tired and needs a break from the interview. D) uncomfortable talking about his son's treatment.

D) uncomfortable talking about his son's treatment. Page: 37 Note the person's position. An open position with the extension of large muscle groups shows relaxation, physical comfort, and a willingness to share information. A closed position with the arms and legs crossed tends to look defensive and anxious. Note any change in posture. If a person in a relaxed position suddenly tenses, it suggests possible discomfort with the new topic.

22. The mother of a 16-month-old toddler tells the nurse that her daughter has an earache. What would be an appropriate response? A. "Maybe she is just teething." B. "I will check her ear for an ear infection." C. "Are you sure she is really having pain?" D. "Please describe what she is doing to indicate she is having pain."

D. "Please describe what she is doing to indicate she is having pain." Page: 60. With a very young child, ask the parent, "How do you know the child is in pain?" Pulling at ears alerts parent to ear pain. The statements about teething and questioning whether the child is really having pain do not explore the symptoms, which should be done before a physical examination.

24. The nurse is preparing to do a functional assessment. Which statement best describes the purpose of a functional assessment? A. It assesses how the individual is coping with life at home. B. It determines how children are meeting developmental milestones. C. It can identify any problems with memory the individual may be experiencing. D. It helps to determine how a person is managing day-to-day activities.

D. It helps to determine how a person is managing day-to-day activities. Page: 67. The functional assessment measures how a person manages day-to-day activities. The other answers do not reflect the purpose of a functional assessment.

21. In recording the childhood illnesses of a patient who denies having had any, which note by the nurse would be most accurate? A. Patient denies usual childhood illnesses. B. Patient states he was a "very healthy" child. C. Patient states sister had measles, but he didn't. D. Patient denies measles, mumps, rubella, chickenpox, pertussis, and strep throat.

D. Patient denies measles, mumps, rubella, chickenpox, pertussis, and strep throat. Page: 51. Childhood illnesses include measles, mumps, rubella, chickenpox, pertussis, and strep throat. Avoid recording "usual childhood illnesses" because an illness common in the person's childhood may be unusual today (e.g., measles).

23. A 5-year-old boy is being admitted to the hospital to have his tonsils removed. Which information should the nurse collect before this procedure? A. The child's birth weight B. The age at which he crawled C. Whether he has had the measles D. Reactions to previous hospitalizations

D. Reactions to previous hospitalizations Assess how the child reacted to hospitalization and any complications. If the child reacted poorly, he or she may be afraid now and will need special preparation for the examination that is to follow. The other items are not significant for the procedure.

Hep b

DNA

Purpose of Gastrointestinal Intubation

Decompress the stomach, Lavage the stomach, Diagnose GI disorders, Administer medications and feeding, to treat an obstruction, to compress a bleeding site, to aspirate contents for analysis

Hiatal Hernia

Diaphragmatic weakness throught which a portion of the stomach protrudes into the thoracic cavity. Most are asymptomatic but some may have daily symptoms of GERD

Potential complications of Tube feeding

Diarrhea, N/V, Gass/Bloating/Cramping, Dumping syndrome, Aspriation pneumonia, tube displacement, tube obstruction, nasophayngeal irritation, hypergylcemia, dehydration

Dysphagia

Difficulty swallowing, most common symptom of esophageal cancer, usually indicates a narrowing of teh esophagus

Chronic Panceatitis (causes)

Duct blockage; Toxins; Alcohol abuse; chronic fibrosis.

Manifestations of GERD

Dyspepsia, Regurgitation, Eructation, Flatulence, Water Brash

Sypmtoms of Diverticula

Dysphagia, Regurgiation, Nocturnal cough, Halitosis

"The nurse is caring for a 68 year old patient admitted with abdominal pain, nausea, and vomiting. The patient has an abdominal mass and a bowel obstruction is suspected. The nurse auscultating the abdomen listens for which of the following types of bowel sounds that is consistent with the patient's clinical picture? "A. low pitched and rumbling above the area of obstruction B. High pitched and hypoactive below the area of obstruction C. low pitched and hyperactive below the area of obstruction (D). high pitched and hyperactive above the area of obstruction

Early in intestinal obstruction, the patient's bowel sounds are hyperactive adn high pitched. This occurs because peristaltic action increases to "push past" the area of obstruction. As the obstruction becomes complete, bowel sounds decrease and finally become absent

Complications of Achalasia

Esophageal candiasis, Lower esophageal diverticula, Airway obstruction, Aspiration Pneumonia

Sliding Hernia

Esophagogastric junction and a portion of the fundus move into the thorax through the esophageal hiatus. Generally moves freely and slided into and out of the chest during position or intr-abdominal pressure. Volvulus and obstruction rarely occur. GERD is a major complication.

Cholesterol stones

Europe & Americas; advanced age, F sex hormones, bc pills, rapid wieght changes, pregnancy. Radiolucent. Can grow several cm

Factor that Decrease LES Pressure

Fatty Foods, Caffeinated beverages, Chocolate, Cirtus fruits, tomatoes, smoking, peppermint, alcohol

Complications Esophageal Cancer

Fistula formation btwn the trachea and esophagus, Abscess, Respiratory

What is Dysphagia?

Food sticking in the esophagus and holding for 30 secs or a min.

What is the result of malabsorption of B12?

Gastric atrophy with development of pernicious anemia

What is the most common cause of gastric CA?

H. pylori *gastric CA occurs many years into the course of ulcer disease when H.pyl destroys the parietal cells leading to achlorhydria. WHO estimates 90% of gastric CA eliminated if H. pylori eradicated, however, it increases the rate of esophageal CA

What is the most common cause ulcer disease in developing world?

H. pylori BY FAR! Declining in developed, possibly better sanitation

What are the most common causes of ulcer disease?

H. pylori and NSAIDS ( more than 95% combined)

coinfection

HDV and HBV same time -mild infection w immunity 90% of time(resolution)

Potential Complications of gastric surgery

Hemorrahage Dietary deficiencies Bile reflux Dumping syndrome

What's the swallowing process?

Highly coordinated! propulsion of food from pharynx to esoph from transient relaxation of UES followed by a peristaltic wave in esoph. This ends w relaxation of LES followed by LES hypertension to prevent reflux. Takes 6-8 seconds.

How is dx made?

Hx, physical, and a normal endoscopy

How does clinical presentation of NSAID and H pylori ulcers differ?

Indistinguishable, but NSAID typically occurs after age 50 (start taking them)

What NSAID mech is most responsible for clinically significant disease?

Inhibition of prostaglandin and protective mucous/ bicard layer breakdown allowing diffusion of H+ across luminal membrane of mucosal lining cells

Methods of Administering Tube Feeding

Intermitten bolus feeding, Intermittent gravity drip, Continuous infusion, Cyclic Feeding

Post-Op Open Nissen Fundoplication

Intially the NG drainage will be dark brown but should become yellowish green within the first 8 hours, Check NGT every 4-8 hrs for proper placement, The tube shoudl be properly anchored, frrquently assess the patency of the tube

Liver stores

Iron, vitamines: ACE, DEAK, B12,

GERD

Most Common upper GI Disorder, result of backwards flow of GI contents into esophagus, causing esophageal irratation or inflammation

Diffuse Esophageal Spasm

Motor disorder of the esophagus, stress is possible factor, more common in women, Manifestation incluse difficult ot painful swallowing, chest pain

cholesterol stones risk factors

N.Europe & S. Amer-older, female sex hormones, oral contraceptives, pregnancy, obesity, rapid wt reduction

Signs of Advanced gastric cancer

N/V, obstructive sypmtoms, iron deficiency anemia, palable epigastric mass, enlarges lymph nodes, weakness and fatigue, progressive weight loss

Do you need to scope it there are no alarm sx?

NO, start the meds

What is the most common cause of upper GI bleeding?

NSAIDS!!!

What does endoscopy reveal?

Normal stomach lining (Endoscopy is required for dx)

Odynophagia

Painful swallowing, A steady, dull, substernal pain that may radiate, occurs most often when the patient drinks cold liquids, the presence of severe or persistent pain oftern indicates tumor invasion of the mediastinal structure

The nurse is teaching the patient and family about possible causative factors for peptic ulcers. The nurse explains that ulcer formation is: (Source: Medical-Surgical Nursing, LDH p. 1004) a) caused by stressful lifestyle and other acid producing factors such as H. pylori. b) inherited within families and reinforced by bacterial spread of Staphylococcus aureus in childhood. c) promoted by factors that tend to cause oversecretion of acid, such as excess dietary fats, smoking, and H. pylor. d) promoted by a combination of possible factors that may result in erosion of the gastric mucosa, including certain drugs and alcohol

Peptic ulcers develop only in the presence of an acidic environment. However, an excess of hydrochloric acid (HCl) may not be necessary for ulcer development. The back-diffusion of HCl into the gastric mucosa results in cellular destruction and inflammation. Histamine is released from the damaged mucosa, resulting in vasodilation and increased capillary permeability and further secretion of acid and pepsin. A variety of agents (i.e., certain infections, medications, and lifestyle factors) can damage the mucosal barrier

Potential Complications of Tube feeding

Pneumothorax, Clotted or displaced catheter, Sepsis, Hypergylcemia, Rebound hypoglycemia, Fluid overload

Colorectal Carcinoma

Polypoid tumors. In situ lesions. Napkin ring constrictions-anular lesions on distal colon. Lumen narrowed, proximal bowel distended.

Intrahepatic portal hypertension

Portal hypertension characterized by hepaticcirrhosis, massive fatty change of liver, diffuse fibrosing granulomatous disease, shistosomiasis,nodular regenerative hyperplasia

Post-Op Gastric Cancer

Prevention of atelactasis, paralytic ileus, wound infection, monitor for complications that are speficic to gastric surgery. Asess the NGT for patency, monitor amount of blood drainage from tube

Cirrhosis

Progressive fibrosis, shunts blood around the parenchyma. Causes loss of fenestrations in sinusoidal epithelial cells. Among top 10 causes of death. Linked w alcoholism & viral hepatitis. Presents as pitting adema (cankles).

Proximal and distal stomach have distinct roles in handling food. What do proximal and distal do?

Proximal stomach receives and stores food, then transfers the emulsified chyme created by the GRINDING movements of DISTAL creates to the duodenum with slow propulsive waves

Posthepatic p. hypert.

R. sided heart failure, constrictive pericarditis & hepatic V outflow obstruction.

Manifestations of Acute Gastritis

Rapid onset of epigastric pain or discomfort, N/V, Hematemesis, Gastric hemorrhage, dyspepsia, anorexia

Achalasia

Rare esophageal motility disorder that results from loss of nerve impulses to the smooth muscle, exact cause is unknown, have chronic to progressive dysphagia

Interventions for patient with gastric surgery

Reduce anxiety Pain relief Patient teaching Individualized nutritional care and support

What is dyspepsia?

Refers to a burning epigastric pain that is typically relieved by food or antacids (Heartburn is acid some place it shouldn't be)

Manifestations of Esophageal Tumors

Regurgitation, Vomiting, Halitosis, Chronic Hiccups, Chronic cough, Hoarseness

Portal Hypertension

Resistance of portal blood flow.

Describe the gastric mucosal barier

Resting lumen pH = 1-2 Intracellular pH = 7.3 Without barrier, HCL in lumen would back diffuse and damage the mucosa **It's a BICARD ENRICHED WATER LAYER

Pigmented stones

Rural Asia; chronic hemolytic syndromes, biliary infection, GI dx ie. crohns dx. Show on Xray. smaller- <1.5cm

Acute Gastritis

S/S thickened, reddened mucosa membrane w/ prominent rugae or folds, various degress of mucosal necrosis and inflammatory reaction

Barrett's Esophagus

S/S usually related to acid reflux and may include frequent heartburn, difficulty swallowing food, chest pain, black tarry stools, vomiting blood.

Esophageal Diverticula

Sacs resulting from the herniation of esophageal mucosa and sub mucosa into the surrounding tissue. May develop anywhere along the length of the esophagus. Zenker's Diverticula is most common

Advantages of Entral Feedings

Safe and cost effective, perserve GI integrity, preserve the normal sequence of intestinal and hepatic metabolism, maintain fat metabolism and lipoprotien synthesis, maintain normal insulin and glucagon ratios

Maintaining Normal Bowel Elimmination w/ TF

Selection of TF formula, prevent contamination of TF, maintain proper nutritional intake, assess for reason for diarrhea and obtain treatment as needed, administer TF slowly, avoid cold TF

Common Causes of Esophageal Perforation

Straining, Seizures, Trauma, Foreign objects, Instrument or tubes, Chemical injury, Complications of surgery, Ulcers

The nurse teaches the client about an anti-ulcer diet. Which of the following statements by the client indicates to the nurse that dietary teaching was successful? 1. "I must eat bland foods to help my stomach heal." 2. "I can eat most foods, as long as they don't bother my stomach." 3. "I cannot eat fruits and vegetables because they cause too much gas." 4. "I should eat a low-fiber diet to delay gastric emptying -

The answer is 2.

Post-OP care for Esophageal Cancer

The number of sites of all the incisions and drains, The placement of a jejunostomy tube for entral feedings, the need for chest tubes if pleural space is entered, the purpose of NG tube, need for IV infusion

Why are acid suppression main treatment?

There are no drugs which effectively modulate LES pressure Proton Pump inhibitors are most effective for healing since they lower the acid the most

Maintaining Fluid balance

Use infusion pump, monitor indicatiors of fluid balance and electrolyte levels, I&O, weights, and monitor blood glucose levels

What is Non-ulcer Dyspepsia?

Visceral hyperalgesia of the stomach

What testing with solid food dysphagia that's not accute?

Visualize by x-ray or better endoscopy in case bx needed.

What is Barrett's disease?

When squamous cells of the esophagus are exposed to acid and become columnar cells. They are unstable and become malignant (adenocarcinoma)

How is NSAID ulcer treated?

Withdraw NSAID and proton pump inhibitor

The inflammation of Crohn's disease involves __

__all layers of the bowel wall with segments of normal bowel occurring between diseased portions—the so-called skip lesions.

The primary symptoms of ulcerative colitis are __

__bloody diarrhea and abdominal pain.

• With Crohn's disease, diarrhea and __

__colicky abdominal pain are common symptoms. If the small intestine is involved, weight loss and nutritional problems are common because of malabsorption. Patients may have systemic symptoms such as fever.

• Bowel surgery can__

__disrupt nerve and vascular supply to the genitals. Radiation therapy, chemotherapy, and medications can also alter sexual function.

• Surgery is the treatment of choice __

__for hernias and prevents strangulation.

• Clinical manifestations are usually __

__nonspecific and differ by the location of the lesion within the colon. Most report hematochezia, melena, abdominal pain, and/or changes in bowel habits.

The cause of diverticulitis is unknown:__

__the majority of patients with diverticular disease are asymptomatic. Those with symptoms typically have abdominal pain, bloating, flatulence, and/or changes in bowel habits.

Although there is sometimes mild inflammation in the terminal ileum__

__ulcerative colitis is a disease of the colon and rectum.

• With a bowel obstruction there is retention of fluid in the intestine and peritoneal cavity__

__which can result in a severe reduction in circulating blood volume and lead to hypotension and hypovolemic shock.

What is heartburn?

a burning pain that starts in the epigastrium and migrates retrosternally often culminating in an acid taste in the throat. Heartburn too, is typically relieved by food or antacids.

• Bowel sounds that are diminished or absent in a quadrant may indicate

a complete bowel obstruction, acute peritonitis, or paralytic ileus.

• Constipation can be defined as

a decrease in the frequency of bowel movements from what is "normal" for the individual; hard, difficult-to-pass stools; a decrease in stool volume; and/or retention of feces in the rectum.

Dyspepsia or heartburn are manifestations of what?

a group of diseases referred to as the "acid-peptic disorders" -gastritis -gastric and duodenal ulcer disease -gastroexophageal reflux disease (GERD)

What is Peptic Ulcer Disease?

a heterogeneous group of disorders consisting of gastritis, gastric ulcer and duodenal ulcer

• Peritonitis results from

a localized or generalized inflammatory process of the peritoneum.

o A nonmechanical obstruction may result from

a neuromuscular or vascular disorder. Vascular disorders are due to interference in the blood supply to the intestine.

What is Pepsinogen?

a proenzyme that is cleaved to pepsin at acid pH and pepsin hydrolyzes peptide bonds in protein. Does 10% of protein digestion and perpetuates ulcer crators. Inactivated at pH >3.5

• An anal fissure is

a skin ulcer or a crack in the lining of the anal wall that is caused by trauma, local infection, or inflammation.

• A pilonidal sinus is

a small tract under the skin between the buttocks in the sacrococcygeal area.

• Accurate diagnosis and management require

a thorough history, physical examination, and laboratory testing. Treatment depends on the cause.

What causes GERD?

acid entering the esophagus Affects 15%

What do the Acid-peptic disorders have in common?

acid induced injury to the mucosa of the esophagus, duodenum or stomach.

Stress ulcers

acute gastric mucosal lesions occuring after medical crisis or trauma

most common cause of chronic

alcohol abuse

ductal obstruction

alcohol increases protein concentration in pancreatic juice -form ductal plugs->calcify adn form calculi

mucinous cystic neoplasm

almost always women can be malignant usually in body or tail slow growing and painless -filled w thick mucin and lined by columnar mucin cells

o Five major classes of medications are used to treat IBD:

aminosalicylates, antimicrobials, corticosteroids, immunosuppressants, and biologic and targeted therapy.

other enzymes secreted by pancreas

amylase lipase nuclease -in active form but need ions present in bile for optimal activity

• An anal fistula is

an abnormal tunnel leading from the anus or rectum, often into the vagina, or outside skin. They are often accompanied by infection and incontinence.

• Celiac disease is

an autoimmune disease characterized by damage to the small intestinal mucosa from the ingestion of wheat, barley, and rye in genetically susceptible individuals.

• Diarrhea is most commonly defined as

an increase in stool frequency or volume and an increase in the looseness of stool.

• Gastroenteritis is

an inflammation of the mucosa of the stomach and small intestine.

• Acute abdominal pain is a symptom of many different types of tissue injury

and can arise from damage to abdominal or pelvic organs and blood vessels.

• Other care for peritonitis includes

antibiotics, nasogastric suction, analgesics, and intravenous (IV) fluid administration.

• Crohn's disease can occur

anywhere in the GI tract from the mouth to the anus, but occurs most commonly in the terminal ileum and colon.

What is though to cause non-ulcer dyspepsia?

appears that the stomach muscles relax poorly and are hypersensitive to stretching.

• If the causative agent of gastroenteritis is identified

appropriate antibiotic and antimicrobial drugs are given. Symptomatic nursing care is given for nausea, vomiting, and diarrhea.

• Gastrointestinal stromal tumors (GISTs)

are a rare form of cancer that originates in cells found in the wall of the GI tract.

pigment stones risk factors

asia, rural more than urban chronic hemoltic syndromes, biliary infection, GI disorders (Crohn's)

Major Goal for a Patient w/ Gastrostomy

attaining optimal level of nutrition, preventing infection, maintaining skin integrity, enhancing coping skills, adjusting to changes in the body image, acquiring knowledge of and skills in self care, prevent complications

Why don't H2 receptor antagonists have lasting efficacy?

because of tolerance. loses suppressing ability. Proton pump inhibs and surgery only way to prevent recurrence

hepatic tumors

benign neoplasm malignant tumor hepatocellular carcinoma cholangiocarcinoma

serous cystadenoma

benign, surrounded by cuboidal cells with clear straw colored fluid -25% of cysts -2* as common in women -ab pain a symptom

epithelial cells lining the smallest ducts produce

bicarbonate -high pH help neutralize acidic chyme from stomach

• Causes of malabsorption include

biochemical or enzyme deficiencies, bacterial proliferation, disruption of small intestine mucosa, disturbed lymphatic and vascular circulation, and surface area loss.

pigment stones are

black brown stones rarely greter than 1.5cm diameter -radio-opaque

• Beginning at age 50

both men and women at average risk for developing CRC should have screening tests done to detect both polyps and cancer

ascities

build up & leakage of blood pressure- consequence of p.hypert.

How are particles larger than 1 mm cleared?

by Housekeeper waves that typically occur every 2 hours in the fasting state. (they are the reason one can see whole corn kernels in stool)

primary acinar cell injury

caused by viruses(mumps) drugs and traumas

CCK

causes bile to be releases from the Lv.

What happens as eating stops?

causes the intragastric pH to decline leading to a decrease in gastrin release --thus reducing stimulus for acid secretion, and a release of somatostatin which inhibits acid secretion with return to baseline levels

3 phased of pancreatic secreation

cephalic gastric intestinal

superinfection

chronic HBV gets HDV leads to chronic hepatitis cirrhosis 80% of time

• There are numerous causes of constipation. Clinical presentation varies from

chronic discomfort to an acute event, often depending on cause.

Atrophic A type

chronic gastritis characterized by a total loss of funday glands, minimal inflammation, thinning of the gastric mucosa and intestinal metaplasia

Factor that increase risk of PUD

cigarette smoking, family hx, alcohol use, incidence increases with age

• Anorectal abscesses are

collections of perianal pus resulting from an infection in the anal glands.

Hep d

combined w B can form "super infectinon"

Esophageal vericies

consequence of p hypert.

• Risk factors for fecal incontinence include

constipation, diarrhea, obstetrical trauma, and fecal impaction.

What type of muscle is UES and what does it do?

cricopharyngeus (tonic at rest preventing air to GI and reflux to pharynx)

o Mechanical obstruction is a

detectable occlusion of the intestinal lumen. Most intestinal obstructions occur in the small intestine.

• SBS is treated with

dietary changes, supplements, and antidiarrheal medications.

• Hemorrhoids are

dilated hemorrhoidal veins. They may be internal (occurring above the internal sphincter) or external (occurring outside the external sphincter).

• Inflammation of the appendix with obstruction results in

distention, venous engorgement, and the accumulation of mucus and bacteria, which can lead to gangrene and perforation.

• Surgery for peritonitis is usually indicated to

drain purulent fluid and repair damage.

autodigestion of pancreas by activated enzymes

duct obstruction primary acinar cell injury defective transport of enzymes

4 hypotheses of chronic

ductal obstruction toxic-metabolic oxidative necrosis-fibrosis

US 65% of pancreatitis

due to excessive alcohol use

What is the major cause of GERD?

dysfunction of LES manifested by heartburn

• Treatment of lactase deficiency consists of

eliminating lactose from the diet by avoiding milk and milk products and/or replacement of lactase with commercially available preparations.

pancreas is composed of both

endocrine and exocrine gland cells

Congestive splenomegaly

enlarged spleen- consequence of portal hypert.

what activates trypsinogen to trypsin

enterokinase (brush border enzyme)

acinar cells produces

enzymes

What is Non-ulcer dyspepsia?

epigastric pain, fullness and/or bloating which is not associated with mucosal damage as seen by endoscopy (beginning of visceral hyperalgesia - pain message to brain when there is no cause)

• Emergency management of abdominal trauma focuses on

establishing a patent airway and adequate breathing, fluid replacement, and prevention of hypovolemic shock.

Hepatic encephalopathy

excessive bilirubin in blood-> accumulation of blood toxins. Consequence of portal hypert.

• SBS is characterized by

failure to maintain protein-energy, fluid, electrolyte, and micronutrient balances on a standard diet; it may be accompanied by diarrhea.

hepatic steatosis

fatty liver -lipid accumulation in hepatocyte -macrovesicualr globules -liver is larger and softer and appears yellow and greasy -reversible is alcohol is stopped

Hep a

fecal-oral transport

• Classic signs of celiac disease include

foul-smelling diarrhea, steatorrhea, flatulence, abdominal distention, and symptoms of malnutrition.

kuppfer cells

found in LV sinusoids. Remove bacteria and dead blood cells.

• Diarrhea can result from alterations in

gastrointestinal motility, increased secretion, and decreased absorption.

Peptic Ulcer Disease

generic term for ulcers or breaks int eh mucosal lining in the GI tract

• Three factors necessary for the development of celiac disease (gluten intolerance) are

genetic predisposition, gluten ingestion, and an immune-mediated response.

alpha cells produce

glucagon

• During an acute exacerbation of IBD nursing care is focused on

hemodynamic stability, pain control, fluid and electrolyte balance, and nutritional support.

Hep e

high maternal mortality

HEV

high mortality rate in pregnant women 20%

What is LES ?

highly speciaized muscle, also tonic at rest preventing reflux from stomach to esophagus

• Treatment of appendicitis is

immediate surgical removal.

• Crohn's disease and ulcerative colitis are

immunologically related disorders that are referred to as "inflammatory bowel disease" (IBD).

• Malabsorption results from

impaired absorption of fats, carbohydrates, proteins, minerals, and vitamins.

• Prevention and treatment of fecal incontinence may be managed by

implementing a bowel training program.

• Ulcerative colitis usually starts

in the rectum and moves in a continual fashion toward the cecum.

• Lactase deficiency is a condition

in which the lactase enzyme is deficient or absent.

• Common injuries of the abdomen include

include lacerated liver, ruptured spleen, pancreatic trauma, mesenteric artery tears, diaphragm rupture, urinary bladder rupture, great vessel tears, renal injury, and stomach or intestine rupture.

• The overall goals are that the patient with constipation will

increase dietary intake of fiber and fluids; increase physical activity; have the passage of soft, formed stools; and not have any complications, such as bleeding hemorrhoids.

• Major risk factors of colorectal cancer (CRC) include

increasing age, family or personal history of colorectal cancer (CRC), colorectal polyps, and IBD.

Early signs of Gastric cancer

indigestion, abdominal discomfort, feeling of fullness, epigastric, back and restrosternal pain

• All cases of acute diarrhea should be considered

infectious until the cause is known. Strict infection control precautions are necessary.

Diverticulitis is

inflammation of the diverticula, resulting in complications such as perforation, abscess, fistula formation, and bleeding.

Type-A-non-erosive Chronic Gastritis

inflammation of the glands, as well as the fundus and body of the stomach, thought to be automimmune, genetic link to disease

chronic pancreatitis

inflammation w destruction of exocrine cells, fibrosis, and eventual destruction of endocrine cells

HAV spread by

ingestion of contaminated water or food shed in stool 2-3wks before and 1 week after jaundice

• Types of hernias include

inguinal, femoral, and ventral or incisional. Diagnosis is based on the history and physical examination relative to the type of hernia.

beta cells produce

insulin

• IBS is a chronic functional disorder characterized by

intermittent and recurrent abdominal pain and stool pattern irregularities (diarrhea, constipation, or both).

• Common causes of chronic abdominal pain include

irritable bowel syndrome (IBS), diverticulitis, peptic ulcer disease, chronic pancreatitis, hepatitis, cholecystitis, pelvic inflammatory disease, and vascular insufficiency.

pancreatic islets

islets of langerhans contain alpha and beta cells tiny clusters that produce hormones

Hep c

leads to cirrohsis 85% of time

pancreas location

lies below stomach and is mostly retroperitoneal -duodenum surrounds head of pancreas

• Celiac disease is treated with

lifelong avoidance of dietary gluten. Wheat, barley, oats, and rye products must be avoided.

insulin hypoglycemic agent

lower blood sugar levels -influences protein and fat metabolism -causes sugar to enter muscle and fat cells

bile salts

makes cholesterol soluble in bile -when cholsterol [] >capacity-?nucleates into solid monohydrate crystal

• The causes of intestinal obstruction can be classified as

mechanical or nonmechanical and can be partial or complete.

Purpose of Entral Feeding

meet nutritional requirements when oral intake is inadequate or not possible, and the GI tract is functioning

• IBD is characterized by

mild to severe acute exacerbations that occur at unpredictable intervals over many years.

pigment stones

mixtures of calcium salts and unconjugated bilirubin -increased amts from intravascualr hemolysis

Open Fundoplication

more bleeding and infection and potental complications, elevate head of bed at least 30 degrees, assist patient out of bed ASAP

Albumin

most abundant plasma protein, 60% of the total protein, made by the liver, plays an important role in osmotic balance, contributes to the viscosity of blood, transportation of lipids/hormones/calcium..., and helps to maintain pH

Hemmorrhage

most serious complication of PUD, occurs more often with gastric ulcer

• Blunt trauma commonly occurs with

motor vehicle accidents and falls, and may not be obvious because it does not leave an open wound.

• Clinical manifestations of Gastroenteritis include

nausea, vomiting, diarrhea, abdominal cramping, and distention. Most cases are self-limiting and do not require hospitalization.

• Pain is the most common symptom of a problem. There may also be

nausea, vomiting, diarrhea, fatigue, fever, and constipation.

• Adenomatous polyps are characterized by

neoplastic changes in the epithelium and are closely linked to colorectal adenocarcinoma.

pancreatic digestive enzymes need

neutral pH for optimal function

progression

non-neoplastic epithelium-> adenoma-> invasive carcinoma will spread to omentum, stomach, spleen, adrenals, and colon -generally silent until impinges surrounding structures

• The goals for the patient with CRC include

normal bowel elimination patterns, quality of life appropriate to disease progression, relief of pain, and feelings of comfort and well-being.

Lifestyle factors associated with CRC include

obesity, smoking, alcohol, and a large intake of processed and/or red meat.

Prehepatic p.hypert.

obstructive thrombosis & stinosis of portal V. Splenomegaly also shunts blood into splenic V

Duodenal Ulcers

occur in the upper portion of the duodenum, they are deep, sharply demaracted leasions that penetrate through the mucosa and submucosa, H. pylori is associated with 90-95%, Blood group O tend to have duodenal ulcers, pain is relieved by eating

Gastric Ulcers

occur most often on the lesser curvature near the pylorus, may result from the back-diffusion of acid or dysfunction of the pyloric sphincter, gastric emypting is often delayed, associated with gastritis and gastric cancer, pain exacerbated by food, deep and penetrating

• Fecal incontinence, the involuntary passage of stool

occurs when the normal structures that maintain continence are disrupted.

defective transport of enzymes

occurs with obstruction or alcohol abuse -transport of proenzymes and lysomal hydrolases together causing enzymes to prematurely activate

What is the phase of acid secretion after food ingestion?

once ingestion starts the rise in intragastric pH releases the hormone gastrin which acts both directly on teh parietal cell and through the adjacent enterochromafin cell to release histamine, the most potent chemical stimulus of acid secretion.

What pH does the bicarb/water (mucus) layer maintain and what mediates it?

pH 5-6 and layer maintained by secretion of mucus and bicarb that is mediated by prostaglandins **THUS NSAIDS, such as ibuprofin and aspirin which inhibit prostaglandin synthesis compromise this defense mech and predispose to ulcer disease.

Assessment of Patient with Gastric Surgery

paitent and family knowledge nutritional status abdominal assessment Postoperatively assess for potential complications

• Treatment for chronic abdominal pain is comprehensive and directed toward

palliation of symptoms using nonopioid analgesics and antiemetics, as well as psychologic or behavioral therapies.

pancreatitis

pancreatic inflammation -can be mild self limiting to acute life threatening -destruction from edema and fat necrosis -cell death w severe hemorrhage

Bariatric Surgery

performed only after nonsurgical methods have failed, Selection Factors include: body weight, patient hx, and failure to lose weight using other means, absence of endocrine disorders and pyschological stability

striations of umbilicus

periumbilical caput medusae. consequence of p hypert.

• Appendicitis typically begins with

periumbilical pain, followed by anorexia, nausea, and vomiting.

• The pain of appendicitis is

persistent and continuous, eventually shifting to the right lower quadrant and localizing at McBurney's point.

What are the primary manifestations of GERD?

primarily heartburn (or chronic cough, sore throat, dental erosions, wheezing or sleep apnea, water brash

injured tissues and leukocytes release

proinflammatory cytokines (IL-1B, IL-6) -local inflammation and edema -edema causes further pressure increase on local blood flow -vascular insufficiency and ischemic injury to acinar cells

Apoferritin

protein in the liver which binds to iron in the liver to form FERRITIN

• A hernia is a

protrusion of a viscus through an abnormal opening or a weakened area in the wall of the cavity in which it is normally contained.

* Treatment of IBS is directed at

psychologic and dietary factors as well as medications to regulate output.

What is water brash?

reflux of saliva. It is a major cause of asthma

CCK

released by duodenum when a presence of protein or fat detected -stimulates pancreas

• Expected outcomes for the patient with acute abdominal pain include

resolution of the cause of the acute abdominal pain; relief of abdominal pain; freedom from complications (especially hypovolemic shock and septicemia); and normal fluid, electrolyte, and nutritional status.

• The goals of treatment for IBD include the following:

rest the bowel, control the inflammation, combat infection, correct malnutrition, alleviate stress, provide symptomatic relief, and improve quality of life.

• Diverticula are

saccular dilations or outpouchings of the mucosa that develop in the colon at points where the vasa recta penetrate the circular muscle layer.

intraductal papillary mucinous neoplasm

same as above -more common in men and involve head and ducts of pancreas

ductules

secrete bicarbonate leading away from acini

pancreatic acini

secrete digestive enzymes

What is Intrinsic factor?

secreted by Parietal cells and is essential for absorption of Vit B12.

types of cysts

serous cystadenoma mucinous cystic neoplasm intraductal papillary mucinous neoplasm

how much is too much

short term ingestion of 80g over 1-3days=mild reversible changes -daily intake greater than that- significant increase in risk for sever hepatic injury

• The patient with an ileostomy should be observed for

signs and symptoms of fluid and electrolyte imbalance, particularly potassium, sodium, and fluid deficits.

Post-OP care for Bariatric Surgery

similar to gastric resection, but greater risk due to obesity, diet is six small feeding totalling 600-800 calories per day, may require pyschosocial interventions to modify their eating habits, follow-up care, educations regarding long-term effects

HCV

small enveloped ssRNA

HAV

small non-enveloped ssRNA picornavirus

strongest risk factor

smoking

why are enzymes release in inactive form

so they do not digest the pancreas

sodium bicarbonate plus HCl in duod =

sodium chloride and carbonic acid -rise in pH to ~7.8 carbonic acid dissociates to CO2 and water -then expired by lungs -

Post-Op Laparoscopic Nissen Fundoplication

stay on soft diet for about a week, remain on antireflux medications, do not drive for a week after surgery, do not drive if taking opiod pain meds, walk every day, but no heavy lifting, wash incision with soap and water, rinse well and pat dry, report a fever above 100, n/v, or uncontrollable bloating and pain, schedule follow-up in 3-4 weeks

Symptoms of Ulcer Perforation

sudden sharp pain begin in midepigastric region, the abdomen is tender, rigid and boardlike, pt assumes fetal position to decrease tension on abdominal muscles

Patient Teaching Open Fundoplication

support incision line during coughing, incentive spirometry and deep breathing are routinely used, adequate pain control, patients with large hiatal hernias are at the highest risk for developing respiratory complications, usually have a large-bore NGT to prevent fundoplication wrap from becoming to tight.

• Fistulas are closed by

surgery or using fibrin glue. Postoperative and nursing care is the same as the patient with hemorrhoids.

• Short bowel syndrome (SBS) results from

surgical resection, congenital defect, or disease-related loss of absorption. The length and portions of small bowel affected are associated with the number and severity of symptoms.

• Familial adenomatous polyposis (FAP) is

the most common hereditary polyp disease.

• If the hernia becomes strangulated

the patient will experience severe pain and symptoms of a bowel obstruction, such as vomiting, cramping abdominal pain, and distention.

energy needs are met first

then glycogen formation -if excess glucose i blood then it is converted to fat

What is the primary defect in GERD?

transient inappropriate relaxation of LES

Hep C sources

trasnfusions IV drug use 60% sex 15%

other causes of pancreatitis

tumors of ducts medications infections(mumps) metabolic diseases (hypertriglyceridemia) acute ischemia trauma 10-20% is idiopathic

• The cause of IBS is

unknown and there are no specific findings.

How is the dx of NSAID ulcers made?

upper GI endoscopy **NOTE - people with NSAID ulcer disease don't feel anything, all of a sudden blood is coming up

Type-B Gastritis

usually affects the glands of the stomach. Most commin form of the disease, usually caused by H. Pylori, chronic local irritation and toxic effects caused by alcohol ingestion, radiation therapy, smoking

Gastric Cancer

usually begins in the glans od the stomach mucosa,

Manifestations of Chronic Gastritis

vague report of epigastric pain that is relieved by food, anorexia, N/V, intolerance of fatty and spicy foods, pernicious anemia

Hepatitis E

water borne infection -young middle aged adults 50% of cases of sporadic acute hepatitis in India -moslty self limiting

• An ostomy is used

when the normal elimination route is no longer possible.

When does PUD occur?

whenever there is an imbalance between the injurious effects of acid and pepsin, and the protective effects of mucous and bicarb. Thus an increase in acid (like with H. pylori duodenal ulcers), a decrease in mucosal defense (NSAID gastric ulcers), or a combo of the two leads to ulcers

parasympathetic NS

will cause pancreas to secret during cephalic and gastric phases

How is non-ulcer treated?

with meds that relax the stomach (anticholinergics or tricyclics) or help it empty (metoclopramide-toxic if used chronically)= help some. Antacids sometimes help.

cholesterol stones are

yellow can be several cm in diameter radio-lucent


Conjuntos de estudio relacionados

state registration of securities

View Set

World History Ch 28.5- Hitler and the Rise of Nazi Germany

View Set

Chapter 12: Demand Planning: Forecasting and Demand Management, OSCM 3001 Exam #2 Ch. 10, SCM-Chapter 15, Supply Chain Chapter 12: Demand Planning: Forecasting and Demand Management, SCM Chapter 15, SCM Chapter 12- Demand Planning: Forecasting and Ma...

View Set

Chapter 13: A House Divided, 1840-1861

View Set

Designing Studies Checkpoint 1 and 2

View Set